SlideShare a Scribd company logo
1 of 52
Dịch vụ viết thuê đề tài – KB Zalo/Tele 0917.193.864 – luanvantrust.com
Kham thảo miễn phí – Kết bạn Zalo/Tele mình 0917.193.864
ĐẠI HỌC THÁI NGUYÊN
TRƯỜNG ĐẠI HỌC KHOA HỌC
--------------

-------------
NGUYỄN THANH TÙNG
ĐA THỨC TRONG CÁC BÀI TOÁN
THI HỌC SINH GIỎI
LUẬN VĂN THẠC SĨ TOÁN HỌC
THÁI NGUYÊN -
Dịch vụ viết thuê đề tài – KB Zalo/Tele 0917.193.864 – luanvantrust.com
Kham thảo miễn phí – Kết bạn Zalo/Tele mình 0917.193.864
ĐẠI HỌC THÁI NGUYÊN
TRƯỜNG ĐẠI HỌC KHOA HỌC
--------------

-------------
NGUYỄN THANH TÙNG
ĐA THỨC TRONG CÁC BÀI TOÁN
THI HỌC SINH GIỎI
LUẬN VĂN THẠC SĨ TOÁN HỌC
Chuyên ngành: Phương pháp Toán sơ cấp
Mã số: 60 46 01 13
NGƯỜI HƯỚNG DẪN KHOA HỌC
GS.TSKH. Đặng Hùng Thắng
THÁI NGUYÊN -
Dịch vụ viết thuê đề tài – KB Zalo/Tele 0917.193.864 – luanvantrust.com
Kham thảo miễn phí – Kết bạn Zalo/Tele mình 0917.193.864
1
Mục lục
Danh sách kí hiệu 3
Mở đầu 4
Chương 1. Đa thức một biến 7
1.1 Định nghĩa và các tính chất . . . . . . . . . . . . . . . . . . . . . 7
1.1.1 Định nghĩa . . . . . . . . . . . . . . . . . . . . . . . . . 7
1.1.2 Các phép tính trên đa thức . . . . . . . . . . . . . . . . . 8
1.1.3 Các tính chất cơ bản . . . . . . . . . . . . . . . . . . . . 9
1.2 Phép chia đa thức. Ước chung lớn nhất và nhỏ nhất . . . . . . . . 11
1.2.1 Phép chia đa thức . . . . . . . . . . . . . . . . . . . . . . 11
1.2.2 Thuật toán Euclide . . . . . . . . . . . . . . . . . . . . . 11
1.3 Nghiệm của đa thức. Phương trình bậc cao . . . . . . . . . . . . . 16
1.3.1 Nghiệm của đa thức . . . . . . . . . . . . . . . . . . . . 16
1.3.2 Phương trình bậc cao . . . . . . . . . . . . . . . . . . . . 22
1.4 Đạo hàm của đa thức. Định lý Taylor . . . . . . . . . . . . . . . . 32
Chương 2. Đa thức bất khả quy 36
2.1 Đa thức bất khả quy . . . . . . . . . . . . . . . . . . . . . . . . . 36
2.1.1 Đa thức với hệ số thực và phức . . . . . . . . . . . . . . . 37
2.1.2 Đa thức bất khả quy của vành Q[x] . . . . . . . . . . . . . 40
Dịch vụ viết thuê đề tài – KB Zalo/Tele 0917.193.864 – luanvantrust.com
Kham thảo miễn phí – Kết bạn Zalo/Tele mình 0917.193.864
2
2.2 Một số bài toán điển hình . . . . . . . . . . . . . . . . . . . . . . 42
Chương 3. Một số chủ đề khác 46
3.1 Đa thức nhiều biến . . . . . . . . . . . . . . . . . . . . . . . . . 46
3.2 Đa thức đối xứng . . . . . . . . . . . . . . . . . . . . . . . . . . 49
3.3 Phương trình hàm đa thức . . . . . . . . . . . . . . . . . . . . . . 53
3.4 Đa thức Chebyshev . . . . . . . . . . . . . . . . . . . . . . . . . 56
3.4.1 Định nghĩa - Tính chất . . . . . . . . . . . . . . . . . . . 57
3.4.2 Một số bài toán chọn lọc . . . . . . . . . . . . . . . . . . 58
Kết luận 63
Tài liệu tham khảo 64
Dịch vụ viết thuê đề tài – KB Zalo/Tele 0917.193.864 – luanvantrust.com
Kham thảo miễn phí – Kết bạn Zalo/Tele mình 0917.193.864
3
Danh sách kí hiệu
Z vành số nguyên
Q trường số hữu tỷ
R trường số thực
C trường số phức
R vành
F trường
R[x] vành đa thức với hệ số trên vành R
deg P(x) bậc của đa thức P(x)
.
.
đa thức Q(x) là ước của đa thức P(x)
P(x) . Q(x), Q(x) j P(x)
gcd(P(X); Q(X)) ước chung lớn nhất của P(X) và Q(X)
a b (mod p) a đồng dư với b theo modulo p
åi
m
=1 ai ký hiệu tổng a1 + a2 + + am
Õi
m
=1 bi ký hiệu tích b1b2 bm
Dịch vụ viết thuê đề tài – KB Zalo/Tele 0917.193.864 – luanvantrust.com
Kham thảo miễn phí – Kết bạn Zalo/Tele mình 0917.193.864
4
Mở đầu
Đa thức là một đối tượng quan trọng của Toán học cả về mặt lý thuyết cũng
như ứng dụng. Đối với Toán học phổ thông, học sinh làm quen với các phép
toán trên đa thức (cộng trừ nhân chia), giải các phương trình bậc nhất, bậc hai
và một số dạng phương trình bậc cao. Trong các kỳ thi học sinh giỏi quốc gia
và quốc tế, chủ đề đa thức cũng được khai thác sâu hơn với các bài toán hay
và tương đối khó về phương trình đại số bậc cao, phương trình hàm đa thức,
đathức bất khả quy, tính chia hết của đa thức . . .
Các bài toán nâng cao về đa thức xuất hiện cũng khá nhiều trong các
tạp chí toán học cho học sinh khá giỏi (như Tạp chí Toán học và Tuổi trẻ,
Kvant, Crux,. . . ). Tuy nhiên hiện nay có ít các tài liệu về tiếng Việt trình bày
một cách hệ thống cả lý thuyết và bài tập về đa thức, với định hướng bồi
dưỡng học sinh giỏi Toán và bồi dưỡng giáo viên dạy chuyên Toán.
Mục tiêu của luận văn là tìm hiểu một cách đầy đủ những kết quả quan trọng
của đa thức có nhiều ứng dụng trong Toán phổ thông. Trên cơ sở đó, phân loại và
hệ thống hoá (theo dạng cũng như phương pháp giải) các bài tập nâng cao về đa
thức đã có cũng như sáng tác, bổ sung thêm những bài toán mới.
Chúng tôi rất cố gắng để luận văn này trở thành một tài liệu tham
khảo tốt, thiết thực phục vụ cho việc giảng dạy học sinh giỏi và bồi
dưỡng giáo viên. Thông qua việc viết luận văn học viên sẽ mở rộng nâng
cao hiểu biết về đa thức, hình thành các kỹ năng giải các bài toán khó về
đa thức, kỹ năng tìm kiếm thu thập chọn lọc các thông tin.
Nội dung của luận văn được trình bày trong ba chương như sau:
Dịch vụ viết thuê đề tài – KB Zalo/Tele 0917.193.864 – luanvantrust.com
Kham thảo miễn phí – Kết bạn Zalo/Tele mình 0917.193.864
5
Chương 1. Đa thức một biến. Trong chương này chúng tôi sẽ trình
bày ngắn gọn về định nghĩa và các tính chất của đa thức. Các vấn
đề nền tảng về phép chia đa thức, ước - bội, nghiệm và phương
trình bậc cao, đạo hàm và khai triển Taylor sẽ được trình bày.
Chương 2. Đa thức bất khả quy. Đa thức bất khả quy là một trong những chủ
đề trọng tâm của lý thuyết các đa thức. Nó vừa mang tính chất lý thuyết, vừa
mang tính ứng dụng, đặc biệt là các bài tập nâng cao trong các đề thi có tính
chất tuyển chọn. Chương này chúng tôi tập trung nghiên cứu các đa thức bất
khả quy trên các vành (trường) số quen biết của toán học sơ cấp.
Chương 3. Một số chủ đề khác. Chương này dành để nghiên cứu một số
vấn đề nâng cao của lý thuyết đa thức, mà mục đích của nó là để hiểu
biết sâu sắc hơn lý thuyết, đồng thời là nền tảng cho các ứng dụng. Các
vấn đề được quan tâm trong chương này là các đa thức nhiều biến, đa
thức đối xứng, phương trình hàm đa thức và đa thức Chebyshev.
Luận văn này được thực hiện tại Trường Đại học Khoa học - Đại học
Thái Nguyên và hoàn thành với sự hướng dẫn của GS.TSKH. Đặng Hùng
Thắng (Trường ĐHKHTN - ĐHQG Hà Nội). Tác giả xin được bày tỏ lòng
biết ơn chân thành và sâu sắc tới người hướng dẫn khoa học của mình,
người đã đặt vấn đề nghiên cứu, dành nhiều thời gian hướng dẫn và tận
tình giải đáp những thắc mắc của tác giả trong suốt quá trình làm luận văn.
Tác giả xin trân trọng cảm ơn Ban Giám hiệu Trường Đại học Khoa học - Đại
học Thái Nguyên, Ban Chủ nhiệm Khoa Toán–Tin, cùng các giảng viên đã tham
gia giảng dạy, đã tạo mọi điều kiện tốt nhất để tác giả học tập và nghiên cứu.
Tác giả muốn gửi những lời cảm ơn tốt đẹp nhất tới tập thể lớp Cao
học Toán khóa 9 (2015-2017) đã động viên và giúp đỡ tác giả rất nhiều
trong suốt quá trình học tập.
Nhân dịp này, tác giả cũng xin chân thành cảm ơn Sở Giáo dục và Đào tạo Hải
Dịch vụ viết thuê đề tài – KB Zalo/Tele 0917.193.864 – luanvantrust.com
Kham thảo miễn phí – Kết bạn Zalo/Tele mình 0917.193.864
6
Phòng, Ban Giám hiệu và các đồng nghiệp ở Trường THPT Hùng Vương đã tạo
điều kiện cho tác giả hoàn thành tốt nhiệm vụ học tập và công tác của mình.
Cuối cùng, tác giả muốn dành những lời cảm ơn đặc biệt nhất đến bố
mẹ và đại gia đình đã luôn động viên và chia sẻ những khó khăn để tác
giả hoàn thành tốt luận văn này.
Thái Nguyên, ngày 02 tháng 11 năm 2017
Tác giả
Nguyễn Thanh Tùng
Dịch vụ viết thuê đề tài – KB Zalo/Tele 0917.193.864 – luanvantrust.com
Kham thảo miễn phí – Kết bạn Zalo/Tele mình 0917.193.864
7
Chương 1
Đa thức một biến
1.1 Định nghĩa và các tính chất
Phần này chúng tôi sẽ trình bày ngắn gọn nhất có thể lý thuyết các đa
thức một biến. Những chi tiết hơn có thể tham khảo Lê Thị Thanh Nhàn
[5] hoặc Nguyễn Văn Mậu [3].
1.1.1 Định nghĩa
Giả sử R là một vành giao hoán có đơn vị.
Định nghĩa 1.1.1. Biểu thức có dạng
anx
n
+ an 1x
n 1
+ : : : + a1x + a0 với an 6= 0
trong đó an; an 1; : : : ; a1; a0 là những phần tử thuộc vành R, được gọi
là một đa thức trên vành R.
Trong định nghĩa này, ai được gọi là các hệ số của đa thức, hệ số an được
gọi là hệ số bậc cao nhất của đa thức, số tự nhiên n được gọi là bậc của đa
thức, ký hiệu là deg P(x), x được gọi là ẩn, hay biến hay đối số của đa thức, an
được gọi là hệ số cao nhất, a0 được gọi là hệ số tự do của đa thức.
Nếu ai = 0 với i = 1; 2; : : : ; n 1 và a0 6= 0 thì ta có bậc của đa thức là
không.
Nếu ai = 0 với i = 1; 2; : : : ; n thì f (x) = 0, ta gọi đa thức này là đa thức không.
Dịch vụ viết thuê đề tài – KB Zalo/Tele 0917.193.864 – luanvantrust.com
Kham thảo miễn phí – Kết bạn Zalo/Tele mình 0917.193.864
8
Nói chung người ta không định nghĩa bậc của đa thức không nhưng ta
coi bậc của nó là ¥.
Hai đa thức f và g được gọi là bằng nhau, và viết f = g, nếu chúng
cùng là đa thức không, hoặc cả hai khác đa thức không, đồng thời deg f
= deg g và các hệ số tương ứng bằng nhau.
Tập hợp tất cả các đa thức lấy hệ số trong vành R được ký hiệu là
R[x], và được gọi là vành đa thức trên R. Khi R là một trường, thì vành
R[x] là một vành giao hoán có đơn vị.
Với lý do là ứng dụng lý thuyết đa thức trong các bài thi học sinh giỏi, hay nói
chung là các kỳ thi có tính chất tuyển chọn, luận văn này thường xét R là Z, Q, R,
hoặc C, khi đó các đa thức thuộc Z[x], Q[x], R[x], hoặc C[x] được gọi tên lần lượt
là các đa thức nguyên, đa thức hữu tỷ, đa thức thực, hoặc đa thức phức.
1.1.2 Các phép tính trên đa thức
Cho hai đa thức
f (x) = anx
n
+ an 1x
n 1
+ : : : + a1x + a0;
g(x) = bnx
n
+ bn 1x
n 1
+ : : : + b1x + b0:
Ta định nghĩa các phép tính số học như sau
Phép cộng
f (x)+g(x) = (an +bn)x
n
+(an 1 +bn 1)x
n 1
+: : :+(a1 +b1)x+(a0 +b0):
Phép trừ
f (x) g(x) = (an bn)x
n
+(an 1 bn 1)x
n 1
+: : :+(a1 b1)x (a0
b0):
Phép nhân
f (x)g(x) = c2nx
2n
+ c2n 1x
2n 1
+ : : : + c1x + c0
Dịch vụ viết thuê đề tài – KB Zalo/Tele 0917.193.864 – luanvantrust.com
Kham thảo miễn phí – Kết bạn Zalo/Tele mình 0917.193.864
9
trong đó
ck = a0bk + a1bk 1 + : : : + akb0 với k = 0; 1; : : : ; n:
1.1.3 Các tính chất cơ bản
Định lí 1.1.2. Giả sử F một trường. Với hai đa thức f (x) và g(x) thuộc
vành đa thức F[x], luôn tồn tại cặp đa thức q(x) và r(x) duy nhất thuộc
vành đa thức F[x] để
f (x) = g(x) q(x) + r(x);
với deg r(x) < deg g(x).
Ta gọi q(x) và r(x) lần lượt là đa thức thương (hoặc thương) và đa
thức dư (hay dư) trong phép chia f (x) cho g(x).
Nếu r(x) = 0 thì ta nói f (x) chia hết cho g(x), hay g(x) chia hết f (x) hay f
(x) là bội của g(x) hay g(x) là ước của f (x). Ta sẽ kí hiệu là f
..
. g hay g j f .
Giả sử f (x) = anx
n
+ an 1x
n 1
+ : : : + a1x + a0 là một đa thức thuộc
vành đa thức R[x]. Xét phần tử a 2 R bất kỳ. Khi đó
f (a) = ana
n
+ an 1a
n 1
+ : : : + a1a + a0
được gọi là giá trị của đa thức f (x) tại a.
Nếu f (a) = 0 thì a được gọi là một nghiệm của đa thức f (x). Nếu tồn tại
. k k+1
.
(x a) a) thì
k 2 N; k > 1 sao cho f (x) . nhưng f (x) không chia hết cho (x
a được gọi là nghiệm bội k của đa thức f (x). Đặc biệt, k = 1 thì a được gọi là
nghiệm đơn, k = 2 thì a được gọi là nghiệm kép.
Bài toán tìm nghiệm của đa thức
f (x) = anx
n
+ an 1x
n 1
+ : : : + a1x + a0 với an 6= 0:
trong vành R được gọi là giải phương trình đại số bậc n trong R.
Dịch vụ viết thuê đề tài – KB Zalo/Tele 0917.193.864 – luanvantrust.com
Kham thảo miễn phí – Kết bạn Zalo/Tele mình 0917.193.864
10
Định lí 1.1.3. Giả sử F một trường, a 2 F và f (x) 2 F[x]. Dư của phép
chia f (x) cho (x a) chính là f (a).
Định lí 1.1.4 (Định lý Bézout). Phần tử a 2 F là nghiệm của đa thức f (x)
2 F[x] khi và chỉ khi f (x) chia hết cho (x a).
Định lý sau đây cho ta một đánh giá về số nghiệm của một đa thức thực.
Định lí 1.1.5. Một đa thức thực bậc n đều có không quá n nghiệm thực.
Định lý này có một số hệ quả sau đây.
Hệ quả 1.1.6. Đa thức có vô số nghiệm là đa thức không.
Hệ quả 1.1.7. Nếu một đa thức có bậc không vượt quá n mà nhận cùng một
giá trị tại n + 1 điểm khác nhau của ẩn thì đa thức đó là đa thức hằng.
Hệ quả 1.1.8. Hai đa thức có bậc không vượt quá n mà nhận cùng một
giá trị thỏa mãn bằng nhau tại n + 1 giá trị khác nhau của ẩn thì hai đa
thức đó đồng nhất bằng nhau.
Trong trường hợp đa thức phức, ta có kết quả sau đây về số lượng nghiệm của
nó.
Định lí 1.1.9. Một đa thức phức bậc n có đúng n nghiệm tính cả bội.
Cuối cùng của mục này, ta sẽ trình bày không chứng minh một kết
quả về dạng biểu diễn của các đa thức thực.
Định lí 1.1.10. Bất kỳ đa thức thực f (x) 2 R[x] nào có bậc n và hệ số bậc cao nhất
an 6= 0 có thể phân tích một cách duy nhất (không tính thứ tự) thành các nhân tử
m
f (x) = an Õ(x di)
s
Õ (x
2
+ bkx + ck)
i=1 k=1
với di; bk; sk 2 R, 2s + m = n và b
2
k 4ck < 0 với m; n 2 N.
Dịch vụ viết thuê đề tài – KB Zalo/Tele 0917.193.864 – luanvantrust.com
Kham thảo miễn phí – Kết bạn Zalo/Tele mình 0917.193.864
11
Trong lý thuyết đa thức, một phần rất quan trọng đó là khảo sát các
nghiệm hữu tỷ và nguyên của một đa thức. Ta có định lý sau đây.
Định lí 1.1.11. Xét đa thức nguyên f (x) 2 Z[x] có dạng
f (x) = a0x
n
+ a1x
n 1
+ : : : + an 1x + an; a0 6= 0:
Nghiệm hữu tỷ nếu có x = q
p
với gcd(p; q) = 1 thì p là ước của hệ số tự
do và q là ước của hệ số bậc cao nhất, tức là p j an, q j a0.
Hệ quả 1.1.12. Xét đa thức nguyên f (x) 2 Z[x] có dạng chuẩn tắc, tức là
hệ số bậc cao nhất là 1. Khi đó mỗi nghiệm hữu tỷ nếu có của đa thức f
(x) đều là nghiệm nguyên.
1.2 Phép chia đa thức. Ước chung lớn nhất và nhỏ nhất
1.2.1 Phép chia đa thức
Định nghĩa 1.2.1. Một đa thức d(x) chia hết hai đa thức f (x) và g(x) gọi
là ước chung của f (x) và g(x). Nếu d(x) là một ước chung chia hết cho
mọi ước chung khác của hai đa thức f (x) và g(x) đúng thì ta gọi d(x) là
ước chung lớn nhất của f (x) và g(x) .
Rõ ràng các ước chung lớn nhất sai khác hằng số, để bảo đảm tính
duy nhất ta có thể quy ước chọn ước chung lớn nhất dạng chuẩn tắc (hệ
số cao nhất bằng 1). Ta kí hiệu là d(x) = ( f (x); g(x)) = gcd(( f (x); g(x)):
1.2.2 Thuật toán Euclide
Ta chia liên tiếp
f (x) = g(x) q(x) + r(x)
g(x) = r(x) q1(x) + r1(x)
Dịch vụ viết thuê đề tài – KB Zalo/Tele 0917.193.864 – luanvantrust.com
Kham thảo miễn phí – Kết bạn Zalo/Tele mình 0917.193.864
12
r(x) = r1(x) q2(x) + r2(x)
: : :
rk 2(x) = rk 1(x) qk(x) + rk(x)
rk 1(x) = rk(x qk+1(x):
Khi đó ( f (x); g(x)) = rk (x) với rk (x) = c rk(x) đa thức có hệ số bậc cao nhất là 1
(người ta thường gọi những đa thức này là đa thức chuẩn tắc hay đa thức monic).
Kết quả nếu d(x) = ( f (x); g(x)) thì tồn tại hai đa thức u(x); v(x) 2 R[x]
để có biểu diễn
f (x) u(x) + g(x) v(x) = d(x):
Hơn nữa ta có thể chọn deg u < deg g và deg v < deg f :
Bây giờ ta xét một số ví dụ để hiểu sâu sắc hơn lý thuyết. Các ví dụ
này chúng tôi tham khảo trong Lê Hoành Phò [6].
Bài toán 1.2.2. Cho P(x) = x + x
3
+ x
9
+ x
27
+ x
81
+ x
243
. Tìm dư của
phép chia đa thức P(x) cho
(a) x 1,
(b) x
2
1.
Lời giải. (a) Ta có P(x) = (x 1)Q(x) + r(x) với deg r(x) < deg(x 1) = 1. Suy
ra deg r(x) = 0 nên dư r(x) = c. Do đó P(x) = (x 1) Q(x) + c. Chọn x = 1
suy ra P(1) = c hay c = P(1) = 6: Vậy dư r(x) = 6:
(b) Ta có P(x) = (x
2
1) H(x) + s(x) với deg s(x) 1. Vậy
P(x) = (x
2
1) H(x) + ax + b:
Chọn x = 1 ta có P(1) = a + b = 6; x = 1 ta có P( 1) = a + b = 6. Do đó a =
6, b = 0. Vậy ta có đa thức dư là r(x) = 6x:
Dịch vụ viết thuê đề tài – KB Zalo/Tele 0917.193.864 – luanvantrust.com
Kham thảo miễn phí – Kết bạn Zalo/Tele mình 0917.193.864
13
Bài toán 1.2.3 (Trung Quốc 1981). Tìm dư của phép chia
(a) x
12
+ x
8
+ x
4
+ 1 cho x
3
+ x
2
+ x + 1;
(b) f (x
100
) cho f (x) với f (x) = x
99
+ x
98
+ : : : + x + 1.
Lời giải. (a) Ta có
x
12
+ x
8
+ x
4
+ 1 = (x
3
+ x
2
+ x + 1)(x
9
x
8
+ 2x
5
+
2x
4
+ 3x 3) + 4:
Suy ra dư là 4.
(b) Ta có
f (x) = x
99
+ x
98
+ : : : + x + 1
nên
f (x
100
) = x
9900
+ x
9800
+ : : : + x
100
+ 1
= f (x) x
9800
+ 2x
9701
2x
9700
+ 3x
9601
3x
9600
+
: : : + 99x 99) + 100
nên dư là 100.
Bài toán 1.2.4. Xác định a và b để đa thức f (x) = 6x
4
7x
3
+ ax
2
+ 3x + 2
chia hết cho x
2
x + b.
Lời giải. Lấy đa thức f (x) = 6x4
7x3
+ ax2
+ 3x + 2 chia cho g(x) = x2
x +
b thì được thương q(x) = 6x2
x + (a 5b 1) và phần dư r(x) = (a 5b + 2)x +
( ab 6b
2
+ a + b + 2).
..
Vì f (x) . g(x) nên r(x) = 0 suy ra
a 5b + 2 = 0; (1.1)
ab + 6b
2
+ b + 2 = 0: (1.2)
Từ (1.1) suy ra a = 5b 2. Thay vào (1.2) ta có b
2
+ 3b + 2 = 0 nên b = 1
hoặc
b = 2: Với b = 1 thì a = 7. Với b = 2 thì a = 12:
Dịch vụ viết thuê đề tài – KB Zalo/Tele 0917.193.864 – luanvantrust.com
Kham thảo miễn phí – Kết bạn Zalo/Tele mình 0917.193.864
Vậy f (x) = 6x
4
7x
3
7x
2
+ 2 hay f (x) = 6x
4
7x
3
12x
2
+ 3x + 2.
Dịch vụ viết thuê đề tài – KB Zalo/Tele 0917.193.864 – luanvantrust.com
Kham thảo miễn phí – Kết bạn Zalo/Tele mình 0917.193.864
14
Bài toán 1.2.5. Tìm ước chung lớn nhất của hai đa thức f (x) = x
2
+ x
3
3x
2
4x 1 và g(x) = x
3
+ x
2
x 1:
Lời giải. Ta thực hiện các phép chia liên tiếp và hỗ trợ với phép nhân
thêm hằng số
f (x) = q(x) g(x) + r(x)
thì
q(x) = x; r(x) = 2x
2
3x 1 và
g(x) = q1(x) r(x) + r1(x) thì
Dịch vụ viết thuê đề tài – KB Zalo/Tele 0917.193.864 – luanvantrust.com
Kham thảo miễn phí – Kết bạn Zalo/Tele mình 0917.193.864
q
1 (x) = 1 x 1 ;
r
1 (x) = 3x 3 ;
2 2
2 2
và tiếp tục r(x) = q2(x) r1(x) + r2(x) thì
q
2(x) = 2(2x + 1); r2(x) = 0
3
Do đó ( f (x); g(x)) = x + 1 với quy ước lấy hệ số cao nhất bằng 1 từ r1(x) =
3
x
3
:
2 2
Bài toán 1.2.6 (New York 1973, Bỉ 1981). Chứng minh rằng với mọi giá
trị n 2 N, đa thức (x + 1)2n+1
+ xn+2
chia hết cho đa thức x2
+ x + 1:
Lời giải. Ta chứng minh bằng quy nạp theo n 2 N.
Với n = 0 khẳng định đúng vì khi đó
(x + 1)
2n+1
+ x
n+2
= x
2
+ x:
Giả sử khẳng định đúng với n 1, nghĩa là (x + 1)
2n 1
+ x
n+1
chia hết cho
x
2
+ x + 1: Khi đó đa thức
(x + 1)2n+1
+ xn+2
= (x + 1)2
(x + 1)2n 1
+ x xn+1
= (x2
+ 2x + 1)(x + 1)2n 1
+ x xn+1
= (x
2
+ x + 1)(x + 1)
2n 1
+ x[x + 1)
2n 1
+ x
n+1
]
chia hết cho x2
+ x + 1. Suy ra điều chứng minh với n.
Dịch vụ viết thuê đề tài – KB Zalo/Tele 0917.193.864 – luanvantrust.com
Kham thảo miễn phí – Kết bạn Zalo/Tele mình 0917.193.864
15
n . k
Bài toán 1.2.7. Cho hai số nguyên dương n và k. Chứng minh rằng x 1
.
1
. x
khi và chỉ khi n là bội số của k.
n . k
Lời giải. Ta có thể phát biểu bài toán dưới dạng: Để x
.
1 điều kiện cần
1 . x
và đủ là n là bội số của k.
Điều kiện đủ. Giả sử n là bội số của k tức là n = km với m nguyên dương. Khi đó
x
n
1 = x
km
1
= (x
k
)
m
1
= (x
k
1)[x
(k 1)
+ x
k(m 1)
+ x
k(m 2)
+ : : : + x
k
+ 1]:
n . k
Đẳng thức này chứng tỏ rằng x 1
.
1:
. x
Điều kiện cần. Ta lấy số nguyên dương n chia cho số nguyên dương k.
Giả sử q và r là thương và số dư trong phép chia, tức là có n = kq + r với
0 r < k. Khi đó
x
n
1 = xkq+r 1 = xkq+r x
r
+ x
r
1 = x
r
(x
kq
1) + x
r
1: (1.3)
kq . k n . k
Ở trên ta đã chứng minh x 1
.
1:Vì vậy nếu x 1
.
1 thì từ (1.3) suy ra
. x . x
r . k r . k n . k
x
.
1: Nhưng r < k nên x
.
1 khi r = 0. Thành ra nếu x
.
1
1 . x 1 . x 1 . x
thì r = 0, tức là n = kq, hay n là bội số của k.
Bài toán 1.2.8 (Rumani 1962). Tìm điều kiện của các số nguyên p và q
sao cho đa thức
(a) P(x) = x
2
+ px + q nhận cùng giá trị chẵn (lẻ) với mọi x 2 Z.
(b) Q(x) = x
3
+ px + q nhận các giá trị chia hết cho 3 với mọi x 2 Z.
Lời giải. (a) P(x) nhận giá trị cùng chẵn (hoặc lẻ) với mọi x 2 Z khi và chỉ
khi mỗi số P(x + 1) P(x) = 2x + 1 + p chia hết cho 2 nghĩa là p lẻ.
Dịch vụ viết thuê đề tài – KB Zalo/Tele 0917.193.864 – luanvantrust.com
Kham thảo miễn phí – Kết bạn Zalo/Tele mình 0917.193.864
16
Khi đó tính chẵn lẻ của P(x) phụ thuộc vào tính chẵn lẻ của q = P(0). Như
vậy tất cả giá trị của P(x) là chẵn (lẻ) khi p lẻ và q chẵn (tương ứng q lẻ).
(b) Vì Q(x) = x(x
2
+ p) + q nên Q(3x) = 3x(9x
2
+ p) + q chia hết cho 3.
Với giá trị đó thì
Q(3x 1) = (3x 1)(9x
2
6x + 1 + p) + q (1 + p) (mod 3)
chia hết cho 3 khi và chỉ khi 1 + p chia hết cho 3.
Vậy Q(x) chia hết cho 3 (với mọi x 2 Z) khi q = 0, q 2 (mod 3).
Bài toán 1.2.9 (Hong Kong 2008). Xét đa thức
f (x) = cmx
m
+ cm 1x
m 1
+ : : : + c1x + c0
với các hệ số ci là những số nguyên khác không. Xây dựng dãy số (an)
như sau: a1 = 0 và an+1 = f (an) với n = 1; 2; : : : Giả sử i, j là những số
nguyên dương và i < j. Chứng minh rằng (a j+1 a j) là bội của (ai+1 ai).
Chứng minh. Do f (x) là đa thức hệ số nguyên nên [ f (a) f (b)] chia hết
cho (a b), với a và b là hai số nguyên phân biệt.
Do đó ai+2 ai+1 = f (ai+1) f (ai) chia hết cho (ai+1
Như vậy nếu i, j là những số nguyên dương và i < j thì
(a j+1 (ai+1 ai). Ta có điều phải chứng minh.
ai), với mọi i > 0.
aj) chia hết cho
1.3 Nghiệm của đa thức. Phương trình bậc cao
1.3.1 Nghiệm của đa thức
Bài toán 1.3.1. Cho đa thức bậc chẵn và tất cả hệ số đều lẻ. Chứng
minh đa thức không có nghiệm hữu tỷ.
Chứng minh. Xét P(x) = a0x
n
+ a1x
n 1
+ : : : + an 1x + an với a0 6= 0.
Dịch vụ viết thuê đề tài – KB Zalo/Tele 0917.193.864 – luanvantrust.com
Kham thảo miễn phí – Kết bạn Zalo/Tele mình 0917.193.864
17
Với n chẵn, các hệ số ai lẻ. Giả sử đa thức có nghiệm hữu tỷ x = q
p
thì p j a0, q j an. Suy ra p, q lẻ.
Thế x = q
p
vào đa thức ta có
an p
n
+ an 1qp
n 1
+ : : : + a0q
n
= 0:
Điều này vô lý vì vế trái là tổng của một số lẻ các số hạng lẻ nên không thể
bằng 0. Vậy đa thức không có nghiệm hữu tỷ. Ta có điều phải chứng minh.
Bài toán 1.3.2. Cho số tự nhiên n 2, chứng minh phương trình
xn
+
xn 1
+:::+
x2
+
x
+1=0: (1.4)
n! (n 1)! 2! 1!
không có nghiệm hữu tỷ.
Chứng minh. Ta chứng minh bài toán bằng phương pháp phản chứng.
Giả sử phương trình đã cho có nghiệm hữu tỷ a. Khi đó a sẽ là nghiệm
hữu tỷ của đa thức
P(x) = x
n
+ nx
n 1
+ : : : + n!
xk
: : : + n!
x2
+ n!
x
+ n!:
k! 2! 1!
Nhưng do P(x) là một đa thức bậc n với hệ số nguyên, và hệ số bậc cao
nhất, của x
n
, là 1 nên a phải là số nguyên và ta có
an
+ nan 1 ak a2
+ : : : + n! + : : : + n! + n! = 0: (1.5)
1!
k!
Gọi p là một ước nguyên tố của n. Với mọi k = 1; 2; : : : ; n, kí hiệu rk
là số mũ cao nhất của p thỏa mãn k!
..
. prk, ta có
rk = p + p2 + : : : + ps : (1.6)
k k k
Với s là số nguyên không âm thỏa mãn p
s
k < p
s+1
, từ (1.6) ta suy ra
k k k p2
rk + +:::+ = k < k:
p p2 ps p 1
Do đó rn rk > rn k. Suy ra rn rk rn k + 1: Vì vậy ta được
n! : pn k+1 với mọi k = 1; 2; : : : ; n: (1.7)
k!
Dịch vụ viết thuê đề tài – KB Zalo/Tele 0917.193.864 – luanvantrust.com
Kham thảo miễn phí – Kết bạn Zalo/Tele mình 0917.193.864
18
Nhưng mà n
..
. p nên từ (1.5) ta có an ..
. p và do đó a
..
. p. Suy ra ak ..
. pk
với mọi
k = 1; 2; : : : ; n.
Kết hợp điều này với (1.7) ta được n!
a
k!
k ..
. p
r
n
+1
với mọi k = 1; 2; : :
: ; n. Từ đây và (1.5) ta suy ra n!
..
. p
r
n
+1
:
Mâu thuẫn vừa nhận được chứng tỏ giả sử ban đầu là sai và vì vậy
ta có điều phải chứng minh.
Bài toán 1.3.3 (Việt Nam 1992). Cho đa thức
P(x) = 1 + x
2
+ x
9
+ x
n
1 + : : : + x
ns
+ x
1992
:
Với n1; : : : ; ns là các số tự nhiên cho trước thỏa mãn
9 < n1 < : : : < ns < 1992:
p
Chứng minh rằng nghiệm của đa thức P(x) (nếu có) không thể lớn hơn
1
2
5
.
Lời giải. Ta có
P(x) = 1 + x
2
+ x
9
+ x
n
1 + : : : + x
ns
+ x
1992
:
Với x 0 thì P(x) 1 > 0: Ta sẽ chứng minh
p
!:
1
; 0
P(x) > 0 với mọi x 2
5
2
Thật vậy với x < 0 và x 6= 1 ta có
P(x) 1 + x + x
3
+ x
5
+ : : : + x
2k+1
+ : : : + x
1991
= 1 + x
(x
1990
+ x
1998
+ : : : + 1):(1 x
2
)
(1 x2
)
1 x996
= 1 + x
x2
1
=
1 x
2
+ x x
997
:
1 x2
1 p
; 0 thì
Mà với x 2
5
2
1 x
2
> 0; x
997
> 0; 1 x
2
+ x > 0:
Dịch vụ viết thuê đề tài – KB Zalo/Tele 0917.193.864 – luanvantrust.com
Kham thảo miễn phí – Kết bạn Zalo/Tele mình 0917.193.864
19
nên
1 p
!:
P(x) > 0 với mọi x 2
5
; 0
2
1 p
; +¥. Ta có điều phải chứng minh.
Vậy P(x) > 0 với x 2
5
2
Bài toán 1.3.4 (IMO 1976). Cho các đa thức Pk(x), với k = 1; 2; 3 : : : xác định bởi
P1(x) = x
2
2; Pi+1 = P1(Pi(x)); với i = 1; 2; 3; : : :
Chứng minh rằng Pn(x) = x có 2
n
nghiệm thực phân biệt.
Lời giải. Ta thu hẹp việc xét nghiệm của phương trình trên đoạn 2 x 2:
Đặt x = 2 cos(t). Khi đó, bằng quy nạp ta chứng minh được
Pn(x) = 2 cos(2
nt
):
Thêm nữa, phương trình Pn(x) = x trở thành cos(2
nt
) = cos(t). Từ đó ta
được 2
n
nghiệm
t =
2kp
; t =
2kp
; k = 1; 2; : : : ; n
2n
1 2n
+ 1
Vậy phương trình Pn(x) = x có 2
n
nghiệm thực phân biệt.
Bài toán 1.3.5. Cho đa thức f (x) = a0 +a1x +: : : +anx
n
có n nghiệm
thực. Chứng minh với mọi p > n 1 thì đa thức
g(x) = a0 + a1:p:x + a2:p(p 1)x
2
+ : : : + an p(p 1): : : (p n +
1)x
n
:
cũng có n nghiệm thực.
Lời giải. Để giải bài toán ta xét hai trường hợp
Trường hợp 1. Đa thức f (x) không nhận x = 0 làm nghiệm.
Ta chứng minh bằng quy nạp.
Dịch vụ viết thuê đề tài – KB Zalo/Tele 0917.193.864 – luanvantrust.com
Kham thảo miễn phí – Kết bạn Zalo/Tele mình 0917.193.864
Với n = 1 bài toán hiển nhiên đúng .
Dịch vụ viết thuê đề tài – KB Zalo/Tele 0917.193.864 – luanvantrust.com
Kham thảo miễn phí – Kết bạn Zalo/Tele mình 0917.193.864
20
Giả sử đúng với n = k, ta chứng minh đúng với n = k + 1, tức là nếu đa thức
f (x) = a0 + a1x + : : : + ak+1x
k+1
có k + 1 nghiệm thực khác 0 thì đa thức
g(x) = a0 + pa1x + : : : + p(p1): : : (p k)ak+1x
k+1
cũng có k + 1 nghiệm thực khác 0 với mọi p > k.
Gọi c là một nghiệm của f (x) thì
f (x) = (x c)q(x); (1.8)
với q(x) là một đa thức nào đó là đa thức bậc k của x,
q(x) = b0 + b1x + : : : + bkxk
: (1.9)
Thay (1.9) vào (1.8) rồi đồng nhất hệ số ta được
a0 = cb0; a1 = cb1 + b0; : : : ak = cbk + bk 1;
a
k+1
= b
k
Do đó
g(x) = a0 + pa1x + : : : + p(p1): : : (p k)ak+1x
k+1
= cb0 + p(cb1 + b0)x + : : : + p(p 1)(p k)bkxk+1
= cQ(x) + pxQ(x) x
2
Q(x)
trong đó
Q(x) = b0 + b1 px + : : : + p(p 1): : : (p k + 1)bkx
k
:
Do f (x) có k + 1 nghiệm thực khác 0 nên q(x) có k nghiệm thực khác 0.
Mặt khác p > k nên p > k 1.
Cho nên theo giả thiết quy nạp ta có đa thức Q(x) có k nghiệm thực. Do
đó g(x) có k + 1 nghiệm thực. Vậy theo nguyên lý quy nạp, bài toán đúng.
Trường hợp 2. Đa thức f (x) nhận x = 0 làm nghiệm.
Dịch vụ viết thuê đề tài – KB Zalo/Tele 0917.193.864 – luanvantrust.com
Kham thảo miễn phí – Kết bạn Zalo/Tele mình 0917.193.864
21
Giả sử x = 0 là nghiệm bội k của f (x), với k 2 Z+, k n. Khi đó ta có
f (x) = akx
k
+ : : : + anx
n
= (anx
n k
+ : : : + ak)x
k
và
g(x) = p(p 1): : : (p k + 1)ak:x
k
+ : : : + p(p1): : : (p n + 1)an:x
n
= p(p 1): : : (p k + 1)x
k
[ak + : : : + (p k): : : (p n + 1)anx
n k
]:
Vì f (x) có n nghiệm thực nên H(x) = ak + : : : + anx
n k
có (n k) nghiệm thực
khác 0.
Do đó áp dụng kết quả của Trường hợp 1 cho H(x) và p
0
= p k > n k 1
(do p > n 1), ta được đa thức
R(x) = ak + : : : + (p k): : : (p n + 1)anxn k
có n k nghiệm thực.
Vậy g(x) có n nghiệm thực.
Bài toán 1.3.6 (Trung Quốc 1996). Cho đa thức p(x) bậc 5 có 5 nghiệm
thực phân biệt. Tìm số bé nhất của các hệ số khác 0.
Lời giải. Xét
p(x) = ax
5
+ bx
4
+ cx
3
+ dx + e; a 6= 0:
Nếu có 4 hệ số bằng 0 thì b = c = d = e = 0 nên p(x) = ax
5
có nghiệm bội (loại) tức
là p(x) không thể có một hệ số khác 0. Do đó p(x) có ít nhất hai hệ số khác 0.
Xét p(x) = ax
5
+bx
n
, n 2 thì p(x) có nghiệm bội. Ta tiếp tục loại trường
hợp này.
Xét p(x) = ax
5
+ dx = ax ax
4
+
a
có tối đa ba nghiệm. Ta cũng loại.
d
Do đó p(x) có ít nhất ba hệ số khác 0. Chọn p(x) = x
5
5x
3
+ 4x = x(x
2
4) thì p(x) có đúng 5 nghiệm phân biệt và đúng ba hệ số khác 0.
Vậy số bé nhất của hệ số khác 0 là 3.
Dịch vụ viết thuê đề tài – KB Zalo/Tele 0917.193.864 – luanvantrust.com
Kham thảo miễn phí – Kết bạn Zalo/Tele mình 0917.193.864
22
1.3.2 Phương trình bậc cao
Lý thuyết giải phương trình bậc 3 tổng quát
Xét phương trình đa thức bậc ba
ax
3
+ bx
2
+ cx + d = 0; a 6= 0: (1.10)
Ngoài việc tách nhóm số hạng hoặc tìm một nghiệm rồi phân tích nhân
tử, dùng hằng đẳng thức, ta có cách giải tổng quát như sau:
Trước hết, chia 2 vế cho a 6= 0 đưa về phương trình: x
3
+ Bx
2
+Cx + D = 0.
Tiếp theo đặt x = y
B
đưa tiếp về phương trình: y
3
py = q, trong đó
3
b2 2B
3
BC
p = C; q = + D:
3 27 3
Có hai hướng để giải phương trình
y3 py = q: (1.11)
Hướng thứ nhất. Đặt y = u + v và chọn u; v = 3
p
thì từ y3
= u3
+ v3
+ 3uv(u
+ v) ta có u3
+ v3
= q và u3
:v3
= P
27
3
. Vậy u3
; v3
là nghiệm phương trình
Z
3
qZ +
p3
= 0:
27
Nếu D < 0 sau này ta dùng số phức để giải quyết.
Hướng thứ hai. Nếu p = 0 thì (1.11) tương đương với y
3
= q, tức là y =
p
q.
q
Nếu p > 0. Đặt y = 2 p
t thì (1.11) tương đương với
3
4t
3
3t = m (1.12)
3p q
trong đó m =
3
2p
p
.
p
Xét jmj 1, đặt m = cos a thì (1.12) có ba nghiệm
t1 = cos a ; t2 = cos a + 2p; t3 = cos a 2p :
3
3 3
Dịch vụ viết thuê đề tài – KB Zalo/Tele 0917.193.864 – luanvantrust.com
Kham thảo miễn phí – Kết bạn Zalo/Tele mình 0917.193.864
23
Xét jmj > 1, đặt m = 2
1
d
3
+ d
1
3 suy ra d
3
= m
p
m
2
1. Phương trình
(1.12) có một nghiệm
t = 2 d + d = 2 q q m p m
2
m +
pm
2 1 +
1 1 1 3 3
Nếu p < 0. Đặt y = 2.
q
p
t thì (1.11) tương đương với
3
4t
3
+ 3t = m:
Ta đặt tiếp
1 1
m = k3 suy ra k3
= m
p
m2
+ 1:
2 k3
1
(1.13)
Phương trình (1.13) có một nghiệm
t = 2 k k = 2 q q :
m + pm2 + 1+ m pm2 + 1
1 1 1 3 3
Ta gọi các phương trình bậc ba 4x
3
+ 3x + m = 0; 4x
3
3x m = 0 là các
dạng phương trình bậc 3 chuẩn tắc. Ý nghĩa cơ bản là mọi phương trình
bậc 3 đều đưa về được dạng chuẩn tắc đó.
Chú ý thêm khi jmj 1,
4x3
+ 3x m = (x a)(4x2
+ 4ax + 4a2
+ 3):
Với q q
a = 2 m +
pm
2 + 1+ m pm
2 + 1
1 3 3
có D
0
= 12(a
2
+ 1) < 0 và
4x
3
3x m = (x b )(4x
2
+ 4b x + 4b
2
+ 3):
Với q q
b = 2 m +
p
m2 1 + m pm
2 1
1 3 3
có D
0
= 12(1 b
2
) < 0.
Dịch vụ viết thuê đề tài – KB Zalo/Tele 0917.193.864 – luanvantrust.com
Kham thảo miễn phí – Kết bạn Zalo/Tele mình 0917.193.864
24
Các phương trình bậc 4 đặc biệt
1. Phương trình có dạng
ax
4
+ bx
2
+ c = 0; a 6= 0: (1.14)
Đặt t = x2
; t 0 thì đưa về phương trình bậc hai at
2
+ bt + c = 0
2. Phương trình có dạng
(x + a)
4
+ (x + b)
4
= c: (1.15)
Đặt t = x +
a+
2
b
thì đưa về phương trình trùng phương At
4
+ Bt
2
+C = 0 và giải như trên.
3. Phương trình có dạng
(ax
2
+ bx + c)(ax
2
+ bx + d) = m: (1.16)
Đặt t = ax
2
+ bx thì đưa về phương trình bậc hai (t + c)(t + d) = m.
4. Phương trình có dạng
(x + a)(x + b)(x + c)(x + d) = m: (1.17)
Nếu có a + d = b + c thì ghép cặp (x + a)(x + d) và (x + b)(x + c) rồi đặt
t = x
2
+ (a + d)x = x
2
+ (b + c)x để đưa về dạng trên.
5. Phương trình có dạng
ax
4
+ bx
3
+ cx
2
+ dx + e = 0: (1.18)
Nếu ad2
= eb2
6= 0 thì chia hai vế cho x2
6= 0 rồi đặt t = x + ax
e
(Phương trình quy hồi mở rộng bậc bốn).
Dịch vụ viết thuê đề tài – KB Zalo/Tele 0917.193.864 – luanvantrust.com
Kham thảo miễn phí – Kết bạn Zalo/Tele mình 0917.193.864
25
Phương trình quy hồi (đối xứng hệ số)
Xét phương trình
a0x
n
+ a1x
n 1
+ a2x
n 2
+ + an 2x
2
+ an 1x + an = 0 (1.19)
trong đó a0 = an; a1 = an 1; a2 = an 2; : : :
Xét n chẵn, n = 2m. Chia hai vế cho x
m
6= 0:. Đặt t = x +
1
x ; jtj 2
đưa về phương trình bậc m =
n
2 .
Xét n lẻ, n = 2m + 1. Phương trình có nghiệm x = 1 nên phân tích ra
thừa số
(x + 1) và thừa số bậc 2m lại là phương trình quy hồi bậc chẵn như trên.
Đôi khi ta mở rộng dạng quy hồi (quy hồi kèm tỉ lệ) với cách đặt
t = x 1 ; t = x + a
x
x
Phương trình bậc cao
Xét phương trình
a0x
n
+ a1x
n 1
+ + an 1x + an = 0; a0 6= 0: (1.20)
Nguyên tắc chung là biến đổi về dạng tích, đặt ẩn phụ để đưa về
phương trình bậc thấp hơn. Đặc biệt
Nếu tổng các hệ số bằng 0 thì có nghiệm x = 1.
Nếu tổng đan dấu các hệ số bằng 0 thì có nghiệm x = 1 .
Nhắc lại, nghiệm hữu tỉ nếu có thì có dạng x = q
p
với p j an và q j
a0. Thế trực tiếp hoặc dúng sơ đồ Hoocne để thử nghiệm.
Đôi khi phương trình bậc cao đối với biến x mà lại bậc thấp đối với
tham số thì ta chuyển về phương trình theo ẩn là tham số đó.
Dịch vụ viết thuê đề tài – KB Zalo/Tele 0917.193.864 – luanvantrust.com
Kham thảo miễn phí – Kết bạn Zalo/Tele mình 0917.193.864
26
Bài toán 1.3.7. Giải các phương trình sau
(a) 4x3
10x2
+ 6x 1 = 0.
(b) 8x3
36x + 27 = 0.
Lời giải. (a) Ta có
4x
3
10x
2
+ 6x 1 = 0 , 4x
3
2x
2
8x
2
+ 4x + 2x 1 = 0
, 2x2
(2x 1) 4x(2x 1) + (2x 1) = 0
, (2x 1)(2x
2
4x + 1) = 0
, 2x 1 = 0 hoặc 2x2
4x + 1 = 0:
Vậy nghiệm của phương trình là
p
x =1 ; x = 2 2 :
2 2
Bài toán 1.3.8. Tìm quan hệ giữa p và q để phương trình x
3
+ px + q = 0
có thể viết dưới dạng x4
= (x2
ax + b)2
. Áp dụng kết quả đó để giải
phương trình: x
3
18x + 27 = 0.
Lời giải. Ta có
x
4
(x
2
ax + b)
2
= m(x
3
+ px + q):
Suy ra
a
2
+ 2b = 0; 2ab = pm; 2a = m; b
2
= qm:
Từ đó b = p, kéo theo p
2
= mq. Vậy m =
p
2
.
q
Mặt khác a
2
= 2b ) a =
m
2 )
m
4
2
= 8 ) p
4
+ 8pq
2
= 0.
Vậy quan hệ giữa p và q: p
3
+ 8q
2
= 0.
Ta có
4
2b = 2p ) m
2
= 8p. Từ đó
p
=
q
2
x3
18x + 27 = 0 , x4
= (x2
+ 6x 18)
2
Dịch vụ viết thuê đề tài – KB Zalo/Tele 0917.193.864 – luanvantrust.com
Kham thảo miễn phí – Kết bạn Zalo/Tele mình 0917.193.864
27
, (6x 18)(2x
2
+ 6x 18) = 0
p
,
x = 3 hoặc x = 3 35 :
2
Bài toán 1.3.9. Giải và biện luận phương trình
x
3
3x
2
+ 3(a + 1)x (a + 1)
2
= 0: (1.21)
Lời giải. Ta có x
3
3x
2
+ 3(a + 1)x (a + 1)
2
= 0 tương đương với
x
3
= 3x
2
+ 3(a + 1)x (a + 1)
2
:
Nếu a 6= 1 thì nhân hai vế với (a + 1), ta được
x
3
(a + 1) = 3x
2
(a + 1) + 3(a + 1)
2
x (a + 1)
3
Cộng hai vế với x
3
ta được ax
3
= (x a 1)
3
. Từ đó ta được x a 1 = p3 ,
ax
suy ra
p
a + 1 p3 2
x = a
3
a:
p
3 + 1 =
a
Nếu a = 1 thì dễ thấy phương trình có hai nghiệm x1 = 0 và x2 = 3.
Bài toán 1.3.10. Giải các phương trình sau
(a) x4
2x3
6x2
+ 16x 8 = 0;
(b) x
4
+ x
2
+ 4x 3 = 0;
Lời giải. (a) Ta có
x
4
2x
3
6x
2
+ 16x 8 = 0 , (x 2)(x
3
6x + 4) = 0
, (x 2)(x 2)(x
2
+ 2x 2) = 0
, (x 2)
2
(x
2
+ 2x 2) = 0:
Vậy nghiệm của phương trình là x = 2, x =
p
.
1 3
Dịch vụ viết thuê đề tài – KB Zalo/Tele 0917.193.864 – luanvantrust.com
Kham thảo miễn phí – Kết bạn Zalo/Tele mình 0917.193.864
28
(b) Ta có
x
4
+ x
2
+ 4x 3 = 0 , x
4
= x
2
4x + 3 , (x
+ 1)
2
= (x 2)
2
, (x2
+ 1)2
(x 2)
2
= 0
, (x2
x + 3)(x2
+ x 1) = 0
p
Vậy nghiệm của phương trình là x = 1 5 .
2
Bài toán 1.3.11 (IMO 1973). Giả sử phương trình x
4
+ ax
3
+ bx
2
+ ax +
1 = 0 có nghiệm. Tìm giá trị bé nhất của a
2
+ b
2
.
Lời giải. Gọi x0 là nghiệm của phương trình đã cho, tức là
x0
4
+ ax0
3
+ bx0
2
+ ax0 + 1 = 0
Từ đây ta có kết luận x0 6= 0.
Chia hai vế cho x0
2
, ta có
x0
2
+ ax0 + b + a
+
1
= 0:
x0 x0
2
Phương trình này tương đương với
x02
x
0
x0
2
+
1
+ a x0 +
1
+ b = 0:
1 1 2
Đặt y = x0 + với điều kiện jyj = jx0j+ j j 2. Ta có (y 2) + ay + b = 0 Suy
x0 x0
ra
(2 y
2
)
2
j2 y
2
j = jay + bj
p
a2 + b2
p
y2 + 1 ) a
2
+ b
2
1 + y
2
Đặt t = y
2
, t 4. Ta chứng minh
(2 t)
2
4
(1.22)
1 +t 5
Thật vậy, ta có (1.22) tương đương với 5(2 t)
2
4(1 +t), tức là 5t
2
24t + 16
0. Nhưng điều này đúng vì t 4. Như vậy giá trị bé nhất của a
2
+ b
2
là 4=5.
Dịch vụ viết thuê đề tài – KB Zalo/Tele 0917.193.864 – luanvantrust.com
Kham thảo miễn phí – Kết bạn Zalo/Tele mình 0917.193.864
29
Bài toán 1.3.12 (Việt Nam 2002). Giải phương trình
q p
4 3 10 3x = x 2 (1.23)
Lời giải. Ta có biến đổi tương đương như sau
p
8
x 2
q
>
4 3 = x 2 ,
<
10 3x >4 3
p
= (x 2)
2
10 3x
:
8
x 2
>
,
<
p
x x
2
>3 10 3x = 42
:
8
x 2; 4x x 0
, > 2 2
<
9(10 3 x) = (4x x )
>
:
8
x 2; 0 x 4
, > 4 3 2 + 27x 90=0:
<x 8x + 16x
>
:
8
2 x 4
, > 4 3 2 + 27x 90=0:
<x 8x + 16x
>
Bây giờ ta sẽ giải phương trình :
x
4
8x
3
+ 16x
2
+ 27x 90=0: (1.24)
Bằng cách thử trực tiếp ta thấy x = 3 là một nghiệm nên phương trình
(1.24) được viết lại thành
(x 3)(x + 2)(x
2
7x + 15) = 0:
Vậy phương trình có nghiệm duy nhất x = 3.
Bài toán 1.3.13 (Việt Nam 1991). Giải phương trình
x
3
3x
2
8x + 40 = 8 p4 : (1.25)
4x + 4
Dịch vụ viết thuê đề tài – KB Zalo/Tele 0917.193.864 – luanvantrust.com
Kham thảo miễn phí – Kết bạn Zalo/Tele mình 0917.193.864
30
p
Lời giải. Từ phương trình x
3
3x
2
8x + 40 = 8 4
4x + 4 ta có điều kiện x 1.
Áp dụng bất đẳng thức AM-GM ta có
8
p
4 = 4
p
4 x + 13:
4x + 4 4:4:4:(x + 1)
Do đó
x
3
3x
2
8x + 40 x + 13 , x
3
3x
2
9x + 27 0
, (x + 3)(x
2
6x + 9) 0
, (x + 3)(x 3)
2
0:
Vì x1 nên (x 3)
2
0, suy ra x = 3. Thử lại ta thấy đúng. Vậy phương trình
có nghiệm duy nhất x = 3.
Bài toán 1.3.14. Chứng minh rằng
r r
q q
(a) x =
3
a +
a+1 8a 1
+
3
a
a+1 8a 1
với a
1
là số tự nhiên
3 3 3 3 8
(b)
p
3 +
p
3 là số vô tỉ.
2 4
Lời giải. (a) Áp dụng hằng đẳng thức (u + v)3
= u3
+ v3
+ 3uv(u + v) ta có
x
3
= 2a + (1 2a)x , x
3
+ (2a 1)x 2a = 0
, (x 1)(x
2
+ x + 2a) = 0:
Xét đa thức bậc hai x
2
+ x + 2a có D = 1 8a 0
q q
Khi a =
1
8 , ta có x = 3 1
8 + 3 1
8 = 1
Khi a >
1
8 , ta có 1 8a âm nên đa thức x
2
+ x + 2a có nghiệm thực
duy nhất x = 1.
Vậy với a
1
thì
8
s s
x = a + a
3 1r
a a 3 1r
8a3 1 + 8a3 1
3 + 3 +
Dịch vụ viết thuê đề tài – KB Zalo/Tele 0917.193.864 – luanvantrust.com
Kham thảo miễn phí – Kết bạn Zalo/Tele mình 0917.193.864
31
là một số tự nhiên.
p p
(b) Đặt x = 3
2 + 3
4. Ta có
p p p
x
3
= 2 + 4 + 3 3
8( 3
2 + 3
4)
Tức là x
3
6x 6:
Giả sử x 2 Q. Do hệ số bậc cao nhất của đa thức là 1 nên x là số nguyên. Ta có
p p
2 < 3
2 + 3
4 < 4 nên x = 3. Do đó x
3
6x 6 = 3, vô lý. Vậy x là số vô tỷ.
Bài toán 1.3.15 (Việt Nam 1984). Tìm đa thức theo x có bậc bé nhất với hệ số
nguyên, biết một nghiệm là
p
+
p
3 .
2 3
Lời giải. Đặt a =
p
+
p
3 ta có
2 3
a
2
= 2 + 2
p
:
p
3 +
p
3
2 3 9
và
p p3 p :
p
3
a
3
= 2 + 3 + 3 (1.26)
2 + 6 3 2 9
Ta rút ra
p3 = a p ;
3 2
p
3 = a
2 2 2
p
(a p ) = a
2
+ 2 2
p
a:
9 2 2 2
Thay vào (1.26) ta có
a3 = 2
p
+ 6(a p ) + 3
p
(a2
+ 2 2
p
a) + 3;
2 2 2 2
a
3
+ 6a 3 =
p
(3a
2
+ 2):
2
Bình phương hai vế ta thấy a là nghiệm của đa thức
x
6
6x
4
6x
3
+ 12x
2
36x + 1
Bằng phép đồng nhất hệ số ta chứng minh đa thức trên không phân tích
được thành hai đa thức bậc thấp hơn có hệ nguyên nên đa thức trên
chính là đa thức có bậc bé nhất thỏa đề bài.
Dịch vụ viết thuê đề tài – KB Zalo/Tele 0917.193.864 – luanvantrust.com
Kham thảo miễn phí – Kết bạn Zalo/Tele mình 0917.193.864
32
1.4 Đạo hàm của đa thức. Định lý Taylor
Đạo hàm của một hàm số là một khải niệm thuộc nhánh Giải tích toán học. Trong
mục này chúng tôi sẽ trình bày ngắn gọn về đạo hàm của hàm số đa thức.
Định nghĩa 1.4.1. Cho f (x) 2 R[x] và deg f (x) = n,
anx
n
+ an 1x
n 1
+ : : : + a1x + a0 với an 6= 0
Đa thức
nanx
n 1
+ (n1)an 1x
n 2
+ : : : + 2a2x + a1
được gọi là đạo hàm cấp một của đa thức f (x) (hoặc ngắn gọn là đạo hàm của
đa thức f (x)), và được ký hiệu là f 0
(x). Bằng quy nạp, ta định nghĩa đạo hàm
cấp k của đa thức f (x) là đạo hàm của đạo hàm cấp k 1. Ta ký hiệu đạo hàm
cấp 2, 3 và cấp k tổng quát lần lượt là f 00
(x), f 000
(x), và f (k)
(x).
Với mọi x0 2 R, biểu thức sau đây được gọi là khai triển Taylor của
đa thức thực f (x)
f (x) = f (x0)+
f 0(x0)
(x x0)+: : :+
f (k)(x0)
(x x0)k +: : :+
f (n)(x0)
(x x0)n:
1! k! n!
Từ định nghĩa này ta có nếu deg f = n thì
deg f
0
= n 1; deg f
00
= n 2;:::;
deg f
(k)
= n k với 1 k n; deg f
(x)
Bây giờ ta xét một số bài toán về đạo hàm hàm
và khai tham khảo [6] trong trình bày.
= 0:
triển Taylor. Chúng tôi
Bài toán 1.4.2 (Ba Lan 1979). Cho đa thức P(x) có bậc n > 1 và có n
nghiệm thực x1; x2; x3; : : : ; xn phân biệt. Chứng minh rằng
1
+
1
+:::+
1
= 0:
P
0
(x1) P
0
(x2) P
0
(xn)
Dịch vụ viết thuê đề tài – KB Zalo/Tele 0917.193.864 – luanvantrust.com
Kham thảo miễn phí – Kết bạn Zalo/Tele mình 0917.193.864
33
Lời giải. Đặt
P(x) = a(x x1)(x x2): : : (x xn); a 6= 0
ta suy ra
n
P
0
(x) = P1(x) + P2(x) + : : : + Pn(x) với Pi(x) = Õ (x x j):
j=1; j6=i
Ta thấy
Pi(x ) = 0 với mọi j = i kéo theo P
0
(x
j
) = P (x ) = 0 với mọi j = 1; 2; : : : ; n
j 6 j j 6
Xét đa thức
n P (x)
F(x) = å i
1
i=1 P
0
(xi)
có bậc không vượt quá n 1. Với i = 1; 2; : : : ; n ta có F(xi) =
Pi(xi)
1 = 0 suy ra
P
0
(xi)
F(x) có n nghiệm phân biệt. Vậy F(x) = 0. Mặt khác hệ số của F(x) đối với x
n 1
bằng 0 nên
a
+
a
+:::+
a
= 0:
P
0
(x1) P
0
(x2) P
0
(xn)
Như vậy
1
+
1
+:::+
1
= 0:
P
0
(x1) P
0
(x2) P
0
(xn)
Bài toán 1.4.3 (CHDCLB Đức 1974).
(a) Chứng minh rằng không tồn tại đa thức P(x) để với mọi x 2 R ta có
các bất đẳng thức
P
0
(x) > P
00
(x); (1.27)
P(x) > P
00
(x): (1.28)
(b) Khẳng định trên còn đúng không nếu thay đổi bất đẳng thức (1.27) bằng
bất đẳng thức
P(x) > P
0
(x): (1.29)
Dịch vụ viết thuê đề tài – KB Zalo/Tele 0917.193.864 – luanvantrust.com
Kham thảo miễn phí – Kết bạn Zalo/Tele mình 0917.193.864
34
Lời giải. (a) Nếu P(x) là hằng số thì P
0
(x) = P
00
(x) = 0, và bất đẳng thức
(1.27) không thỏa mãn.
Giả sử deg P(x) = n 1 khi đó nếu n lẻ thì deg(P(x) P
00
(x)) = n là số lẻ,
từ đó P(x) P
00
(x) 0 với ít nhất một điểm x 2 R.
Như vậy đối với đa thức P(x) không thỏa mãn hoặc bất đẳng thức (1.28) hoặc
bất đẳng thức (1.27).
Vậy (a) được chứng minh xong.
(b) Chọn P(x) = x2 + 3. Khi đó với x 2 R ta có P(x) P0(x) x2 2x + 3 > 0 và
P(x) P00(x) x2 + 1 > 0 nghĩa là khẳng định trên không còn đúng nữa.
Bài toán 1.4.4 (Việt Nam 1986). Cho f 2 R[x] có deg f = n và f (k) = 2k, với
k = 0; 1; 2; : : : ; n. Tính f (n + 1).
Lời giải. Xét đa thức
g(x) = 1 + x + x(x 1) + : : : + x(x 1)(x 2): : : (x n + 1):
1! 2! n!
Khi đó deg g = n và
n
g(k) = åCk
i
= 2
k
= f (k)
i=0
với n + 1 giá trị nên f g.
Do đó
n
f (n + 1) = g(n + 1) = åCi
+ = 2n+1 1:
n 1
i=0
Bài toán 1.4.5 (Singapore 1978). Cho đa thức P(x) bậc n và hai số a < b
thỏa mãn
P(a) < 0; P
0
(a) 0; P(a) 0; : : : ; ( 1)
n
P
(n)
(a) 0;
P(b) > 0; P
0
(b) 0; P(b) 0; : : : ; P
(n)
(b) 0:
Chứng minh các nghiệm thực của P(x) thuộc (a; b).
Dịch vụ viết thuê đề tài – KB Zalo/Tele 0917.193.864 – luanvantrust.com
Kham thảo miễn phí – Kết bạn Zalo/Tele mình 0917.193.864
35
Lời giải. Khai triển Taylor ta có
P(x) = P(b) +
P0(b)
(x b) +
P(b)
(x b)2 + : : : +
P(n)(b)
(x b)n:
1! 2! n!
Nếu x b thì P(x) > 0 suy ra P(x) không có nghiệm x b.
Tương tự
P(x) = P(a) +P
0
(a) (x a) + P(a)(x a)2
+ : : : + P
(n)
(a)(x a)n
2!
1! n!
= P(a) + P
0
(a) (a x) + P(a)(a x)
2
+ : : : + ( 1)
n
(a) (a x)
n
1! 2! n!
Nếu x < a thì P(x) < 0 suy ra P(x) không có nghiệm x a
Vậy các nghiệm phải thuộc (a; b). Bài toán được chứng minh xong.
Ta gọi ước lượng về nghiệm ở trên là ước lượng Newton.
Dịch vụ viết thuê đề tài – KB Zalo/Tele 0917.193.864 – luanvantrust.com
Kham thảo miễn phí – Kết bạn Zalo/Tele mình 0917.193.864
36
Chương 2
Đa thức bất khả quy
2.1 Đa thức bất khả quy
Định nghĩa 2.1.1. Cho đa thức f 2 Z[x]. Ta gọi đa thức f là bất khả quy
trên Z[x] nếu f không phân tích được thành tích hai đa thức thuộc Z[x]
với bậc lớn hơn hay bằng 1.
Trường hợp f là một đa thức hữu tỷ, ta định nghĩa hoàn toàn tương tự.
Mối quan hệ về tính bất khả quy trên Z[x] và Q[x] được phát biểu như sau:
Định lí 2.1.2. Nếu đa thức f 2 Z[x] bất khả quy trên Z[x] thì f cũng bất khả
quy trên Q[x].
Để chứng minh định lý này, ta cần bổ đề sau đây.
Trước hết, ta gọi đa thức f 2 Z[x] là nguyên bản nếu các hệ số
nguyên tố cùng nhau.
Bổ đề 2.1.3 (Bổ đề Gauss). Tích của hai đa thức nguyên bản là một đa
thức nguyên bản.
Chứng minh. Cho hai đa thức nguyên bản
f (x) = a0x
n
+ a1x
n
1 + : : : + an;
g(x) = b0x
m
+ b1x
m 1
+ : : : + bm:
Dịch vụ viết thuê đề tài – KB Zalo/Tele 0917.193.864 – luanvantrust.com
Kham thảo miễn phí – Kết bạn Zalo/Tele mình 0917.193.864
37
Khi đó
f (x)g(x) = c0x
n+m
+ c1x
n+m 1
+ : : : + cn+m:
Giả sử đa thức tích f (x)g(x) không nguyên bản. Khi đó tồn tại một số nguyên tố
p là ước chung của các hệ số c0; : : : ; cn+m
Vì f nguyên bản nên gọi ai là số đầu tiên mà ai
..
. p.
Vì g nguyên bản nên gọi b j là số đầu tiên mà b j
..
. p.
Bằng cách xét hệ số theo lũy thừa x
i+ j
ta có hệ số tương ứng không
chia hết cho p. Điều này vô lý. Vậy f (x)g(x) là đa thức nguyên bản.
Chứng minh Định lý 2.1.2. Cho f 2 Z[x] là một đa thức bất khả quy. Giả
sử f khả quy trên Q[x], tức là ta có biểu diễn f (x) = f1(x) f2(x) với f1; f2 2
Q[x], có bậc lớn hơn hoặc bằng 1.
Đặt
f1(x) =
a1 g1(x); f2(x) =
a2
g2(x)
b1 b2
voi ai tối giản và g1(x), g2(x) nguyên bản. Khi đó
b
i
a
1
a
2 p
f (x) = f1 (x) f2 (x) = g1 (x)g2(x) = g1 (x)g2(x) với (p; q) = 1:
b1b2 q
Do đó f 2 Z[x] nên mọi hệ số của khai triển tích g1(x)g2(x) đều là bội số
của q. Suy ra đa thức tích g1(x)g2(x) không nguyên bản. Điều này trái
với kết quả của Bổ đề Gauss. Vậy f bất khả quy trên Q[x].
2.1.1 Đa thức với hệ số thực và phức
Cho một đa thức với hệ số thực thì chưa chắc đa thức đó có nghiệm trong
trường số thực, cụ thể đa thức x2
+ 1 không có nghiệm trong trường số
thực. Dưới đây ta sẽ thấy mọi đa thức bậc n với hệ số phức có đúng n
nghiệm phức. Để chứng minh ra hãy đưa vào các bổ đề sau đây:
Bổ đề 2.1.4. Mọi đa thức với hệ số thực có bậc lẻ có ít nhất một nghiệm thực.
Dịch vụ viết thuê đề tài – KB Zalo/Tele 0917.193.864 – luanvantrust.com
Kham thảo miễn phí – Kết bạn Zalo/Tele mình 0917.193.864
38
Chứng minh. Giả sử
f (x) = anxn
+ an 1xn 1
+ :::: + a0; an 6= 0 và n lẻ.
Qua giáo trình giải tích ta biết rằng với những giá trị dương và âm của x,
khá lớn về giá trị tuyệt đối, hàm số f (x) có các dấu trái nhau. Vậy có
những giá trị thực của x, a và b chẳng hạn, sao cho.
f (a) < 0; f (b) > 0:
Mặt khác hàm số f (x) là liên tục, vì vậy có một giá trị c của x, nằm giữa a
và b, sao cho f (x) = 0.
Bổ đề 2.1.5. Mọi đa thức bậc hai ax
2
+ bx + c, với hệ số phức, bao giờ cũng có
hai nghiệm phức.
Bổ đề 2.1.6. Mọi đa thức bậc lớn hơn 0 với hệ số thực có ít nhất một nghiệm phức.
Định lí 2.1.7. Mọi đa thức bậc lớn hơn 0 với hệ số phức có ít nhất một
nghiệm phức.
Chứng minh. Giả sử f (x) là một đa thức bậc n > 0
f (x) = a0 + a1x + :::: + anx
n
với hệ số phức. Đặt
f(x) = a0 + a1x + :::: + anxn
với các ai là các liên hợp của các ai với i = 0; : : : ; n. Xét đa thức
g(x) = f (x) f (x):
Ta có
g(x) = b0 + b1x + :::: + b2nx
2n
Với
bk = å aia j; k = 0; 1; ::::; 2n:
i+ j=k
Dịch vụ viết thuê đề tài – KB Zalo/Tele 0917.193.864 – luanvantrust.com
Kham thảo miễn phí – Kết bạn Zalo/Tele mình 0917.193.864
39
Vì
b
k
=
å a
i
a
j
= b
k
i+ j=k
nên các hệ số bk là thực. Theo Bổ đề 2.1.6 g(x) có ít nhất một nghiệm
phức z = s + it,
g(z) = f (z) f (z) = 0:
Do đó hoặc f (x) = 0 hoặc f (z) = 0. Nếu f (z) = 0,
f (z) = a0 + a1z + :::: + anzn
= 0:
thì
a0 + a1z + : : : anz
n
= a0 + a1z + : : : + anz
n
= 0;
tức là f (z) = 0. Như vậy hoặc z hoặc z là nghiệm của f (x).
Hệ quả 2.1.8. Các đa thức bất khả quy của vành C[x], C là trường số
phức, là các đa thức bậc nhất.
Chứng minh. Các đa thức bậc nhất là bất khả quy. Giả sử f (x) là một đa
thức của C[x] có bậc lớn hơn 1. Theo Định lí 2.1.7, f (x) có một nghiệm
phức c. Vậy f (x) có một ước thực sự x c, do đó f (x) không bất khả quy.
Hệ quả 2.1.9. Mọi đa thức bậc n > 0 với hệ số phức có n nghiệm phức.
Hệ quả 2.1.10. Các đa thức bất khả quy của R[x], R là trường số thực, là các
đa thức bậc nhất và các đa thức bậc hai ax
2
+ bx + c với biệt số b
2
4ac < 0.
Chứng minh. Các đa thức bậc nhất và các đa thức bậc hai với biệt số âm rõ ràng
là những đa thức bất khả quy của R[x]. Giả sử p(x) là một đa thức bất khả quy của
R[x] với bậc lớn hơn một. Vậy p(x) không có nghiệm thực. Theo Định lí 2.1.7, p(x)
có một nghiệm phức z và p(x) chia hết cho đa thức bậc hai với hệ số thực.
g(x) = x
2
(z + z)x + zz:
Dịch vụ viết thuê đề tài – KB Zalo/Tele 0917.193.864 – luanvantrust.com
Kham thảo miễn phí – Kết bạn Zalo/Tele mình 0917.193.864
40
Đa thức g(x) không khả nghịch và là ước của phần tử bất khả quy p(x),
vậy g(x) phải là liên kết của p(x), tức là
p(x) = ug(x); 0 6= u 2 R:
Ta có điều cần chứng minh.
2.1.2 Đa thức bất khả quy của vành Q[x]
Đối với trường số thực R và trường số phức C, vấn đề xét xem một đa thức đã
cho của vành R[x] hay C[x] có bất khả quy hay không rất đơn giản, nhưng
trong vành Q[x] với Q là trường số hữu tỉ thì vấn đề phức tạp hơn nhiều. Đối
với các đa thức bậc hai và ba của Q[x], việc xét xem có bất khả quy hay không
được đưa về việc tìm nghiệm hữu tỉ của đa thức đó. Các đa thức bậc hai và
bậc ba của Q[x] là bất khả quy khi và chỉ khi chúng không có nghiệm hữu tỉ.
Đối với các đa thức bậc lớn hơn ba thì vấn đề phức tạp hơn nhiều. Chẳng hạn
đa thức x
4
+ 2x
2
+ 1 = (x
2
+ 1)
2
rõ ràng không có nghiệm hữu tỉ nào, nhưng nó
có một ước thực sự x
2
+ 1 , vậy không phải là bất khả quy.
Ta đã biết, mọi đa thức f (x) với hệ số hữu tỉ đều có thể viết dưới dạng
f (x) = b
1
g(x)
trong đó b là một số nguyên khác 0, g(x) là một đa thức với hệ số nguyên. Trong vành
Q[x], f(x) và g(x) là liên kết vậy f (x) là bất khả quy khi và chỉ khi g(x) là bất khả quy. Do
đó tiêu chuẩn Eisenstein mà ta đưa ra dưới đây để xét một đa thức của Q[x] có bất
khả quy hay không là tiêu chuẩn cho các đa thức với hệ số nguyên.
Bổ đề 2.1.11. Nếu f (x) là một đa thức với hệ số nguyên có bậc lớn hơn
0 và f (x) không bất khả quy trong Q[x], thì f (x) phân tích được thành một
tích những đa thức bậc khác 0 với hệ số nguyên.
Trong nghiên cứu các đa thức bất khả quy, Tiêu chuẩn Eisenstein
sau đây là đặc biệt quan trọng.
Dịch vụ viết thuê đề tài – KB Zalo/Tele 0917.193.864 – luanvantrust.com
Kham thảo miễn phí – Kết bạn Zalo/Tele mình 0917.193.864
41
Định lí 2.1.12 (Tiêu chuẩn Eisenstein). Cho f (x) 2 Z[x], deg f
(x) = n, f (x) = a0x
n
+ a1x
n
1 + : : : + an:
Nếu có số nguyên tố p thỏa mãn ba điều kiện
(1) a0 không chia hết cho p;
(2) a1; a2; : : : ; an chia hết cho p;
(3) an không chia hết cho p
2
.
thì đa thức f (x) bất khả quy trên Q[x].
Chứng minh. Giả sử f (x) có những ước thực sự trong Q[x] theo Bổ đề
2.1.11, f (x) có thể viết
f(x) = g(x)h(x);
trong đó
g(x) = b0 + b1x + : : : + brxr;
h(x) = c0 + c1x + : : : +s xs;
bi 2 Z;
ci 2 Z;
0 < r < n;
0 < s < n:
Ta có
a0 = b0c0;
a1 = b1c0 + b0c1;
: : :
ak = bkc0 + bk 1c1 + : : : + b0ck;
an = brcs:
Theo giả thiết p chia hết a0 = b0c0; vậy vì p là nguyên tố nên hoặc p chia hết cho
b0 hoặc p chia hết cho c0. Giả sử p chia hết b0 thế thì p không chia hết c0, vì nếu
thế thì p
2
sẽ chia hết a0=b0c0, trái với giả thiết, p không thể chia hết mọi hệ số
Dịch vụ viết thuê đề tài – KB Zalo/Tele 0917.193.864 – luanvantrust.com
Kham thảo miễn phí – Kết bạn Zalo/Tele mình 0917.193.864
42
của g(x), vì nếu thế thì p sẽ chia hết an = brcs, trái với giả thiết. Vậy giả
sử bk là hệ số đầu tiên của g(x) không chia hết cho p. Ta hãy xét
ak = bkc0 + bk 1c1 + : : : + b0ck;
trong đó ak; bk 1; : : : ; b0 đều chia
hết cho nguyên tố, ta suy ra hoặc bk
chia hết cho giả thiết về bk và c0.
p. Vậy bkc0 phải chia hết cho p. Vì p là
p, hoặc c0 chia hết cho p, mâu thuẫn với
2.2 Một số bài toán điển hình
Bài toán 2.2.1 (IMO 1993). Cho n 2 N và n > 1. Chứng minh rằng đa thức
f (x) = xn
+ 5xn 1
+ 3
bất khả quy trên Z[x]:
Lời giải. Với n = 2 có f (x) = x
2
+ 5x + 3 thì bất khả quy trên Z[x].
Xét n 3. Giả sử f (x) = g(x)h(x) với g(x); h(x) 2 Z[x] và có bậc lơn hơn
hoặc bằng 1. Do deg g(x) +deg h(x) = n 3 nên suy ra trong hai số deg
g(x) và deg h(x) có một số lơn hơn 1.
Vì f (0) = 3 là số nguyên tố nên hoặc jg(0)j = 1 hoặc jh(0)j = 1. Giả sử
g(x) = x
k
+ b1x
k 1
+ : : : + bk với k > 1
và jg(0)j = 1.
Gọi a1; a2; : : : ; ak là các nghiệm (nói chung là nghiệm phức) của
g(x). Khi đó ta có
g(x) = (x a1)(x a2): : : (x ak):
Vì jg(0)j = 1 nên
ja1a2 : : : akj = 1: (2.1)
Dịch vụ viết thuê đề tài – KB Zalo/Tele 0917.193.864 – luanvantrust.com
Kham thảo miễn phí – Kết bạn Zalo/Tele mình 0917.193.864
43
Do g(ai) = 0 nên f (ai) = 0, với i 2 f1; : : : ; kg.
Nhân các đẳng thức đó lại và sử dụng (2.1) ta được
j(a1 + 5)(a2 + 5): : : (ak + 5)j = 3
k
(2.2)
Mặt khác ta có
g( 5) = j(a1 + 5)(a2 + 5): : : (ak + 5)j và 3 = f ( 5) = g( 5)h( 5)
nên j(a1 + 5)(a2 + 5): : : (ak + 5)j nhận giá trị 1 hoặc 3.
Điều này trái với (2.2) vì k > 1. Từ đó ta có ta có điều phải chứng minh.
Bài toán 2.2.2 (Dự tuyển IMO). Cho đa thức f (x) = Õ
n
= (x ai) 2
với n 3 và
i 1
a1; a2; : : : an là các số nguyên đôi một khác nhau. Chứng minh rằng
nếu f (x) khả quy trên Z[x] thì n = 3.
Lời giải. Giả sử f (x) = g(x)h(x) với g(x); h(x) 2 Z[x] và deg g = p q
= deg h 1.
Ngoài ra, giả sử các hệ số bậc cao nhất của g và h đều bằng 1.
Trong phép đặt cho x = ai ta sẽ nhận được f (ai) = g(ai)h(ai) = 2: Như
vậy ta có
(g(ai); h(ai)) 2 f( 2; 1); (2; 1); (1; 2); ( 1; 2)g
Vậy với mọi i, ta có g(ai) + h(ai) = +1 với ít nhất
n
2 giá trị của i, hoặc
g(ai) + h(ai) = 1 với ít nhất
n
2 giá trị của i.
Giả sử g(ai) + h(ai) =1 với k giá trị của i 2 f1; : : : ; ng. Rõ ràng k < n
(vì đa thức g(x) + h(x) 1 6= 0 và có bậc nhỏ hơn thực sự n.
Giả sử k
n
2 2, vì nếu k <
n
2 thì xét đa thức g(x) + h(x) + 1.
Giả sử g(ai) + h(ai) = 1 với i 2 f1; : : : ; kg, nếu không như vậy thì ta
đặt lại thứ tự của chỉ số.
Suy ra
Dịch vụ viết thuê đề tài – KB Zalo/Tele 0917.193.864 – luanvantrust.com
Kham thảo miễn phí – Kết bạn Zalo/Tele mình 0917.193.864
g(x) + h(x) 1 = (x a1)(x a2): : : (x ak)w(x) với w(x) 2 Z[x]:
Dịch vụ viết thuê đề tài – KB Zalo/Tele 0917.193.864 – luanvantrust.com
Kham thảo miễn phí – Kết bạn Zalo/Tele mình 0917.193.864
44
Cho x = ak+1 với chú ý g(ak+1) + h(ak+1) = 1 ta được
2 = (ak+1 a1)(ak+1 a2): : : (ak+1 ak)w(ak+1):
Thế nhưng trị tuyệt đối của một tích gồm ít nhất là bốn thừa số nguyên tố
khác nhau và khác 0 thì luôn luôn lớn hơn 2. Vậy nên k 3 và dẫn đến 2 k 3.
Xét k = 3. Nếu n > 4 thì ta có
(a4 a1)(a4
(a5 a1)(a5
a2)(a4
a2)(a5
a3)w(a4) =
a3)w(a5) =
2;
2
Do đó fa1; a2; a3g fa4 1; a4 2; a4 2g. Nhưng a5 > a4 > a3 nên ja5 a3j 3
với mọi i 2 f1; 2; 3g. Điều này vô lý.
Nếu n = 4 thì
g(x) + h(x) 1 = (x a1)(x a2)w(x)
do deg w 1 do deg g 3 và deg h 3. Suy ra (ai
i = 3; 4.
Nếu deg w = 1 thì deg g(x) = 3, h(x) = x a
và
a1)(ai a2)w(ai) = 2 với
0 = f (a) = (aa1)(a a2)(a a3)(a a4) = 2:
Vậy w(x) là đa thức hằng. Do đó deg g = deg h = 2, w(x) 2 và w(a3) =
w(a4) = 2. Từ đó
1 = (a3 a1)(a3 a2) và fa1; a2g fa3 1; a3 + 1g:
Tương tự ta cũng có 1 = (a4 a1)(a4 a2) nên fa1; a2g fa4 1; a4 + 1g. Do
đó a4 = a3 vô lý.
Tóm lại, chỉ có n = 3 thỏa mãn yêu cầu của bài toán.
Bài toán 2.2.3. Cho n số ai 2 Z. Chứng minh rằng đa thức
P(x) = (x a1)(x a2): : : (x an) 1
bất khả quy trong Z[x].
Dịch vụ viết thuê đề tài – KB Zalo/Tele 0917.193.864 – luanvantrust.com
Kham thảo miễn phí – Kết bạn Zalo/Tele mình 0917.193.864
45
Lời giải. Giả sử ta có phan tích P(x) = G(x)H(x) trong Z[x] và deg G < n,
deg H < n.
Ta có P(ai) = G(ai)H(ai) = 1: Vậy G(ai) và H(ai) nhận hai giá trị 1 nên
G(ai)+H(ai) = 0 Khi đó đa thức G(x)+H(x) có deg(G+H) < n mà có n
nghiệm. Suy ra G(x) + H(x) 0, tức là G(x) = H(x). Do đó P(x) = [H(x)]
2
. So
sánh hệ số của xn
của hai vế thì vế trái lớn hơn không, vế phải nhỏ hơn
không, điều này vô lý, nên ta có điều phải chứng minh.
Bài toán 2.2.4. Cho n số ai 2 Z. Chứng minh rằng đa thức
P(x) = (x a1)
2
(x a2)
2
: : : (xan)
2
+ 1
bất khả quy trong Z[x].
Lời giải. Vì P(x) > 0 với mọi x nên đa thức P(x) vô nghiệm. Suy ra hai
nhân tử G(x) và H(x) cũng vô nghiệm nên không đổi dấu.
Giả sử G(x) và H(x) dương với mọi x. Ta có H(ai)G(ai) = 1 nên suy ra
H(ai) = 1 và G(ai) = 1. Nếu deg H < n suy ra H(x) 1 có n nghiệm. Kéo
theo rằng H(x) 1. Điều này vô lý, nên deg H = deg G = n, suy ra
H(x) 1 = A(x a1): : : (x an);
G(x) 1 = B(x a1): : : (x an):
Thế vào ta có
(x a1)
2
(x a2)
2
: : : (xan)
2
+ 1
= [1 + A(x a1): : : (x an)][1 + B(x a1): : : (x an)]:
So sánh hệ số của x
2n
thì ta có 1 = AB. So sánh hệ số tự do
a
2
1a
2
2 : : : a
2
n + 1 = 1 + (A + B)a1a2 : : : an + a
2
1a
2
2 : : : a
2
n:
Suy ra A + B = 0. Do đó 1 = AB = A
2
. Điều này vô lý. Vậy bài toán được
chứng minh.
Dịch vụ viết thuê đề tài – KB Zalo/Tele 0917.193.864 – luanvantrust.com
Kham thảo miễn phí – Kết bạn Zalo/Tele mình 0917.193.864
46
Chương 3
Một số chủ đề khác

More Related Content

Similar to Luận văn thạc sĩ - Đa thức trong các bài toán thi học sinh giỏi.doc

Luận án tiến sĩ toán học ngưỡng chính tắc của hàm chỉnh hình và hàm đa điều h...
Luận án tiến sĩ toán học ngưỡng chính tắc của hàm chỉnh hình và hàm đa điều h...Luận án tiến sĩ toán học ngưỡng chính tắc của hàm chỉnh hình và hàm đa điều h...
Luận án tiến sĩ toán học ngưỡng chính tắc của hàm chỉnh hình và hàm đa điều h...https://www.facebook.com/garmentspace
 
Giải tích các hàm nhiều biến.pdf
Giải tích các hàm nhiều biến.pdfGiải tích các hàm nhiều biến.pdf
Giải tích các hàm nhiều biến.pdfMan_Ebook
 
Luận văn: Nghiên cứu DIDACTIC về dạy học các bài toán tối ưu trong chủ đề giả...
Luận văn: Nghiên cứu DIDACTIC về dạy học các bài toán tối ưu trong chủ đề giả...Luận văn: Nghiên cứu DIDACTIC về dạy học các bài toán tối ưu trong chủ đề giả...
Luận văn: Nghiên cứu DIDACTIC về dạy học các bài toán tối ưu trong chủ đề giả...Viết thuê trọn gói ZALO 0934573149
 
Xây Dựng Hệ Thống Phân Lịch Thi Tín Chỉ Tại Trường Cao Đẳng Thương Mại Đà Nẵn...
Xây Dựng Hệ Thống Phân Lịch Thi Tín Chỉ Tại Trường Cao Đẳng Thương Mại Đà Nẵn...Xây Dựng Hệ Thống Phân Lịch Thi Tín Chỉ Tại Trường Cao Đẳng Thương Mại Đà Nẵn...
Xây Dựng Hệ Thống Phân Lịch Thi Tín Chỉ Tại Trường Cao Đẳng Thương Mại Đà Nẵn...Dịch vụ viết thuê Luận Văn - ZALO 0932091562
 
Luận văn thạc sĩ toán học
Luận văn thạc sĩ toán họcLuận văn thạc sĩ toán học
Luận văn thạc sĩ toán họcDang Van Ly
 
Vmo 2015-solution-1421633776
Vmo 2015-solution-1421633776Vmo 2015-solution-1421633776
Vmo 2015-solution-1421633776Nguyen Van Tai
 
Luận Văn Các Nguyên Lý Biến Phân Thường Dùng Trong Cơ Học Công Trình.docx
Luận Văn Các Nguyên Lý Biến Phân Thường Dùng Trong Cơ Học Công Trình.docxLuận Văn Các Nguyên Lý Biến Phân Thường Dùng Trong Cơ Học Công Trình.docx
Luận Văn Các Nguyên Lý Biến Phân Thường Dùng Trong Cơ Học Công Trình.docxsividocz
 
Luận án tiến sĩ toán học tính hyperbolic của không gian phức và nhóm các cr t...
Luận án tiến sĩ toán học tính hyperbolic của không gian phức và nhóm các cr t...Luận án tiến sĩ toán học tính hyperbolic của không gian phức và nhóm các cr t...
Luận án tiến sĩ toán học tính hyperbolic của không gian phức và nhóm các cr t...https://www.facebook.com/garmentspace
 

Similar to Luận văn thạc sĩ - Đa thức trong các bài toán thi học sinh giỏi.doc (20)

Một số thuật toán phân tích số nguyên hiện đại và ứng dụng.doc
Một số thuật toán phân tích số nguyên hiện đại và ứng dụng.docMột số thuật toán phân tích số nguyên hiện đại và ứng dụng.doc
Một số thuật toán phân tích số nguyên hiện đại và ứng dụng.doc
 
Luận án tiến sĩ toán học ngưỡng chính tắc của hàm chỉnh hình và hàm đa điều h...
Luận án tiến sĩ toán học ngưỡng chính tắc của hàm chỉnh hình và hàm đa điều h...Luận án tiến sĩ toán học ngưỡng chính tắc của hàm chỉnh hình và hàm đa điều h...
Luận án tiến sĩ toán học ngưỡng chính tắc của hàm chỉnh hình và hàm đa điều h...
 
Bat Đang Thức V I Hàm Loi B Ph N Và Ứng Dụng.docx
Bat Đang Thức V I Hàm Loi B Ph N Và Ứng Dụng.docxBat Đang Thức V I Hàm Loi B Ph N Và Ứng Dụng.docx
Bat Đang Thức V I Hàm Loi B Ph N Và Ứng Dụng.docx
 
M T So L P Phương Trình Diophantine.docx
M T So L P Phương Trình Diophantine.docxM T So L P Phương Trình Diophantine.docx
M T So L P Phương Trình Diophantine.docx
 
Giải tích các hàm nhiều biến.pdf
Giải tích các hàm nhiều biến.pdfGiải tích các hàm nhiều biến.pdf
Giải tích các hàm nhiều biến.pdf
 
Bat Phương Trình Hàm Sinh B I Các Đại Lư Ng Trung Bình B C Tùy Ý Và Các Dạng ...
Bat Phương Trình Hàm Sinh B I Các Đại Lư Ng Trung Bình B C Tùy Ý Và Các Dạng ...Bat Phương Trình Hàm Sinh B I Các Đại Lư Ng Trung Bình B C Tùy Ý Và Các Dạng ...
Bat Phương Trình Hàm Sinh B I Các Đại Lư Ng Trung Bình B C Tùy Ý Và Các Dạng ...
 
Luận văn: Nghiên cứu DIDACTIC về dạy học các bài toán tối ưu trong chủ đề giả...
Luận văn: Nghiên cứu DIDACTIC về dạy học các bài toán tối ưu trong chủ đề giả...Luận văn: Nghiên cứu DIDACTIC về dạy học các bài toán tối ưu trong chủ đề giả...
Luận văn: Nghiên cứu DIDACTIC về dạy học các bài toán tối ưu trong chủ đề giả...
 
Đề tài: Về môđun giả BUCHSBAUM, HAY
Đề tài: Về môđun giả BUCHSBAUM, HAYĐề tài: Về môđun giả BUCHSBAUM, HAY
Đề tài: Về môđun giả BUCHSBAUM, HAY
 
Xây Dựng Hệ Thống Phân Lịch Thi Tín Chỉ Tại Trường Cao Đẳng Thương Mại Đà Nẵn...
Xây Dựng Hệ Thống Phân Lịch Thi Tín Chỉ Tại Trường Cao Đẳng Thương Mại Đà Nẵn...Xây Dựng Hệ Thống Phân Lịch Thi Tín Chỉ Tại Trường Cao Đẳng Thương Mại Đà Nẵn...
Xây Dựng Hệ Thống Phân Lịch Thi Tín Chỉ Tại Trường Cao Đẳng Thương Mại Đà Nẵn...
 
Luận văn: Giải gần đúng phương trình phi tuyến và phương trình vi phân trên m...
Luận văn: Giải gần đúng phương trình phi tuyến và phương trình vi phân trên m...Luận văn: Giải gần đúng phương trình phi tuyến và phương trình vi phân trên m...
Luận văn: Giải gần đúng phương trình phi tuyến và phương trình vi phân trên m...
 
Luận văn thạc sĩ toán học
Luận văn thạc sĩ toán họcLuận văn thạc sĩ toán học
Luận văn thạc sĩ toán học
 
Kỹ thuật biến đổi tâm tỷ cự và ứng dụng vào giải Toán.doc
Kỹ thuật biến đổi tâm tỷ cự và ứng dụng vào giải Toán.docKỹ thuật biến đổi tâm tỷ cự và ứng dụng vào giải Toán.doc
Kỹ thuật biến đổi tâm tỷ cự và ứng dụng vào giải Toán.doc
 
M T So Dạng Toán Ve Dãy So Sinh B I Các Hàm So Sơ Cap.docx
M T So Dạng Toán Ve Dãy So Sinh B I Các Hàm So Sơ Cap.docxM T So Dạng Toán Ve Dãy So Sinh B I Các Hàm So Sơ Cap.docx
M T So Dạng Toán Ve Dãy So Sinh B I Các Hàm So Sơ Cap.docx
 
Vmo 2015-solution-1421633776
Vmo 2015-solution-1421633776Vmo 2015-solution-1421633776
Vmo 2015-solution-1421633776
 
Phương Pháp Bình Phương Nhỏ Nhất Và Ứng Dụng.doc
Phương Pháp Bình Phương Nhỏ Nhất Và Ứng Dụng.docPhương Pháp Bình Phương Nhỏ Nhất Và Ứng Dụng.doc
Phương Pháp Bình Phương Nhỏ Nhất Và Ứng Dụng.doc
 
Luận Văn Các Nguyên Lý Biến Phân Thường Dùng Trong Cơ Học Công Trình.docx
Luận Văn Các Nguyên Lý Biến Phân Thường Dùng Trong Cơ Học Công Trình.docxLuận Văn Các Nguyên Lý Biến Phân Thường Dùng Trong Cơ Học Công Trình.docx
Luận Văn Các Nguyên Lý Biến Phân Thường Dùng Trong Cơ Học Công Trình.docx
 
Luận án tiến sĩ toán học tính hyperbolic của không gian phức và nhóm các cr t...
Luận án tiến sĩ toán học tính hyperbolic của không gian phức và nhóm các cr t...Luận án tiến sĩ toán học tính hyperbolic của không gian phức và nhóm các cr t...
Luận án tiến sĩ toán học tính hyperbolic của không gian phức và nhóm các cr t...
 
Phương Trình Bậc Bốn Và Các Hệ Thức Hình Học Trong Tứ Giác Hai Tâm.docx
Phương Trình Bậc Bốn Và Các Hệ Thức Hình Học Trong Tứ Giác Hai Tâm.docxPhương Trình Bậc Bốn Và Các Hệ Thức Hình Học Trong Tứ Giác Hai Tâm.docx
Phương Trình Bậc Bốn Và Các Hệ Thức Hình Học Trong Tứ Giác Hai Tâm.docx
 
Bất đẳng thức Trong lớp các hàm lượng giác và lượng giác ngược.docx
Bất đẳng thức Trong lớp các hàm lượng giác và lượng giác ngược.docxBất đẳng thức Trong lớp các hàm lượng giác và lượng giác ngược.docx
Bất đẳng thức Trong lớp các hàm lượng giác và lượng giác ngược.docx
 
Luận văn: Nghiên cứu didactic việc dạy học hàm số, HAY, 9đ
Luận văn: Nghiên cứu didactic việc dạy học hàm số, HAY, 9đLuận văn: Nghiên cứu didactic việc dạy học hàm số, HAY, 9đ
Luận văn: Nghiên cứu didactic việc dạy học hàm số, HAY, 9đ
 

More from Dịch vụ viết thuê đề tài trọn gói ☎☎☎ Liên hệ ZALO/TELE: 0973.287.149 👍👍

More from Dịch vụ viết thuê đề tài trọn gói ☎☎☎ Liên hệ ZALO/TELE: 0973.287.149 👍👍 (20)

Phân tích các yếu tố ảnh hưởng đến lòng trung thành của nhân viên tại khách s...
Phân tích các yếu tố ảnh hưởng đến lòng trung thành của nhân viên tại khách s...Phân tích các yếu tố ảnh hưởng đến lòng trung thành của nhân viên tại khách s...
Phân tích các yếu tố ảnh hưởng đến lòng trung thành của nhân viên tại khách s...
 
Nghiên cứu về phát triển hệ thống kênh phân phối sản phẩm của các doanh nghiệ...
Nghiên cứu về phát triển hệ thống kênh phân phối sản phẩm của các doanh nghiệ...Nghiên cứu về phát triển hệ thống kênh phân phối sản phẩm của các doanh nghiệ...
Nghiên cứu về phát triển hệ thống kênh phân phối sản phẩm của các doanh nghiệ...
 
CƠ SỞ LÝ LUẬN VỀ THƯƠNG HIỆU.docx
CƠ SỞ LÝ LUẬN VỀ THƯƠNG HIỆU.docxCƠ SỞ LÝ LUẬN VỀ THƯƠNG HIỆU.docx
CƠ SỞ LÝ LUẬN VỀ THƯƠNG HIỆU.docx
 
Cơ sở lý luận của việc nâng cao chất lượng phục vụ tại bộ phận đón tiếp của k...
Cơ sở lý luận của việc nâng cao chất lượng phục vụ tại bộ phận đón tiếp của k...Cơ sở lý luận của việc nâng cao chất lượng phục vụ tại bộ phận đón tiếp của k...
Cơ sở lý luận của việc nâng cao chất lượng phục vụ tại bộ phận đón tiếp của k...
 
Cơ sở lý luận về phát triển thị trƣờng khách inbound dưới góc độ marketing củ...
Cơ sở lý luận về phát triển thị trƣờng khách inbound dưới góc độ marketing củ...Cơ sở lý luận về phát triển thị trƣờng khách inbound dưới góc độ marketing củ...
Cơ sở lý luận về phát triển thị trƣờng khách inbound dưới góc độ marketing củ...
 
Cơ sở lý luận về thị trường và sử dụng marketing nhằm mở rộng thị trường của ...
Cơ sở lý luận về thị trường và sử dụng marketing nhằm mở rộng thị trường của ...Cơ sở lý luận về thị trường và sử dụng marketing nhằm mở rộng thị trường của ...
Cơ sở lý luận về thị trường và sử dụng marketing nhằm mở rộng thị trường của ...
 
Tính toán thiết kế chế tạo và vận hành thử nghiệm hệ thống cấp đông I-Q-F thẳ...
Tính toán thiết kế chế tạo và vận hành thử nghiệm hệ thống cấp đông I-Q-F thẳ...Tính toán thiết kế chế tạo và vận hành thử nghiệm hệ thống cấp đông I-Q-F thẳ...
Tính toán thiết kế chế tạo và vận hành thử nghiệm hệ thống cấp đông I-Q-F thẳ...
 
Tính toán, thiết kế máy sấy bơm nhiệt sấy thanh long cắt lát với năng suất 20...
Tính toán, thiết kế máy sấy bơm nhiệt sấy thanh long cắt lát với năng suất 20...Tính toán, thiết kế máy sấy bơm nhiệt sấy thanh long cắt lát với năng suất 20...
Tính toán, thiết kế máy sấy bơm nhiệt sấy thanh long cắt lát với năng suất 20...
 
Nghiên cứu nhiệt phân gỗ nhằm nâng cao chất lượng sản phẩm than hoa.doc
Nghiên cứu nhiệt phân gỗ nhằm nâng cao chất lượng sản phẩm than hoa.docNghiên cứu nhiệt phân gỗ nhằm nâng cao chất lượng sản phẩm than hoa.doc
Nghiên cứu nhiệt phân gỗ nhằm nâng cao chất lượng sản phẩm than hoa.doc
 
Hoàn thiện quy trình sản xuất thanh long sấy bằng phương pháp sấy đối ...
Hoàn thiện quy trình sản xuất thanh long sấy bằng phương pháp sấy đối ...Hoàn thiện quy trình sản xuất thanh long sấy bằng phương pháp sấy đối ...
Hoàn thiện quy trình sản xuất thanh long sấy bằng phương pháp sấy đối ...
 
Nghiên cứu ứng dụng hệ điều khiển dự báo để điều khiển mức nước bao hơi của n...
Nghiên cứu ứng dụng hệ điều khiển dự báo để điều khiển mức nước bao hơi của n...Nghiên cứu ứng dụng hệ điều khiển dự báo để điều khiển mức nước bao hơi của n...
Nghiên cứu ứng dụng hệ điều khiển dự báo để điều khiển mức nước bao hơi của n...
 
ĐỒ ÁN - BÁO CÁO MÔ HÌNH KHO LẠNH DÀN TRẢI.doc
ĐỒ ÁN - BÁO CÁO MÔ HÌNH KHO LẠNH DÀN TRẢI.docĐỒ ÁN - BÁO CÁO MÔ HÌNH KHO LẠNH DÀN TRẢI.doc
ĐỒ ÁN - BÁO CÁO MÔ HÌNH KHO LẠNH DÀN TRẢI.doc
 
ĐỒ ÁN - Tính toán thiết kế máy sấy khoai lang năng suất 100 kg mẻ.doc
ĐỒ ÁN - Tính toán thiết kế máy sấy khoai lang năng suất 100 kg mẻ.docĐỒ ÁN - Tính toán thiết kế máy sấy khoai lang năng suất 100 kg mẻ.doc
ĐỒ ÁN - Tính toán thiết kế máy sấy khoai lang năng suất 100 kg mẻ.doc
 
Đồ án tốt nghiệp - Sấy bã mía, 9 điểm.doc
Đồ án tốt nghiệp - Sấy bã mía, 9 điểm.docĐồ án tốt nghiệp - Sấy bã mía, 9 điểm.doc
Đồ án tốt nghiệp - Sấy bã mía, 9 điểm.doc
 
Hoàn thiện quy trình sản xuất thanh long sấy bằng phương pháp sấy đối lưu.doc
Hoàn thiện quy trình sản xuất thanh long sấy bằng phương pháp sấy đối lưu.docHoàn thiện quy trình sản xuất thanh long sấy bằng phương pháp sấy đối lưu.doc
Hoàn thiện quy trình sản xuất thanh long sấy bằng phương pháp sấy đối lưu.doc
 
ĐỒ ÁN - Điều khiển lưu lượng không khí trong phòng sạch thông qua biến tần.doc
ĐỒ ÁN - Điều khiển lưu lượng không khí trong phòng sạch thông qua biến tần.docĐỒ ÁN - Điều khiển lưu lượng không khí trong phòng sạch thông qua biến tần.doc
ĐỒ ÁN - Điều khiển lưu lượng không khí trong phòng sạch thông qua biến tần.doc
 
ĐỒ ÁN - Tính toán thiết bị sấy nấm kểu sấy hầm, năng suất nhập liệu 650kgmẻ.doc
ĐỒ ÁN - Tính toán thiết bị sấy nấm kểu sấy hầm, năng suất nhập liệu 650kgmẻ.docĐỒ ÁN - Tính toán thiết bị sấy nấm kểu sấy hầm, năng suất nhập liệu 650kgmẻ.doc
ĐỒ ÁN - Tính toán thiết bị sấy nấm kểu sấy hầm, năng suất nhập liệu 650kgmẻ.doc
 
Thiết kế nhà máy sản xuất bia năng suất 91,8 triệu lít sản phẩm năm.docx
Thiết kế nhà máy sản xuất bia năng suất 91,8 triệu lít sản phẩm năm.docxThiết kế nhà máy sản xuất bia năng suất 91,8 triệu lít sản phẩm năm.docx
Thiết kế nhà máy sản xuất bia năng suất 91,8 triệu lít sản phẩm năm.docx
 
Tính toán thiết kế hệ thống sấy thùng quay sấy cà phê nhân theo năng suất nhậ...
Tính toán thiết kế hệ thống sấy thùng quay sấy cà phê nhân theo năng suất nhậ...Tính toán thiết kế hệ thống sấy thùng quay sấy cà phê nhân theo năng suất nhậ...
Tính toán thiết kế hệ thống sấy thùng quay sấy cà phê nhân theo năng suất nhậ...
 
Thiết kế hệ thống sấy thùng quay sấy bắp với năng suất 800 kgh.docx
Thiết kế hệ thống sấy thùng quay sấy bắp với năng suất 800 kgh.docxThiết kế hệ thống sấy thùng quay sấy bắp với năng suất 800 kgh.docx
Thiết kế hệ thống sấy thùng quay sấy bắp với năng suất 800 kgh.docx
 

Recently uploaded

Campbell _2011_ - Sinh học - Tế bào - Ref.pdf
Campbell _2011_ - Sinh học - Tế bào - Ref.pdfCampbell _2011_ - Sinh học - Tế bào - Ref.pdf
Campbell _2011_ - Sinh học - Tế bào - Ref.pdfTrnHoa46
 
GNHH và KBHQ - giao nhận hàng hoá và khai báo hải quan
GNHH và KBHQ - giao nhận hàng hoá và khai báo hải quanGNHH và KBHQ - giao nhận hàng hoá và khai báo hải quan
GNHH và KBHQ - giao nhận hàng hoá và khai báo hải quanmyvh40253
 
powerpoint mẫu họp phụ huynh cuối kì 2 học sinh lớp 7 bgs
powerpoint mẫu họp phụ huynh cuối kì 2 học sinh lớp 7 bgspowerpoint mẫu họp phụ huynh cuối kì 2 học sinh lớp 7 bgs
powerpoint mẫu họp phụ huynh cuối kì 2 học sinh lớp 7 bgsNmmeomeo
 
GIÁO ÁN DẠY THÊM (KẾ HOẠCH BÀI DẠY BUỔI 2) - TIẾNG ANH 7 GLOBAL SUCCESS (2 CỘ...
GIÁO ÁN DẠY THÊM (KẾ HOẠCH BÀI DẠY BUỔI 2) - TIẾNG ANH 7 GLOBAL SUCCESS (2 CỘ...GIÁO ÁN DẠY THÊM (KẾ HOẠCH BÀI DẠY BUỔI 2) - TIẾNG ANH 7 GLOBAL SUCCESS (2 CỘ...
GIÁO ÁN DẠY THÊM (KẾ HOẠCH BÀI DẠY BUỔI 2) - TIẾNG ANH 7 GLOBAL SUCCESS (2 CỘ...Nguyen Thanh Tu Collection
 
ĐỀ CHÍNH THỨC KỲ THI TUYỂN SINH VÀO LỚP 10 THPT CÁC TỈNH THÀNH NĂM HỌC 2020 –...
ĐỀ CHÍNH THỨC KỲ THI TUYỂN SINH VÀO LỚP 10 THPT CÁC TỈNH THÀNH NĂM HỌC 2020 –...ĐỀ CHÍNH THỨC KỲ THI TUYỂN SINH VÀO LỚP 10 THPT CÁC TỈNH THÀNH NĂM HỌC 2020 –...
ĐỀ CHÍNH THỨC KỲ THI TUYỂN SINH VÀO LỚP 10 THPT CÁC TỈNH THÀNH NĂM HỌC 2020 –...Nguyen Thanh Tu Collection
 
Đề cương môn giải phẫu......................
Đề cương môn giải phẫu......................Đề cương môn giải phẫu......................
Đề cương môn giải phẫu......................TrnHoa46
 
cac-cau-noi-tthcm.pdf-cac-cau-noi-tthcm-
cac-cau-noi-tthcm.pdf-cac-cau-noi-tthcm-cac-cau-noi-tthcm.pdf-cac-cau-noi-tthcm-
cac-cau-noi-tthcm.pdf-cac-cau-noi-tthcm-KhnhHuyn546843
 
TÀI LIỆU BỒI DƯỠNG HỌC SINH GIỎI KỸ NĂNG VIẾT ĐOẠN VĂN NGHỊ LUẬN XÃ HỘI 200 C...
TÀI LIỆU BỒI DƯỠNG HỌC SINH GIỎI KỸ NĂNG VIẾT ĐOẠN VĂN NGHỊ LUẬN XÃ HỘI 200 C...TÀI LIỆU BỒI DƯỠNG HỌC SINH GIỎI KỸ NĂNG VIẾT ĐOẠN VĂN NGHỊ LUẬN XÃ HỘI 200 C...
TÀI LIỆU BỒI DƯỠNG HỌC SINH GIỎI KỸ NĂNG VIẾT ĐOẠN VĂN NGHỊ LUẬN XÃ HỘI 200 C...Nguyen Thanh Tu Collection
 
SLIDE - Tu van, huong dan cong tac tuyen sinh-2024 (đầy đủ chi tiết).pdf
SLIDE - Tu van, huong dan cong tac tuyen sinh-2024 (đầy đủ chi tiết).pdfSLIDE - Tu van, huong dan cong tac tuyen sinh-2024 (đầy đủ chi tiết).pdf
SLIDE - Tu van, huong dan cong tac tuyen sinh-2024 (đầy đủ chi tiết).pdfhoangtuansinh1
 
BỘ LUYỆN NGHE VÀO 10 TIẾNG ANH DẠNG TRẮC NGHIỆM 4 CÂU TRẢ LỜI - CÓ FILE NGHE.pdf
BỘ LUYỆN NGHE VÀO 10 TIẾNG ANH DẠNG TRẮC NGHIỆM 4 CÂU TRẢ LỜI - CÓ FILE NGHE.pdfBỘ LUYỆN NGHE VÀO 10 TIẾNG ANH DẠNG TRẮC NGHIỆM 4 CÂU TRẢ LỜI - CÓ FILE NGHE.pdf
BỘ LUYỆN NGHE VÀO 10 TIẾNG ANH DẠNG TRẮC NGHIỆM 4 CÂU TRẢ LỜI - CÓ FILE NGHE.pdfNguyen Thanh Tu Collection
 
30 ĐỀ PHÁT TRIỂN THEO CẤU TRÚC ĐỀ MINH HỌA BGD NGÀY 22-3-2024 KỲ THI TỐT NGHI...
30 ĐỀ PHÁT TRIỂN THEO CẤU TRÚC ĐỀ MINH HỌA BGD NGÀY 22-3-2024 KỲ THI TỐT NGHI...30 ĐỀ PHÁT TRIỂN THEO CẤU TRÚC ĐỀ MINH HỌA BGD NGÀY 22-3-2024 KỲ THI TỐT NGHI...
30 ĐỀ PHÁT TRIỂN THEO CẤU TRÚC ĐỀ MINH HỌA BGD NGÀY 22-3-2024 KỲ THI TỐT NGHI...Nguyen Thanh Tu Collection
 
3-BẢNG MÃ LỖI CỦA CÁC HÃNG ĐIỀU HÒA .pdf - ĐIỆN LẠNH BÁCH KHOA HÀ NỘI
3-BẢNG MÃ LỖI CỦA CÁC HÃNG ĐIỀU HÒA .pdf - ĐIỆN LẠNH BÁCH KHOA HÀ NỘI3-BẢNG MÃ LỖI CỦA CÁC HÃNG ĐIỀU HÒA .pdf - ĐIỆN LẠNH BÁCH KHOA HÀ NỘI
3-BẢNG MÃ LỖI CỦA CÁC HÃNG ĐIỀU HÒA .pdf - ĐIỆN LẠNH BÁCH KHOA HÀ NỘIĐiện Lạnh Bách Khoa Hà Nội
 
PHƯƠNG THỨC VẬN TẢI ĐƯỜNG SẮT TRONG VẬN TẢI
PHƯƠNG THỨC VẬN TẢI ĐƯỜNG SẮT TRONG VẬN TẢIPHƯƠNG THỨC VẬN TẢI ĐƯỜNG SẮT TRONG VẬN TẢI
PHƯƠNG THỨC VẬN TẢI ĐƯỜNG SẮT TRONG VẬN TẢImyvh40253
 
Các điều kiện bảo hiểm trong bảo hiểm hàng hoá
Các điều kiện bảo hiểm trong bảo hiểm hàng hoáCác điều kiện bảo hiểm trong bảo hiểm hàng hoá
Các điều kiện bảo hiểm trong bảo hiểm hàng hoámyvh40253
 
TÀI LIỆU BỒI DƯỠNG HỌC SINH GIỎI LÝ LUẬN VĂN HỌC NĂM HỌC 2023-2024 - MÔN NGỮ ...
TÀI LIỆU BỒI DƯỠNG HỌC SINH GIỎI LÝ LUẬN VĂN HỌC NĂM HỌC 2023-2024 - MÔN NGỮ ...TÀI LIỆU BỒI DƯỠNG HỌC SINH GIỎI LÝ LUẬN VĂN HỌC NĂM HỌC 2023-2024 - MÔN NGỮ ...
TÀI LIỆU BỒI DƯỠNG HỌC SINH GIỎI LÝ LUẬN VĂN HỌC NĂM HỌC 2023-2024 - MÔN NGỮ ...Nguyen Thanh Tu Collection
 
kinh tế chính trị mác lênin chương hai và hàng hoá và sxxhh
kinh tế chính trị mác lênin chương hai và hàng hoá và sxxhhkinh tế chính trị mác lênin chương hai và hàng hoá và sxxhh
kinh tế chính trị mác lênin chương hai và hàng hoá và sxxhhdtlnnm
 
Kiểm tra cuối học kì 1 sinh học 12 đề tham khảo
Kiểm tra cuối học kì 1 sinh học 12 đề tham khảoKiểm tra cuối học kì 1 sinh học 12 đề tham khảo
Kiểm tra cuối học kì 1 sinh học 12 đề tham khảohoanhv296
 
sách sinh học đại cương - Textbook.pdf
sách sinh học đại cương   -   Textbook.pdfsách sinh học đại cương   -   Textbook.pdf
sách sinh học đại cương - Textbook.pdfTrnHoa46
 
30 ĐỀ PHÁT TRIỂN THEO CẤU TRÚC ĐỀ MINH HỌA BGD NGÀY 22-3-2024 KỲ THI TỐT NGHI...
30 ĐỀ PHÁT TRIỂN THEO CẤU TRÚC ĐỀ MINH HỌA BGD NGÀY 22-3-2024 KỲ THI TỐT NGHI...30 ĐỀ PHÁT TRIỂN THEO CẤU TRÚC ĐỀ MINH HỌA BGD NGÀY 22-3-2024 KỲ THI TỐT NGHI...
30 ĐỀ PHÁT TRIỂN THEO CẤU TRÚC ĐỀ MINH HỌA BGD NGÀY 22-3-2024 KỲ THI TỐT NGHI...Nguyen Thanh Tu Collection
 

Recently uploaded (20)

Campbell _2011_ - Sinh học - Tế bào - Ref.pdf
Campbell _2011_ - Sinh học - Tế bào - Ref.pdfCampbell _2011_ - Sinh học - Tế bào - Ref.pdf
Campbell _2011_ - Sinh học - Tế bào - Ref.pdf
 
GNHH và KBHQ - giao nhận hàng hoá và khai báo hải quan
GNHH và KBHQ - giao nhận hàng hoá và khai báo hải quanGNHH và KBHQ - giao nhận hàng hoá và khai báo hải quan
GNHH và KBHQ - giao nhận hàng hoá và khai báo hải quan
 
powerpoint mẫu họp phụ huynh cuối kì 2 học sinh lớp 7 bgs
powerpoint mẫu họp phụ huynh cuối kì 2 học sinh lớp 7 bgspowerpoint mẫu họp phụ huynh cuối kì 2 học sinh lớp 7 bgs
powerpoint mẫu họp phụ huynh cuối kì 2 học sinh lớp 7 bgs
 
GIÁO ÁN DẠY THÊM (KẾ HOẠCH BÀI DẠY BUỔI 2) - TIẾNG ANH 7 GLOBAL SUCCESS (2 CỘ...
GIÁO ÁN DẠY THÊM (KẾ HOẠCH BÀI DẠY BUỔI 2) - TIẾNG ANH 7 GLOBAL SUCCESS (2 CỘ...GIÁO ÁN DẠY THÊM (KẾ HOẠCH BÀI DẠY BUỔI 2) - TIẾNG ANH 7 GLOBAL SUCCESS (2 CỘ...
GIÁO ÁN DẠY THÊM (KẾ HOẠCH BÀI DẠY BUỔI 2) - TIẾNG ANH 7 GLOBAL SUCCESS (2 CỘ...
 
ĐỀ CHÍNH THỨC KỲ THI TUYỂN SINH VÀO LỚP 10 THPT CÁC TỈNH THÀNH NĂM HỌC 2020 –...
ĐỀ CHÍNH THỨC KỲ THI TUYỂN SINH VÀO LỚP 10 THPT CÁC TỈNH THÀNH NĂM HỌC 2020 –...ĐỀ CHÍNH THỨC KỲ THI TUYỂN SINH VÀO LỚP 10 THPT CÁC TỈNH THÀNH NĂM HỌC 2020 –...
ĐỀ CHÍNH THỨC KỲ THI TUYỂN SINH VÀO LỚP 10 THPT CÁC TỈNH THÀNH NĂM HỌC 2020 –...
 
Đề cương môn giải phẫu......................
Đề cương môn giải phẫu......................Đề cương môn giải phẫu......................
Đề cương môn giải phẫu......................
 
cac-cau-noi-tthcm.pdf-cac-cau-noi-tthcm-
cac-cau-noi-tthcm.pdf-cac-cau-noi-tthcm-cac-cau-noi-tthcm.pdf-cac-cau-noi-tthcm-
cac-cau-noi-tthcm.pdf-cac-cau-noi-tthcm-
 
TÀI LIỆU BỒI DƯỠNG HỌC SINH GIỎI KỸ NĂNG VIẾT ĐOẠN VĂN NGHỊ LUẬN XÃ HỘI 200 C...
TÀI LIỆU BỒI DƯỠNG HỌC SINH GIỎI KỸ NĂNG VIẾT ĐOẠN VĂN NGHỊ LUẬN XÃ HỘI 200 C...TÀI LIỆU BỒI DƯỠNG HỌC SINH GIỎI KỸ NĂNG VIẾT ĐOẠN VĂN NGHỊ LUẬN XÃ HỘI 200 C...
TÀI LIỆU BỒI DƯỠNG HỌC SINH GIỎI KỸ NĂNG VIẾT ĐOẠN VĂN NGHỊ LUẬN XÃ HỘI 200 C...
 
SLIDE - Tu van, huong dan cong tac tuyen sinh-2024 (đầy đủ chi tiết).pdf
SLIDE - Tu van, huong dan cong tac tuyen sinh-2024 (đầy đủ chi tiết).pdfSLIDE - Tu van, huong dan cong tac tuyen sinh-2024 (đầy đủ chi tiết).pdf
SLIDE - Tu van, huong dan cong tac tuyen sinh-2024 (đầy đủ chi tiết).pdf
 
1 - MÃ LỖI SỬA CHỮA BOARD MẠCH BẾP TỪ.pdf
1 - MÃ LỖI SỬA CHỮA BOARD MẠCH BẾP TỪ.pdf1 - MÃ LỖI SỬA CHỮA BOARD MẠCH BẾP TỪ.pdf
1 - MÃ LỖI SỬA CHỮA BOARD MẠCH BẾP TỪ.pdf
 
BỘ LUYỆN NGHE VÀO 10 TIẾNG ANH DẠNG TRẮC NGHIỆM 4 CÂU TRẢ LỜI - CÓ FILE NGHE.pdf
BỘ LUYỆN NGHE VÀO 10 TIẾNG ANH DẠNG TRẮC NGHIỆM 4 CÂU TRẢ LỜI - CÓ FILE NGHE.pdfBỘ LUYỆN NGHE VÀO 10 TIẾNG ANH DẠNG TRẮC NGHIỆM 4 CÂU TRẢ LỜI - CÓ FILE NGHE.pdf
BỘ LUYỆN NGHE VÀO 10 TIẾNG ANH DẠNG TRẮC NGHIỆM 4 CÂU TRẢ LỜI - CÓ FILE NGHE.pdf
 
30 ĐỀ PHÁT TRIỂN THEO CẤU TRÚC ĐỀ MINH HỌA BGD NGÀY 22-3-2024 KỲ THI TỐT NGHI...
30 ĐỀ PHÁT TRIỂN THEO CẤU TRÚC ĐỀ MINH HỌA BGD NGÀY 22-3-2024 KỲ THI TỐT NGHI...30 ĐỀ PHÁT TRIỂN THEO CẤU TRÚC ĐỀ MINH HỌA BGD NGÀY 22-3-2024 KỲ THI TỐT NGHI...
30 ĐỀ PHÁT TRIỂN THEO CẤU TRÚC ĐỀ MINH HỌA BGD NGÀY 22-3-2024 KỲ THI TỐT NGHI...
 
3-BẢNG MÃ LỖI CỦA CÁC HÃNG ĐIỀU HÒA .pdf - ĐIỆN LẠNH BÁCH KHOA HÀ NỘI
3-BẢNG MÃ LỖI CỦA CÁC HÃNG ĐIỀU HÒA .pdf - ĐIỆN LẠNH BÁCH KHOA HÀ NỘI3-BẢNG MÃ LỖI CỦA CÁC HÃNG ĐIỀU HÒA .pdf - ĐIỆN LẠNH BÁCH KHOA HÀ NỘI
3-BẢNG MÃ LỖI CỦA CÁC HÃNG ĐIỀU HÒA .pdf - ĐIỆN LẠNH BÁCH KHOA HÀ NỘI
 
PHƯƠNG THỨC VẬN TẢI ĐƯỜNG SẮT TRONG VẬN TẢI
PHƯƠNG THỨC VẬN TẢI ĐƯỜNG SẮT TRONG VẬN TẢIPHƯƠNG THỨC VẬN TẢI ĐƯỜNG SẮT TRONG VẬN TẢI
PHƯƠNG THỨC VẬN TẢI ĐƯỜNG SẮT TRONG VẬN TẢI
 
Các điều kiện bảo hiểm trong bảo hiểm hàng hoá
Các điều kiện bảo hiểm trong bảo hiểm hàng hoáCác điều kiện bảo hiểm trong bảo hiểm hàng hoá
Các điều kiện bảo hiểm trong bảo hiểm hàng hoá
 
TÀI LIỆU BỒI DƯỠNG HỌC SINH GIỎI LÝ LUẬN VĂN HỌC NĂM HỌC 2023-2024 - MÔN NGỮ ...
TÀI LIỆU BỒI DƯỠNG HỌC SINH GIỎI LÝ LUẬN VĂN HỌC NĂM HỌC 2023-2024 - MÔN NGỮ ...TÀI LIỆU BỒI DƯỠNG HỌC SINH GIỎI LÝ LUẬN VĂN HỌC NĂM HỌC 2023-2024 - MÔN NGỮ ...
TÀI LIỆU BỒI DƯỠNG HỌC SINH GIỎI LÝ LUẬN VĂN HỌC NĂM HỌC 2023-2024 - MÔN NGỮ ...
 
kinh tế chính trị mác lênin chương hai và hàng hoá và sxxhh
kinh tế chính trị mác lênin chương hai và hàng hoá và sxxhhkinh tế chính trị mác lênin chương hai và hàng hoá và sxxhh
kinh tế chính trị mác lênin chương hai và hàng hoá và sxxhh
 
Kiểm tra cuối học kì 1 sinh học 12 đề tham khảo
Kiểm tra cuối học kì 1 sinh học 12 đề tham khảoKiểm tra cuối học kì 1 sinh học 12 đề tham khảo
Kiểm tra cuối học kì 1 sinh học 12 đề tham khảo
 
sách sinh học đại cương - Textbook.pdf
sách sinh học đại cương   -   Textbook.pdfsách sinh học đại cương   -   Textbook.pdf
sách sinh học đại cương - Textbook.pdf
 
30 ĐỀ PHÁT TRIỂN THEO CẤU TRÚC ĐỀ MINH HỌA BGD NGÀY 22-3-2024 KỲ THI TỐT NGHI...
30 ĐỀ PHÁT TRIỂN THEO CẤU TRÚC ĐỀ MINH HỌA BGD NGÀY 22-3-2024 KỲ THI TỐT NGHI...30 ĐỀ PHÁT TRIỂN THEO CẤU TRÚC ĐỀ MINH HỌA BGD NGÀY 22-3-2024 KỲ THI TỐT NGHI...
30 ĐỀ PHÁT TRIỂN THEO CẤU TRÚC ĐỀ MINH HỌA BGD NGÀY 22-3-2024 KỲ THI TỐT NGHI...
 

Luận văn thạc sĩ - Đa thức trong các bài toán thi học sinh giỏi.doc

  • 1. Dịch vụ viết thuê đề tài – KB Zalo/Tele 0917.193.864 – luanvantrust.com Kham thảo miễn phí – Kết bạn Zalo/Tele mình 0917.193.864 ĐẠI HỌC THÁI NGUYÊN TRƯỜNG ĐẠI HỌC KHOA HỌC --------------  ------------- NGUYỄN THANH TÙNG ĐA THỨC TRONG CÁC BÀI TOÁN THI HỌC SINH GIỎI LUẬN VĂN THẠC SĨ TOÁN HỌC THÁI NGUYÊN -
  • 2. Dịch vụ viết thuê đề tài – KB Zalo/Tele 0917.193.864 – luanvantrust.com Kham thảo miễn phí – Kết bạn Zalo/Tele mình 0917.193.864 ĐẠI HỌC THÁI NGUYÊN TRƯỜNG ĐẠI HỌC KHOA HỌC --------------  ------------- NGUYỄN THANH TÙNG ĐA THỨC TRONG CÁC BÀI TOÁN THI HỌC SINH GIỎI LUẬN VĂN THẠC SĨ TOÁN HỌC Chuyên ngành: Phương pháp Toán sơ cấp Mã số: 60 46 01 13 NGƯỜI HƯỚNG DẪN KHOA HỌC GS.TSKH. Đặng Hùng Thắng THÁI NGUYÊN -
  • 3. Dịch vụ viết thuê đề tài – KB Zalo/Tele 0917.193.864 – luanvantrust.com Kham thảo miễn phí – Kết bạn Zalo/Tele mình 0917.193.864 1 Mục lục Danh sách kí hiệu 3 Mở đầu 4 Chương 1. Đa thức một biến 7 1.1 Định nghĩa và các tính chất . . . . . . . . . . . . . . . . . . . . . 7 1.1.1 Định nghĩa . . . . . . . . . . . . . . . . . . . . . . . . . 7 1.1.2 Các phép tính trên đa thức . . . . . . . . . . . . . . . . . 8 1.1.3 Các tính chất cơ bản . . . . . . . . . . . . . . . . . . . . 9 1.2 Phép chia đa thức. Ước chung lớn nhất và nhỏ nhất . . . . . . . . 11 1.2.1 Phép chia đa thức . . . . . . . . . . . . . . . . . . . . . . 11 1.2.2 Thuật toán Euclide . . . . . . . . . . . . . . . . . . . . . 11 1.3 Nghiệm của đa thức. Phương trình bậc cao . . . . . . . . . . . . . 16 1.3.1 Nghiệm của đa thức . . . . . . . . . . . . . . . . . . . . 16 1.3.2 Phương trình bậc cao . . . . . . . . . . . . . . . . . . . . 22 1.4 Đạo hàm của đa thức. Định lý Taylor . . . . . . . . . . . . . . . . 32 Chương 2. Đa thức bất khả quy 36 2.1 Đa thức bất khả quy . . . . . . . . . . . . . . . . . . . . . . . . . 36 2.1.1 Đa thức với hệ số thực và phức . . . . . . . . . . . . . . . 37 2.1.2 Đa thức bất khả quy của vành Q[x] . . . . . . . . . . . . . 40
  • 4. Dịch vụ viết thuê đề tài – KB Zalo/Tele 0917.193.864 – luanvantrust.com Kham thảo miễn phí – Kết bạn Zalo/Tele mình 0917.193.864 2 2.2 Một số bài toán điển hình . . . . . . . . . . . . . . . . . . . . . . 42 Chương 3. Một số chủ đề khác 46 3.1 Đa thức nhiều biến . . . . . . . . . . . . . . . . . . . . . . . . . 46 3.2 Đa thức đối xứng . . . . . . . . . . . . . . . . . . . . . . . . . . 49 3.3 Phương trình hàm đa thức . . . . . . . . . . . . . . . . . . . . . . 53 3.4 Đa thức Chebyshev . . . . . . . . . . . . . . . . . . . . . . . . . 56 3.4.1 Định nghĩa - Tính chất . . . . . . . . . . . . . . . . . . . 57 3.4.2 Một số bài toán chọn lọc . . . . . . . . . . . . . . . . . . 58 Kết luận 63 Tài liệu tham khảo 64
  • 5. Dịch vụ viết thuê đề tài – KB Zalo/Tele 0917.193.864 – luanvantrust.com Kham thảo miễn phí – Kết bạn Zalo/Tele mình 0917.193.864 3 Danh sách kí hiệu Z vành số nguyên Q trường số hữu tỷ R trường số thực C trường số phức R vành F trường R[x] vành đa thức với hệ số trên vành R deg P(x) bậc của đa thức P(x) . . đa thức Q(x) là ước của đa thức P(x) P(x) . Q(x), Q(x) j P(x) gcd(P(X); Q(X)) ước chung lớn nhất của P(X) và Q(X) a b (mod p) a đồng dư với b theo modulo p åi m =1 ai ký hiệu tổng a1 + a2 + + am Õi m =1 bi ký hiệu tích b1b2 bm
  • 6. Dịch vụ viết thuê đề tài – KB Zalo/Tele 0917.193.864 – luanvantrust.com Kham thảo miễn phí – Kết bạn Zalo/Tele mình 0917.193.864 4 Mở đầu Đa thức là một đối tượng quan trọng của Toán học cả về mặt lý thuyết cũng như ứng dụng. Đối với Toán học phổ thông, học sinh làm quen với các phép toán trên đa thức (cộng trừ nhân chia), giải các phương trình bậc nhất, bậc hai và một số dạng phương trình bậc cao. Trong các kỳ thi học sinh giỏi quốc gia và quốc tế, chủ đề đa thức cũng được khai thác sâu hơn với các bài toán hay và tương đối khó về phương trình đại số bậc cao, phương trình hàm đa thức, đathức bất khả quy, tính chia hết của đa thức . . . Các bài toán nâng cao về đa thức xuất hiện cũng khá nhiều trong các tạp chí toán học cho học sinh khá giỏi (như Tạp chí Toán học và Tuổi trẻ, Kvant, Crux,. . . ). Tuy nhiên hiện nay có ít các tài liệu về tiếng Việt trình bày một cách hệ thống cả lý thuyết và bài tập về đa thức, với định hướng bồi dưỡng học sinh giỏi Toán và bồi dưỡng giáo viên dạy chuyên Toán. Mục tiêu của luận văn là tìm hiểu một cách đầy đủ những kết quả quan trọng của đa thức có nhiều ứng dụng trong Toán phổ thông. Trên cơ sở đó, phân loại và hệ thống hoá (theo dạng cũng như phương pháp giải) các bài tập nâng cao về đa thức đã có cũng như sáng tác, bổ sung thêm những bài toán mới. Chúng tôi rất cố gắng để luận văn này trở thành một tài liệu tham khảo tốt, thiết thực phục vụ cho việc giảng dạy học sinh giỏi và bồi dưỡng giáo viên. Thông qua việc viết luận văn học viên sẽ mở rộng nâng cao hiểu biết về đa thức, hình thành các kỹ năng giải các bài toán khó về đa thức, kỹ năng tìm kiếm thu thập chọn lọc các thông tin. Nội dung của luận văn được trình bày trong ba chương như sau:
  • 7. Dịch vụ viết thuê đề tài – KB Zalo/Tele 0917.193.864 – luanvantrust.com Kham thảo miễn phí – Kết bạn Zalo/Tele mình 0917.193.864 5 Chương 1. Đa thức một biến. Trong chương này chúng tôi sẽ trình bày ngắn gọn về định nghĩa và các tính chất của đa thức. Các vấn đề nền tảng về phép chia đa thức, ước - bội, nghiệm và phương trình bậc cao, đạo hàm và khai triển Taylor sẽ được trình bày. Chương 2. Đa thức bất khả quy. Đa thức bất khả quy là một trong những chủ đề trọng tâm của lý thuyết các đa thức. Nó vừa mang tính chất lý thuyết, vừa mang tính ứng dụng, đặc biệt là các bài tập nâng cao trong các đề thi có tính chất tuyển chọn. Chương này chúng tôi tập trung nghiên cứu các đa thức bất khả quy trên các vành (trường) số quen biết của toán học sơ cấp. Chương 3. Một số chủ đề khác. Chương này dành để nghiên cứu một số vấn đề nâng cao của lý thuyết đa thức, mà mục đích của nó là để hiểu biết sâu sắc hơn lý thuyết, đồng thời là nền tảng cho các ứng dụng. Các vấn đề được quan tâm trong chương này là các đa thức nhiều biến, đa thức đối xứng, phương trình hàm đa thức và đa thức Chebyshev. Luận văn này được thực hiện tại Trường Đại học Khoa học - Đại học Thái Nguyên và hoàn thành với sự hướng dẫn của GS.TSKH. Đặng Hùng Thắng (Trường ĐHKHTN - ĐHQG Hà Nội). Tác giả xin được bày tỏ lòng biết ơn chân thành và sâu sắc tới người hướng dẫn khoa học của mình, người đã đặt vấn đề nghiên cứu, dành nhiều thời gian hướng dẫn và tận tình giải đáp những thắc mắc của tác giả trong suốt quá trình làm luận văn. Tác giả xin trân trọng cảm ơn Ban Giám hiệu Trường Đại học Khoa học - Đại học Thái Nguyên, Ban Chủ nhiệm Khoa Toán–Tin, cùng các giảng viên đã tham gia giảng dạy, đã tạo mọi điều kiện tốt nhất để tác giả học tập và nghiên cứu. Tác giả muốn gửi những lời cảm ơn tốt đẹp nhất tới tập thể lớp Cao học Toán khóa 9 (2015-2017) đã động viên và giúp đỡ tác giả rất nhiều trong suốt quá trình học tập. Nhân dịp này, tác giả cũng xin chân thành cảm ơn Sở Giáo dục và Đào tạo Hải
  • 8. Dịch vụ viết thuê đề tài – KB Zalo/Tele 0917.193.864 – luanvantrust.com Kham thảo miễn phí – Kết bạn Zalo/Tele mình 0917.193.864 6 Phòng, Ban Giám hiệu và các đồng nghiệp ở Trường THPT Hùng Vương đã tạo điều kiện cho tác giả hoàn thành tốt nhiệm vụ học tập và công tác của mình. Cuối cùng, tác giả muốn dành những lời cảm ơn đặc biệt nhất đến bố mẹ và đại gia đình đã luôn động viên và chia sẻ những khó khăn để tác giả hoàn thành tốt luận văn này. Thái Nguyên, ngày 02 tháng 11 năm 2017 Tác giả Nguyễn Thanh Tùng
  • 9. Dịch vụ viết thuê đề tài – KB Zalo/Tele 0917.193.864 – luanvantrust.com Kham thảo miễn phí – Kết bạn Zalo/Tele mình 0917.193.864 7 Chương 1 Đa thức một biến 1.1 Định nghĩa và các tính chất Phần này chúng tôi sẽ trình bày ngắn gọn nhất có thể lý thuyết các đa thức một biến. Những chi tiết hơn có thể tham khảo Lê Thị Thanh Nhàn [5] hoặc Nguyễn Văn Mậu [3]. 1.1.1 Định nghĩa Giả sử R là một vành giao hoán có đơn vị. Định nghĩa 1.1.1. Biểu thức có dạng anx n + an 1x n 1 + : : : + a1x + a0 với an 6= 0 trong đó an; an 1; : : : ; a1; a0 là những phần tử thuộc vành R, được gọi là một đa thức trên vành R. Trong định nghĩa này, ai được gọi là các hệ số của đa thức, hệ số an được gọi là hệ số bậc cao nhất của đa thức, số tự nhiên n được gọi là bậc của đa thức, ký hiệu là deg P(x), x được gọi là ẩn, hay biến hay đối số của đa thức, an được gọi là hệ số cao nhất, a0 được gọi là hệ số tự do của đa thức. Nếu ai = 0 với i = 1; 2; : : : ; n 1 và a0 6= 0 thì ta có bậc của đa thức là không. Nếu ai = 0 với i = 1; 2; : : : ; n thì f (x) = 0, ta gọi đa thức này là đa thức không.
  • 10. Dịch vụ viết thuê đề tài – KB Zalo/Tele 0917.193.864 – luanvantrust.com Kham thảo miễn phí – Kết bạn Zalo/Tele mình 0917.193.864 8 Nói chung người ta không định nghĩa bậc của đa thức không nhưng ta coi bậc của nó là ¥. Hai đa thức f và g được gọi là bằng nhau, và viết f = g, nếu chúng cùng là đa thức không, hoặc cả hai khác đa thức không, đồng thời deg f = deg g và các hệ số tương ứng bằng nhau. Tập hợp tất cả các đa thức lấy hệ số trong vành R được ký hiệu là R[x], và được gọi là vành đa thức trên R. Khi R là một trường, thì vành R[x] là một vành giao hoán có đơn vị. Với lý do là ứng dụng lý thuyết đa thức trong các bài thi học sinh giỏi, hay nói chung là các kỳ thi có tính chất tuyển chọn, luận văn này thường xét R là Z, Q, R, hoặc C, khi đó các đa thức thuộc Z[x], Q[x], R[x], hoặc C[x] được gọi tên lần lượt là các đa thức nguyên, đa thức hữu tỷ, đa thức thực, hoặc đa thức phức. 1.1.2 Các phép tính trên đa thức Cho hai đa thức f (x) = anx n + an 1x n 1 + : : : + a1x + a0; g(x) = bnx n + bn 1x n 1 + : : : + b1x + b0: Ta định nghĩa các phép tính số học như sau Phép cộng f (x)+g(x) = (an +bn)x n +(an 1 +bn 1)x n 1 +: : :+(a1 +b1)x+(a0 +b0): Phép trừ f (x) g(x) = (an bn)x n +(an 1 bn 1)x n 1 +: : :+(a1 b1)x (a0 b0): Phép nhân f (x)g(x) = c2nx 2n + c2n 1x 2n 1 + : : : + c1x + c0
  • 11. Dịch vụ viết thuê đề tài – KB Zalo/Tele 0917.193.864 – luanvantrust.com Kham thảo miễn phí – Kết bạn Zalo/Tele mình 0917.193.864 9 trong đó ck = a0bk + a1bk 1 + : : : + akb0 với k = 0; 1; : : : ; n: 1.1.3 Các tính chất cơ bản Định lí 1.1.2. Giả sử F một trường. Với hai đa thức f (x) và g(x) thuộc vành đa thức F[x], luôn tồn tại cặp đa thức q(x) và r(x) duy nhất thuộc vành đa thức F[x] để f (x) = g(x) q(x) + r(x); với deg r(x) < deg g(x). Ta gọi q(x) và r(x) lần lượt là đa thức thương (hoặc thương) và đa thức dư (hay dư) trong phép chia f (x) cho g(x). Nếu r(x) = 0 thì ta nói f (x) chia hết cho g(x), hay g(x) chia hết f (x) hay f (x) là bội của g(x) hay g(x) là ước của f (x). Ta sẽ kí hiệu là f .. . g hay g j f . Giả sử f (x) = anx n + an 1x n 1 + : : : + a1x + a0 là một đa thức thuộc vành đa thức R[x]. Xét phần tử a 2 R bất kỳ. Khi đó f (a) = ana n + an 1a n 1 + : : : + a1a + a0 được gọi là giá trị của đa thức f (x) tại a. Nếu f (a) = 0 thì a được gọi là một nghiệm của đa thức f (x). Nếu tồn tại . k k+1 . (x a) a) thì k 2 N; k > 1 sao cho f (x) . nhưng f (x) không chia hết cho (x a được gọi là nghiệm bội k của đa thức f (x). Đặc biệt, k = 1 thì a được gọi là nghiệm đơn, k = 2 thì a được gọi là nghiệm kép. Bài toán tìm nghiệm của đa thức f (x) = anx n + an 1x n 1 + : : : + a1x + a0 với an 6= 0: trong vành R được gọi là giải phương trình đại số bậc n trong R.
  • 12. Dịch vụ viết thuê đề tài – KB Zalo/Tele 0917.193.864 – luanvantrust.com Kham thảo miễn phí – Kết bạn Zalo/Tele mình 0917.193.864 10 Định lí 1.1.3. Giả sử F một trường, a 2 F và f (x) 2 F[x]. Dư của phép chia f (x) cho (x a) chính là f (a). Định lí 1.1.4 (Định lý Bézout). Phần tử a 2 F là nghiệm của đa thức f (x) 2 F[x] khi và chỉ khi f (x) chia hết cho (x a). Định lý sau đây cho ta một đánh giá về số nghiệm của một đa thức thực. Định lí 1.1.5. Một đa thức thực bậc n đều có không quá n nghiệm thực. Định lý này có một số hệ quả sau đây. Hệ quả 1.1.6. Đa thức có vô số nghiệm là đa thức không. Hệ quả 1.1.7. Nếu một đa thức có bậc không vượt quá n mà nhận cùng một giá trị tại n + 1 điểm khác nhau của ẩn thì đa thức đó là đa thức hằng. Hệ quả 1.1.8. Hai đa thức có bậc không vượt quá n mà nhận cùng một giá trị thỏa mãn bằng nhau tại n + 1 giá trị khác nhau của ẩn thì hai đa thức đó đồng nhất bằng nhau. Trong trường hợp đa thức phức, ta có kết quả sau đây về số lượng nghiệm của nó. Định lí 1.1.9. Một đa thức phức bậc n có đúng n nghiệm tính cả bội. Cuối cùng của mục này, ta sẽ trình bày không chứng minh một kết quả về dạng biểu diễn của các đa thức thực. Định lí 1.1.10. Bất kỳ đa thức thực f (x) 2 R[x] nào có bậc n và hệ số bậc cao nhất an 6= 0 có thể phân tích một cách duy nhất (không tính thứ tự) thành các nhân tử m f (x) = an Õ(x di) s Õ (x 2 + bkx + ck) i=1 k=1 với di; bk; sk 2 R, 2s + m = n và b 2 k 4ck < 0 với m; n 2 N.
  • 13. Dịch vụ viết thuê đề tài – KB Zalo/Tele 0917.193.864 – luanvantrust.com Kham thảo miễn phí – Kết bạn Zalo/Tele mình 0917.193.864 11 Trong lý thuyết đa thức, một phần rất quan trọng đó là khảo sát các nghiệm hữu tỷ và nguyên của một đa thức. Ta có định lý sau đây. Định lí 1.1.11. Xét đa thức nguyên f (x) 2 Z[x] có dạng f (x) = a0x n + a1x n 1 + : : : + an 1x + an; a0 6= 0: Nghiệm hữu tỷ nếu có x = q p với gcd(p; q) = 1 thì p là ước của hệ số tự do và q là ước của hệ số bậc cao nhất, tức là p j an, q j a0. Hệ quả 1.1.12. Xét đa thức nguyên f (x) 2 Z[x] có dạng chuẩn tắc, tức là hệ số bậc cao nhất là 1. Khi đó mỗi nghiệm hữu tỷ nếu có của đa thức f (x) đều là nghiệm nguyên. 1.2 Phép chia đa thức. Ước chung lớn nhất và nhỏ nhất 1.2.1 Phép chia đa thức Định nghĩa 1.2.1. Một đa thức d(x) chia hết hai đa thức f (x) và g(x) gọi là ước chung của f (x) và g(x). Nếu d(x) là một ước chung chia hết cho mọi ước chung khác của hai đa thức f (x) và g(x) đúng thì ta gọi d(x) là ước chung lớn nhất của f (x) và g(x) . Rõ ràng các ước chung lớn nhất sai khác hằng số, để bảo đảm tính duy nhất ta có thể quy ước chọn ước chung lớn nhất dạng chuẩn tắc (hệ số cao nhất bằng 1). Ta kí hiệu là d(x) = ( f (x); g(x)) = gcd(( f (x); g(x)): 1.2.2 Thuật toán Euclide Ta chia liên tiếp f (x) = g(x) q(x) + r(x) g(x) = r(x) q1(x) + r1(x)
  • 14. Dịch vụ viết thuê đề tài – KB Zalo/Tele 0917.193.864 – luanvantrust.com Kham thảo miễn phí – Kết bạn Zalo/Tele mình 0917.193.864 12 r(x) = r1(x) q2(x) + r2(x) : : : rk 2(x) = rk 1(x) qk(x) + rk(x) rk 1(x) = rk(x qk+1(x): Khi đó ( f (x); g(x)) = rk (x) với rk (x) = c rk(x) đa thức có hệ số bậc cao nhất là 1 (người ta thường gọi những đa thức này là đa thức chuẩn tắc hay đa thức monic). Kết quả nếu d(x) = ( f (x); g(x)) thì tồn tại hai đa thức u(x); v(x) 2 R[x] để có biểu diễn f (x) u(x) + g(x) v(x) = d(x): Hơn nữa ta có thể chọn deg u < deg g và deg v < deg f : Bây giờ ta xét một số ví dụ để hiểu sâu sắc hơn lý thuyết. Các ví dụ này chúng tôi tham khảo trong Lê Hoành Phò [6]. Bài toán 1.2.2. Cho P(x) = x + x 3 + x 9 + x 27 + x 81 + x 243 . Tìm dư của phép chia đa thức P(x) cho (a) x 1, (b) x 2 1. Lời giải. (a) Ta có P(x) = (x 1)Q(x) + r(x) với deg r(x) < deg(x 1) = 1. Suy ra deg r(x) = 0 nên dư r(x) = c. Do đó P(x) = (x 1) Q(x) + c. Chọn x = 1 suy ra P(1) = c hay c = P(1) = 6: Vậy dư r(x) = 6: (b) Ta có P(x) = (x 2 1) H(x) + s(x) với deg s(x) 1. Vậy P(x) = (x 2 1) H(x) + ax + b: Chọn x = 1 ta có P(1) = a + b = 6; x = 1 ta có P( 1) = a + b = 6. Do đó a = 6, b = 0. Vậy ta có đa thức dư là r(x) = 6x:
  • 15. Dịch vụ viết thuê đề tài – KB Zalo/Tele 0917.193.864 – luanvantrust.com Kham thảo miễn phí – Kết bạn Zalo/Tele mình 0917.193.864 13 Bài toán 1.2.3 (Trung Quốc 1981). Tìm dư của phép chia (a) x 12 + x 8 + x 4 + 1 cho x 3 + x 2 + x + 1; (b) f (x 100 ) cho f (x) với f (x) = x 99 + x 98 + : : : + x + 1. Lời giải. (a) Ta có x 12 + x 8 + x 4 + 1 = (x 3 + x 2 + x + 1)(x 9 x 8 + 2x 5 + 2x 4 + 3x 3) + 4: Suy ra dư là 4. (b) Ta có f (x) = x 99 + x 98 + : : : + x + 1 nên f (x 100 ) = x 9900 + x 9800 + : : : + x 100 + 1 = f (x) x 9800 + 2x 9701 2x 9700 + 3x 9601 3x 9600 + : : : + 99x 99) + 100 nên dư là 100. Bài toán 1.2.4. Xác định a và b để đa thức f (x) = 6x 4 7x 3 + ax 2 + 3x + 2 chia hết cho x 2 x + b. Lời giải. Lấy đa thức f (x) = 6x4 7x3 + ax2 + 3x + 2 chia cho g(x) = x2 x + b thì được thương q(x) = 6x2 x + (a 5b 1) và phần dư r(x) = (a 5b + 2)x + ( ab 6b 2 + a + b + 2). .. Vì f (x) . g(x) nên r(x) = 0 suy ra a 5b + 2 = 0; (1.1) ab + 6b 2 + b + 2 = 0: (1.2) Từ (1.1) suy ra a = 5b 2. Thay vào (1.2) ta có b 2 + 3b + 2 = 0 nên b = 1 hoặc b = 2: Với b = 1 thì a = 7. Với b = 2 thì a = 12:
  • 16. Dịch vụ viết thuê đề tài – KB Zalo/Tele 0917.193.864 – luanvantrust.com Kham thảo miễn phí – Kết bạn Zalo/Tele mình 0917.193.864 Vậy f (x) = 6x 4 7x 3 7x 2 + 2 hay f (x) = 6x 4 7x 3 12x 2 + 3x + 2.
  • 17. Dịch vụ viết thuê đề tài – KB Zalo/Tele 0917.193.864 – luanvantrust.com Kham thảo miễn phí – Kết bạn Zalo/Tele mình 0917.193.864 14 Bài toán 1.2.5. Tìm ước chung lớn nhất của hai đa thức f (x) = x 2 + x 3 3x 2 4x 1 và g(x) = x 3 + x 2 x 1: Lời giải. Ta thực hiện các phép chia liên tiếp và hỗ trợ với phép nhân thêm hằng số f (x) = q(x) g(x) + r(x) thì q(x) = x; r(x) = 2x 2 3x 1 và g(x) = q1(x) r(x) + r1(x) thì
  • 18. Dịch vụ viết thuê đề tài – KB Zalo/Tele 0917.193.864 – luanvantrust.com Kham thảo miễn phí – Kết bạn Zalo/Tele mình 0917.193.864 q 1 (x) = 1 x 1 ; r 1 (x) = 3x 3 ; 2 2 2 2 và tiếp tục r(x) = q2(x) r1(x) + r2(x) thì q 2(x) = 2(2x + 1); r2(x) = 0 3 Do đó ( f (x); g(x)) = x + 1 với quy ước lấy hệ số cao nhất bằng 1 từ r1(x) = 3 x 3 : 2 2 Bài toán 1.2.6 (New York 1973, Bỉ 1981). Chứng minh rằng với mọi giá trị n 2 N, đa thức (x + 1)2n+1 + xn+2 chia hết cho đa thức x2 + x + 1: Lời giải. Ta chứng minh bằng quy nạp theo n 2 N. Với n = 0 khẳng định đúng vì khi đó (x + 1) 2n+1 + x n+2 = x 2 + x: Giả sử khẳng định đúng với n 1, nghĩa là (x + 1) 2n 1 + x n+1 chia hết cho x 2 + x + 1: Khi đó đa thức (x + 1)2n+1 + xn+2 = (x + 1)2 (x + 1)2n 1 + x xn+1 = (x2 + 2x + 1)(x + 1)2n 1 + x xn+1 = (x 2 + x + 1)(x + 1) 2n 1 + x[x + 1) 2n 1 + x n+1 ] chia hết cho x2 + x + 1. Suy ra điều chứng minh với n.
  • 19. Dịch vụ viết thuê đề tài – KB Zalo/Tele 0917.193.864 – luanvantrust.com Kham thảo miễn phí – Kết bạn Zalo/Tele mình 0917.193.864 15 n . k Bài toán 1.2.7. Cho hai số nguyên dương n và k. Chứng minh rằng x 1 . 1 . x khi và chỉ khi n là bội số của k. n . k Lời giải. Ta có thể phát biểu bài toán dưới dạng: Để x . 1 điều kiện cần 1 . x và đủ là n là bội số của k. Điều kiện đủ. Giả sử n là bội số của k tức là n = km với m nguyên dương. Khi đó x n 1 = x km 1 = (x k ) m 1 = (x k 1)[x (k 1) + x k(m 1) + x k(m 2) + : : : + x k + 1]: n . k Đẳng thức này chứng tỏ rằng x 1 . 1: . x Điều kiện cần. Ta lấy số nguyên dương n chia cho số nguyên dương k. Giả sử q và r là thương và số dư trong phép chia, tức là có n = kq + r với 0 r < k. Khi đó x n 1 = xkq+r 1 = xkq+r x r + x r 1 = x r (x kq 1) + x r 1: (1.3) kq . k n . k Ở trên ta đã chứng minh x 1 . 1:Vì vậy nếu x 1 . 1 thì từ (1.3) suy ra . x . x r . k r . k n . k x . 1: Nhưng r < k nên x . 1 khi r = 0. Thành ra nếu x . 1 1 . x 1 . x 1 . x thì r = 0, tức là n = kq, hay n là bội số của k. Bài toán 1.2.8 (Rumani 1962). Tìm điều kiện của các số nguyên p và q sao cho đa thức (a) P(x) = x 2 + px + q nhận cùng giá trị chẵn (lẻ) với mọi x 2 Z. (b) Q(x) = x 3 + px + q nhận các giá trị chia hết cho 3 với mọi x 2 Z. Lời giải. (a) P(x) nhận giá trị cùng chẵn (hoặc lẻ) với mọi x 2 Z khi và chỉ khi mỗi số P(x + 1) P(x) = 2x + 1 + p chia hết cho 2 nghĩa là p lẻ.
  • 20. Dịch vụ viết thuê đề tài – KB Zalo/Tele 0917.193.864 – luanvantrust.com Kham thảo miễn phí – Kết bạn Zalo/Tele mình 0917.193.864 16 Khi đó tính chẵn lẻ của P(x) phụ thuộc vào tính chẵn lẻ của q = P(0). Như vậy tất cả giá trị của P(x) là chẵn (lẻ) khi p lẻ và q chẵn (tương ứng q lẻ). (b) Vì Q(x) = x(x 2 + p) + q nên Q(3x) = 3x(9x 2 + p) + q chia hết cho 3. Với giá trị đó thì Q(3x 1) = (3x 1)(9x 2 6x + 1 + p) + q (1 + p) (mod 3) chia hết cho 3 khi và chỉ khi 1 + p chia hết cho 3. Vậy Q(x) chia hết cho 3 (với mọi x 2 Z) khi q = 0, q 2 (mod 3). Bài toán 1.2.9 (Hong Kong 2008). Xét đa thức f (x) = cmx m + cm 1x m 1 + : : : + c1x + c0 với các hệ số ci là những số nguyên khác không. Xây dựng dãy số (an) như sau: a1 = 0 và an+1 = f (an) với n = 1; 2; : : : Giả sử i, j là những số nguyên dương và i < j. Chứng minh rằng (a j+1 a j) là bội của (ai+1 ai). Chứng minh. Do f (x) là đa thức hệ số nguyên nên [ f (a) f (b)] chia hết cho (a b), với a và b là hai số nguyên phân biệt. Do đó ai+2 ai+1 = f (ai+1) f (ai) chia hết cho (ai+1 Như vậy nếu i, j là những số nguyên dương và i < j thì (a j+1 (ai+1 ai). Ta có điều phải chứng minh. ai), với mọi i > 0. aj) chia hết cho 1.3 Nghiệm của đa thức. Phương trình bậc cao 1.3.1 Nghiệm của đa thức Bài toán 1.3.1. Cho đa thức bậc chẵn và tất cả hệ số đều lẻ. Chứng minh đa thức không có nghiệm hữu tỷ. Chứng minh. Xét P(x) = a0x n + a1x n 1 + : : : + an 1x + an với a0 6= 0.
  • 21. Dịch vụ viết thuê đề tài – KB Zalo/Tele 0917.193.864 – luanvantrust.com Kham thảo miễn phí – Kết bạn Zalo/Tele mình 0917.193.864 17 Với n chẵn, các hệ số ai lẻ. Giả sử đa thức có nghiệm hữu tỷ x = q p thì p j a0, q j an. Suy ra p, q lẻ. Thế x = q p vào đa thức ta có an p n + an 1qp n 1 + : : : + a0q n = 0: Điều này vô lý vì vế trái là tổng của một số lẻ các số hạng lẻ nên không thể bằng 0. Vậy đa thức không có nghiệm hữu tỷ. Ta có điều phải chứng minh. Bài toán 1.3.2. Cho số tự nhiên n 2, chứng minh phương trình xn + xn 1 +:::+ x2 + x +1=0: (1.4) n! (n 1)! 2! 1! không có nghiệm hữu tỷ. Chứng minh. Ta chứng minh bài toán bằng phương pháp phản chứng. Giả sử phương trình đã cho có nghiệm hữu tỷ a. Khi đó a sẽ là nghiệm hữu tỷ của đa thức P(x) = x n + nx n 1 + : : : + n! xk : : : + n! x2 + n! x + n!: k! 2! 1! Nhưng do P(x) là một đa thức bậc n với hệ số nguyên, và hệ số bậc cao nhất, của x n , là 1 nên a phải là số nguyên và ta có an + nan 1 ak a2 + : : : + n! + : : : + n! + n! = 0: (1.5) 1! k! Gọi p là một ước nguyên tố của n. Với mọi k = 1; 2; : : : ; n, kí hiệu rk là số mũ cao nhất của p thỏa mãn k! .. . prk, ta có rk = p + p2 + : : : + ps : (1.6) k k k Với s là số nguyên không âm thỏa mãn p s k < p s+1 , từ (1.6) ta suy ra k k k p2 rk + +:::+ = k < k: p p2 ps p 1 Do đó rn rk > rn k. Suy ra rn rk rn k + 1: Vì vậy ta được n! : pn k+1 với mọi k = 1; 2; : : : ; n: (1.7) k!
  • 22. Dịch vụ viết thuê đề tài – KB Zalo/Tele 0917.193.864 – luanvantrust.com Kham thảo miễn phí – Kết bạn Zalo/Tele mình 0917.193.864 18 Nhưng mà n .. . p nên từ (1.5) ta có an .. . p và do đó a .. . p. Suy ra ak .. . pk với mọi k = 1; 2; : : : ; n. Kết hợp điều này với (1.7) ta được n! a k! k .. . p r n +1 với mọi k = 1; 2; : : : ; n. Từ đây và (1.5) ta suy ra n! .. . p r n +1 : Mâu thuẫn vừa nhận được chứng tỏ giả sử ban đầu là sai và vì vậy ta có điều phải chứng minh. Bài toán 1.3.3 (Việt Nam 1992). Cho đa thức P(x) = 1 + x 2 + x 9 + x n 1 + : : : + x ns + x 1992 : Với n1; : : : ; ns là các số tự nhiên cho trước thỏa mãn 9 < n1 < : : : < ns < 1992: p Chứng minh rằng nghiệm của đa thức P(x) (nếu có) không thể lớn hơn 1 2 5 . Lời giải. Ta có P(x) = 1 + x 2 + x 9 + x n 1 + : : : + x ns + x 1992 : Với x 0 thì P(x) 1 > 0: Ta sẽ chứng minh p !: 1 ; 0 P(x) > 0 với mọi x 2 5 2 Thật vậy với x < 0 và x 6= 1 ta có P(x) 1 + x + x 3 + x 5 + : : : + x 2k+1 + : : : + x 1991 = 1 + x (x 1990 + x 1998 + : : : + 1):(1 x 2 ) (1 x2 ) 1 x996 = 1 + x x2 1 = 1 x 2 + x x 997 : 1 x2 1 p ; 0 thì Mà với x 2 5 2 1 x 2 > 0; x 997 > 0; 1 x 2 + x > 0:
  • 23. Dịch vụ viết thuê đề tài – KB Zalo/Tele 0917.193.864 – luanvantrust.com Kham thảo miễn phí – Kết bạn Zalo/Tele mình 0917.193.864 19 nên 1 p !: P(x) > 0 với mọi x 2 5 ; 0 2 1 p ; +¥. Ta có điều phải chứng minh. Vậy P(x) > 0 với x 2 5 2 Bài toán 1.3.4 (IMO 1976). Cho các đa thức Pk(x), với k = 1; 2; 3 : : : xác định bởi P1(x) = x 2 2; Pi+1 = P1(Pi(x)); với i = 1; 2; 3; : : : Chứng minh rằng Pn(x) = x có 2 n nghiệm thực phân biệt. Lời giải. Ta thu hẹp việc xét nghiệm của phương trình trên đoạn 2 x 2: Đặt x = 2 cos(t). Khi đó, bằng quy nạp ta chứng minh được Pn(x) = 2 cos(2 nt ): Thêm nữa, phương trình Pn(x) = x trở thành cos(2 nt ) = cos(t). Từ đó ta được 2 n nghiệm t = 2kp ; t = 2kp ; k = 1; 2; : : : ; n 2n 1 2n + 1 Vậy phương trình Pn(x) = x có 2 n nghiệm thực phân biệt. Bài toán 1.3.5. Cho đa thức f (x) = a0 +a1x +: : : +anx n có n nghiệm thực. Chứng minh với mọi p > n 1 thì đa thức g(x) = a0 + a1:p:x + a2:p(p 1)x 2 + : : : + an p(p 1): : : (p n + 1)x n : cũng có n nghiệm thực. Lời giải. Để giải bài toán ta xét hai trường hợp Trường hợp 1. Đa thức f (x) không nhận x = 0 làm nghiệm. Ta chứng minh bằng quy nạp.
  • 24. Dịch vụ viết thuê đề tài – KB Zalo/Tele 0917.193.864 – luanvantrust.com Kham thảo miễn phí – Kết bạn Zalo/Tele mình 0917.193.864 Với n = 1 bài toán hiển nhiên đúng .
  • 25. Dịch vụ viết thuê đề tài – KB Zalo/Tele 0917.193.864 – luanvantrust.com Kham thảo miễn phí – Kết bạn Zalo/Tele mình 0917.193.864 20 Giả sử đúng với n = k, ta chứng minh đúng với n = k + 1, tức là nếu đa thức f (x) = a0 + a1x + : : : + ak+1x k+1 có k + 1 nghiệm thực khác 0 thì đa thức g(x) = a0 + pa1x + : : : + p(p1): : : (p k)ak+1x k+1 cũng có k + 1 nghiệm thực khác 0 với mọi p > k. Gọi c là một nghiệm của f (x) thì f (x) = (x c)q(x); (1.8) với q(x) là một đa thức nào đó là đa thức bậc k của x, q(x) = b0 + b1x + : : : + bkxk : (1.9) Thay (1.9) vào (1.8) rồi đồng nhất hệ số ta được a0 = cb0; a1 = cb1 + b0; : : : ak = cbk + bk 1; a k+1 = b k Do đó g(x) = a0 + pa1x + : : : + p(p1): : : (p k)ak+1x k+1 = cb0 + p(cb1 + b0)x + : : : + p(p 1)(p k)bkxk+1 = cQ(x) + pxQ(x) x 2 Q(x) trong đó Q(x) = b0 + b1 px + : : : + p(p 1): : : (p k + 1)bkx k : Do f (x) có k + 1 nghiệm thực khác 0 nên q(x) có k nghiệm thực khác 0. Mặt khác p > k nên p > k 1. Cho nên theo giả thiết quy nạp ta có đa thức Q(x) có k nghiệm thực. Do đó g(x) có k + 1 nghiệm thực. Vậy theo nguyên lý quy nạp, bài toán đúng. Trường hợp 2. Đa thức f (x) nhận x = 0 làm nghiệm.
  • 26. Dịch vụ viết thuê đề tài – KB Zalo/Tele 0917.193.864 – luanvantrust.com Kham thảo miễn phí – Kết bạn Zalo/Tele mình 0917.193.864 21 Giả sử x = 0 là nghiệm bội k của f (x), với k 2 Z+, k n. Khi đó ta có f (x) = akx k + : : : + anx n = (anx n k + : : : + ak)x k và g(x) = p(p 1): : : (p k + 1)ak:x k + : : : + p(p1): : : (p n + 1)an:x n = p(p 1): : : (p k + 1)x k [ak + : : : + (p k): : : (p n + 1)anx n k ]: Vì f (x) có n nghiệm thực nên H(x) = ak + : : : + anx n k có (n k) nghiệm thực khác 0. Do đó áp dụng kết quả của Trường hợp 1 cho H(x) và p 0 = p k > n k 1 (do p > n 1), ta được đa thức R(x) = ak + : : : + (p k): : : (p n + 1)anxn k có n k nghiệm thực. Vậy g(x) có n nghiệm thực. Bài toán 1.3.6 (Trung Quốc 1996). Cho đa thức p(x) bậc 5 có 5 nghiệm thực phân biệt. Tìm số bé nhất của các hệ số khác 0. Lời giải. Xét p(x) = ax 5 + bx 4 + cx 3 + dx + e; a 6= 0: Nếu có 4 hệ số bằng 0 thì b = c = d = e = 0 nên p(x) = ax 5 có nghiệm bội (loại) tức là p(x) không thể có một hệ số khác 0. Do đó p(x) có ít nhất hai hệ số khác 0. Xét p(x) = ax 5 +bx n , n 2 thì p(x) có nghiệm bội. Ta tiếp tục loại trường hợp này. Xét p(x) = ax 5 + dx = ax ax 4 + a có tối đa ba nghiệm. Ta cũng loại. d Do đó p(x) có ít nhất ba hệ số khác 0. Chọn p(x) = x 5 5x 3 + 4x = x(x 2 4) thì p(x) có đúng 5 nghiệm phân biệt và đúng ba hệ số khác 0. Vậy số bé nhất của hệ số khác 0 là 3.
  • 27. Dịch vụ viết thuê đề tài – KB Zalo/Tele 0917.193.864 – luanvantrust.com Kham thảo miễn phí – Kết bạn Zalo/Tele mình 0917.193.864 22 1.3.2 Phương trình bậc cao Lý thuyết giải phương trình bậc 3 tổng quát Xét phương trình đa thức bậc ba ax 3 + bx 2 + cx + d = 0; a 6= 0: (1.10) Ngoài việc tách nhóm số hạng hoặc tìm một nghiệm rồi phân tích nhân tử, dùng hằng đẳng thức, ta có cách giải tổng quát như sau: Trước hết, chia 2 vế cho a 6= 0 đưa về phương trình: x 3 + Bx 2 +Cx + D = 0. Tiếp theo đặt x = y B đưa tiếp về phương trình: y 3 py = q, trong đó 3 b2 2B 3 BC p = C; q = + D: 3 27 3 Có hai hướng để giải phương trình y3 py = q: (1.11) Hướng thứ nhất. Đặt y = u + v và chọn u; v = 3 p thì từ y3 = u3 + v3 + 3uv(u + v) ta có u3 + v3 = q và u3 :v3 = P 27 3 . Vậy u3 ; v3 là nghiệm phương trình Z 3 qZ + p3 = 0: 27 Nếu D < 0 sau này ta dùng số phức để giải quyết. Hướng thứ hai. Nếu p = 0 thì (1.11) tương đương với y 3 = q, tức là y = p q. q Nếu p > 0. Đặt y = 2 p t thì (1.11) tương đương với 3 4t 3 3t = m (1.12) 3p q trong đó m = 3 2p p . p Xét jmj 1, đặt m = cos a thì (1.12) có ba nghiệm t1 = cos a ; t2 = cos a + 2p; t3 = cos a 2p : 3 3 3
  • 28. Dịch vụ viết thuê đề tài – KB Zalo/Tele 0917.193.864 – luanvantrust.com Kham thảo miễn phí – Kết bạn Zalo/Tele mình 0917.193.864 23 Xét jmj > 1, đặt m = 2 1 d 3 + d 1 3 suy ra d 3 = m p m 2 1. Phương trình (1.12) có một nghiệm t = 2 d + d = 2 q q m p m 2 m + pm 2 1 + 1 1 1 3 3 Nếu p < 0. Đặt y = 2. q p t thì (1.11) tương đương với 3 4t 3 + 3t = m: Ta đặt tiếp 1 1 m = k3 suy ra k3 = m p m2 + 1: 2 k3 1 (1.13) Phương trình (1.13) có một nghiệm t = 2 k k = 2 q q : m + pm2 + 1+ m pm2 + 1 1 1 1 3 3 Ta gọi các phương trình bậc ba 4x 3 + 3x + m = 0; 4x 3 3x m = 0 là các dạng phương trình bậc 3 chuẩn tắc. Ý nghĩa cơ bản là mọi phương trình bậc 3 đều đưa về được dạng chuẩn tắc đó. Chú ý thêm khi jmj 1, 4x3 + 3x m = (x a)(4x2 + 4ax + 4a2 + 3): Với q q a = 2 m + pm 2 + 1+ m pm 2 + 1 1 3 3 có D 0 = 12(a 2 + 1) < 0 và 4x 3 3x m = (x b )(4x 2 + 4b x + 4b 2 + 3): Với q q b = 2 m + p m2 1 + m pm 2 1 1 3 3 có D 0 = 12(1 b 2 ) < 0.
  • 29. Dịch vụ viết thuê đề tài – KB Zalo/Tele 0917.193.864 – luanvantrust.com Kham thảo miễn phí – Kết bạn Zalo/Tele mình 0917.193.864 24 Các phương trình bậc 4 đặc biệt 1. Phương trình có dạng ax 4 + bx 2 + c = 0; a 6= 0: (1.14) Đặt t = x2 ; t 0 thì đưa về phương trình bậc hai at 2 + bt + c = 0 2. Phương trình có dạng (x + a) 4 + (x + b) 4 = c: (1.15) Đặt t = x + a+ 2 b thì đưa về phương trình trùng phương At 4 + Bt 2 +C = 0 và giải như trên. 3. Phương trình có dạng (ax 2 + bx + c)(ax 2 + bx + d) = m: (1.16) Đặt t = ax 2 + bx thì đưa về phương trình bậc hai (t + c)(t + d) = m. 4. Phương trình có dạng (x + a)(x + b)(x + c)(x + d) = m: (1.17) Nếu có a + d = b + c thì ghép cặp (x + a)(x + d) và (x + b)(x + c) rồi đặt t = x 2 + (a + d)x = x 2 + (b + c)x để đưa về dạng trên. 5. Phương trình có dạng ax 4 + bx 3 + cx 2 + dx + e = 0: (1.18) Nếu ad2 = eb2 6= 0 thì chia hai vế cho x2 6= 0 rồi đặt t = x + ax e (Phương trình quy hồi mở rộng bậc bốn).
  • 30. Dịch vụ viết thuê đề tài – KB Zalo/Tele 0917.193.864 – luanvantrust.com Kham thảo miễn phí – Kết bạn Zalo/Tele mình 0917.193.864 25 Phương trình quy hồi (đối xứng hệ số) Xét phương trình a0x n + a1x n 1 + a2x n 2 + + an 2x 2 + an 1x + an = 0 (1.19) trong đó a0 = an; a1 = an 1; a2 = an 2; : : : Xét n chẵn, n = 2m. Chia hai vế cho x m 6= 0:. Đặt t = x + 1 x ; jtj 2 đưa về phương trình bậc m = n 2 . Xét n lẻ, n = 2m + 1. Phương trình có nghiệm x = 1 nên phân tích ra thừa số (x + 1) và thừa số bậc 2m lại là phương trình quy hồi bậc chẵn như trên. Đôi khi ta mở rộng dạng quy hồi (quy hồi kèm tỉ lệ) với cách đặt t = x 1 ; t = x + a x x Phương trình bậc cao Xét phương trình a0x n + a1x n 1 + + an 1x + an = 0; a0 6= 0: (1.20) Nguyên tắc chung là biến đổi về dạng tích, đặt ẩn phụ để đưa về phương trình bậc thấp hơn. Đặc biệt Nếu tổng các hệ số bằng 0 thì có nghiệm x = 1. Nếu tổng đan dấu các hệ số bằng 0 thì có nghiệm x = 1 . Nhắc lại, nghiệm hữu tỉ nếu có thì có dạng x = q p với p j an và q j a0. Thế trực tiếp hoặc dúng sơ đồ Hoocne để thử nghiệm. Đôi khi phương trình bậc cao đối với biến x mà lại bậc thấp đối với tham số thì ta chuyển về phương trình theo ẩn là tham số đó.
  • 31. Dịch vụ viết thuê đề tài – KB Zalo/Tele 0917.193.864 – luanvantrust.com Kham thảo miễn phí – Kết bạn Zalo/Tele mình 0917.193.864 26 Bài toán 1.3.7. Giải các phương trình sau (a) 4x3 10x2 + 6x 1 = 0. (b) 8x3 36x + 27 = 0. Lời giải. (a) Ta có 4x 3 10x 2 + 6x 1 = 0 , 4x 3 2x 2 8x 2 + 4x + 2x 1 = 0 , 2x2 (2x 1) 4x(2x 1) + (2x 1) = 0 , (2x 1)(2x 2 4x + 1) = 0 , 2x 1 = 0 hoặc 2x2 4x + 1 = 0: Vậy nghiệm của phương trình là p x =1 ; x = 2 2 : 2 2 Bài toán 1.3.8. Tìm quan hệ giữa p và q để phương trình x 3 + px + q = 0 có thể viết dưới dạng x4 = (x2 ax + b)2 . Áp dụng kết quả đó để giải phương trình: x 3 18x + 27 = 0. Lời giải. Ta có x 4 (x 2 ax + b) 2 = m(x 3 + px + q): Suy ra a 2 + 2b = 0; 2ab = pm; 2a = m; b 2 = qm: Từ đó b = p, kéo theo p 2 = mq. Vậy m = p 2 . q Mặt khác a 2 = 2b ) a = m 2 ) m 4 2 = 8 ) p 4 + 8pq 2 = 0. Vậy quan hệ giữa p và q: p 3 + 8q 2 = 0. Ta có 4 2b = 2p ) m 2 = 8p. Từ đó p = q 2 x3 18x + 27 = 0 , x4 = (x2 + 6x 18) 2
  • 32. Dịch vụ viết thuê đề tài – KB Zalo/Tele 0917.193.864 – luanvantrust.com Kham thảo miễn phí – Kết bạn Zalo/Tele mình 0917.193.864 27 , (6x 18)(2x 2 + 6x 18) = 0 p , x = 3 hoặc x = 3 35 : 2 Bài toán 1.3.9. Giải và biện luận phương trình x 3 3x 2 + 3(a + 1)x (a + 1) 2 = 0: (1.21) Lời giải. Ta có x 3 3x 2 + 3(a + 1)x (a + 1) 2 = 0 tương đương với x 3 = 3x 2 + 3(a + 1)x (a + 1) 2 : Nếu a 6= 1 thì nhân hai vế với (a + 1), ta được x 3 (a + 1) = 3x 2 (a + 1) + 3(a + 1) 2 x (a + 1) 3 Cộng hai vế với x 3 ta được ax 3 = (x a 1) 3 . Từ đó ta được x a 1 = p3 , ax suy ra p a + 1 p3 2 x = a 3 a: p 3 + 1 = a Nếu a = 1 thì dễ thấy phương trình có hai nghiệm x1 = 0 và x2 = 3. Bài toán 1.3.10. Giải các phương trình sau (a) x4 2x3 6x2 + 16x 8 = 0; (b) x 4 + x 2 + 4x 3 = 0; Lời giải. (a) Ta có x 4 2x 3 6x 2 + 16x 8 = 0 , (x 2)(x 3 6x + 4) = 0 , (x 2)(x 2)(x 2 + 2x 2) = 0 , (x 2) 2 (x 2 + 2x 2) = 0: Vậy nghiệm của phương trình là x = 2, x = p . 1 3
  • 33. Dịch vụ viết thuê đề tài – KB Zalo/Tele 0917.193.864 – luanvantrust.com Kham thảo miễn phí – Kết bạn Zalo/Tele mình 0917.193.864 28 (b) Ta có x 4 + x 2 + 4x 3 = 0 , x 4 = x 2 4x + 3 , (x + 1) 2 = (x 2) 2 , (x2 + 1)2 (x 2) 2 = 0 , (x2 x + 3)(x2 + x 1) = 0 p Vậy nghiệm của phương trình là x = 1 5 . 2 Bài toán 1.3.11 (IMO 1973). Giả sử phương trình x 4 + ax 3 + bx 2 + ax + 1 = 0 có nghiệm. Tìm giá trị bé nhất của a 2 + b 2 . Lời giải. Gọi x0 là nghiệm của phương trình đã cho, tức là x0 4 + ax0 3 + bx0 2 + ax0 + 1 = 0 Từ đây ta có kết luận x0 6= 0. Chia hai vế cho x0 2 , ta có x0 2 + ax0 + b + a + 1 = 0: x0 x0 2 Phương trình này tương đương với x02 x 0 x0 2 + 1 + a x0 + 1 + b = 0: 1 1 2 Đặt y = x0 + với điều kiện jyj = jx0j+ j j 2. Ta có (y 2) + ay + b = 0 Suy x0 x0 ra (2 y 2 ) 2 j2 y 2 j = jay + bj p a2 + b2 p y2 + 1 ) a 2 + b 2 1 + y 2 Đặt t = y 2 , t 4. Ta chứng minh (2 t) 2 4 (1.22) 1 +t 5 Thật vậy, ta có (1.22) tương đương với 5(2 t) 2 4(1 +t), tức là 5t 2 24t + 16 0. Nhưng điều này đúng vì t 4. Như vậy giá trị bé nhất của a 2 + b 2 là 4=5.
  • 34. Dịch vụ viết thuê đề tài – KB Zalo/Tele 0917.193.864 – luanvantrust.com Kham thảo miễn phí – Kết bạn Zalo/Tele mình 0917.193.864 29 Bài toán 1.3.12 (Việt Nam 2002). Giải phương trình q p 4 3 10 3x = x 2 (1.23) Lời giải. Ta có biến đổi tương đương như sau p 8 x 2 q > 4 3 = x 2 , < 10 3x >4 3 p = (x 2) 2 10 3x : 8 x 2 > , < p x x 2 >3 10 3x = 42 : 8 x 2; 4x x 0 , > 2 2 < 9(10 3 x) = (4x x ) > : 8 x 2; 0 x 4 , > 4 3 2 + 27x 90=0: <x 8x + 16x > : 8 2 x 4 , > 4 3 2 + 27x 90=0: <x 8x + 16x > Bây giờ ta sẽ giải phương trình : x 4 8x 3 + 16x 2 + 27x 90=0: (1.24) Bằng cách thử trực tiếp ta thấy x = 3 là một nghiệm nên phương trình (1.24) được viết lại thành (x 3)(x + 2)(x 2 7x + 15) = 0: Vậy phương trình có nghiệm duy nhất x = 3. Bài toán 1.3.13 (Việt Nam 1991). Giải phương trình x 3 3x 2 8x + 40 = 8 p4 : (1.25) 4x + 4
  • 35. Dịch vụ viết thuê đề tài – KB Zalo/Tele 0917.193.864 – luanvantrust.com Kham thảo miễn phí – Kết bạn Zalo/Tele mình 0917.193.864 30 p Lời giải. Từ phương trình x 3 3x 2 8x + 40 = 8 4 4x + 4 ta có điều kiện x 1. Áp dụng bất đẳng thức AM-GM ta có 8 p 4 = 4 p 4 x + 13: 4x + 4 4:4:4:(x + 1) Do đó x 3 3x 2 8x + 40 x + 13 , x 3 3x 2 9x + 27 0 , (x + 3)(x 2 6x + 9) 0 , (x + 3)(x 3) 2 0: Vì x1 nên (x 3) 2 0, suy ra x = 3. Thử lại ta thấy đúng. Vậy phương trình có nghiệm duy nhất x = 3. Bài toán 1.3.14. Chứng minh rằng r r q q (a) x = 3 a + a+1 8a 1 + 3 a a+1 8a 1 với a 1 là số tự nhiên 3 3 3 3 8 (b) p 3 + p 3 là số vô tỉ. 2 4 Lời giải. (a) Áp dụng hằng đẳng thức (u + v)3 = u3 + v3 + 3uv(u + v) ta có x 3 = 2a + (1 2a)x , x 3 + (2a 1)x 2a = 0 , (x 1)(x 2 + x + 2a) = 0: Xét đa thức bậc hai x 2 + x + 2a có D = 1 8a 0 q q Khi a = 1 8 , ta có x = 3 1 8 + 3 1 8 = 1 Khi a > 1 8 , ta có 1 8a âm nên đa thức x 2 + x + 2a có nghiệm thực duy nhất x = 1. Vậy với a 1 thì 8 s s x = a + a 3 1r a a 3 1r 8a3 1 + 8a3 1 3 + 3 +
  • 36. Dịch vụ viết thuê đề tài – KB Zalo/Tele 0917.193.864 – luanvantrust.com Kham thảo miễn phí – Kết bạn Zalo/Tele mình 0917.193.864 31 là một số tự nhiên. p p (b) Đặt x = 3 2 + 3 4. Ta có p p p x 3 = 2 + 4 + 3 3 8( 3 2 + 3 4) Tức là x 3 6x 6: Giả sử x 2 Q. Do hệ số bậc cao nhất của đa thức là 1 nên x là số nguyên. Ta có p p 2 < 3 2 + 3 4 < 4 nên x = 3. Do đó x 3 6x 6 = 3, vô lý. Vậy x là số vô tỷ. Bài toán 1.3.15 (Việt Nam 1984). Tìm đa thức theo x có bậc bé nhất với hệ số nguyên, biết một nghiệm là p + p 3 . 2 3 Lời giải. Đặt a = p + p 3 ta có 2 3 a 2 = 2 + 2 p : p 3 + p 3 2 3 9 và p p3 p : p 3 a 3 = 2 + 3 + 3 (1.26) 2 + 6 3 2 9 Ta rút ra p3 = a p ; 3 2 p 3 = a 2 2 2 p (a p ) = a 2 + 2 2 p a: 9 2 2 2 Thay vào (1.26) ta có a3 = 2 p + 6(a p ) + 3 p (a2 + 2 2 p a) + 3; 2 2 2 2 a 3 + 6a 3 = p (3a 2 + 2): 2 Bình phương hai vế ta thấy a là nghiệm của đa thức x 6 6x 4 6x 3 + 12x 2 36x + 1 Bằng phép đồng nhất hệ số ta chứng minh đa thức trên không phân tích được thành hai đa thức bậc thấp hơn có hệ nguyên nên đa thức trên chính là đa thức có bậc bé nhất thỏa đề bài.
  • 37. Dịch vụ viết thuê đề tài – KB Zalo/Tele 0917.193.864 – luanvantrust.com Kham thảo miễn phí – Kết bạn Zalo/Tele mình 0917.193.864 32 1.4 Đạo hàm của đa thức. Định lý Taylor Đạo hàm của một hàm số là một khải niệm thuộc nhánh Giải tích toán học. Trong mục này chúng tôi sẽ trình bày ngắn gọn về đạo hàm của hàm số đa thức. Định nghĩa 1.4.1. Cho f (x) 2 R[x] và deg f (x) = n, anx n + an 1x n 1 + : : : + a1x + a0 với an 6= 0 Đa thức nanx n 1 + (n1)an 1x n 2 + : : : + 2a2x + a1 được gọi là đạo hàm cấp một của đa thức f (x) (hoặc ngắn gọn là đạo hàm của đa thức f (x)), và được ký hiệu là f 0 (x). Bằng quy nạp, ta định nghĩa đạo hàm cấp k của đa thức f (x) là đạo hàm của đạo hàm cấp k 1. Ta ký hiệu đạo hàm cấp 2, 3 và cấp k tổng quát lần lượt là f 00 (x), f 000 (x), và f (k) (x). Với mọi x0 2 R, biểu thức sau đây được gọi là khai triển Taylor của đa thức thực f (x) f (x) = f (x0)+ f 0(x0) (x x0)+: : :+ f (k)(x0) (x x0)k +: : :+ f (n)(x0) (x x0)n: 1! k! n! Từ định nghĩa này ta có nếu deg f = n thì deg f 0 = n 1; deg f 00 = n 2;:::; deg f (k) = n k với 1 k n; deg f (x) Bây giờ ta xét một số bài toán về đạo hàm hàm và khai tham khảo [6] trong trình bày. = 0: triển Taylor. Chúng tôi Bài toán 1.4.2 (Ba Lan 1979). Cho đa thức P(x) có bậc n > 1 và có n nghiệm thực x1; x2; x3; : : : ; xn phân biệt. Chứng minh rằng 1 + 1 +:::+ 1 = 0: P 0 (x1) P 0 (x2) P 0 (xn)
  • 38. Dịch vụ viết thuê đề tài – KB Zalo/Tele 0917.193.864 – luanvantrust.com Kham thảo miễn phí – Kết bạn Zalo/Tele mình 0917.193.864 33 Lời giải. Đặt P(x) = a(x x1)(x x2): : : (x xn); a 6= 0 ta suy ra n P 0 (x) = P1(x) + P2(x) + : : : + Pn(x) với Pi(x) = Õ (x x j): j=1; j6=i Ta thấy Pi(x ) = 0 với mọi j = i kéo theo P 0 (x j ) = P (x ) = 0 với mọi j = 1; 2; : : : ; n j 6 j j 6 Xét đa thức n P (x) F(x) = å i 1 i=1 P 0 (xi) có bậc không vượt quá n 1. Với i = 1; 2; : : : ; n ta có F(xi) = Pi(xi) 1 = 0 suy ra P 0 (xi) F(x) có n nghiệm phân biệt. Vậy F(x) = 0. Mặt khác hệ số của F(x) đối với x n 1 bằng 0 nên a + a +:::+ a = 0: P 0 (x1) P 0 (x2) P 0 (xn) Như vậy 1 + 1 +:::+ 1 = 0: P 0 (x1) P 0 (x2) P 0 (xn) Bài toán 1.4.3 (CHDCLB Đức 1974). (a) Chứng minh rằng không tồn tại đa thức P(x) để với mọi x 2 R ta có các bất đẳng thức P 0 (x) > P 00 (x); (1.27) P(x) > P 00 (x): (1.28) (b) Khẳng định trên còn đúng không nếu thay đổi bất đẳng thức (1.27) bằng bất đẳng thức P(x) > P 0 (x): (1.29)
  • 39. Dịch vụ viết thuê đề tài – KB Zalo/Tele 0917.193.864 – luanvantrust.com Kham thảo miễn phí – Kết bạn Zalo/Tele mình 0917.193.864 34 Lời giải. (a) Nếu P(x) là hằng số thì P 0 (x) = P 00 (x) = 0, và bất đẳng thức (1.27) không thỏa mãn. Giả sử deg P(x) = n 1 khi đó nếu n lẻ thì deg(P(x) P 00 (x)) = n là số lẻ, từ đó P(x) P 00 (x) 0 với ít nhất một điểm x 2 R. Như vậy đối với đa thức P(x) không thỏa mãn hoặc bất đẳng thức (1.28) hoặc bất đẳng thức (1.27). Vậy (a) được chứng minh xong. (b) Chọn P(x) = x2 + 3. Khi đó với x 2 R ta có P(x) P0(x) x2 2x + 3 > 0 và P(x) P00(x) x2 + 1 > 0 nghĩa là khẳng định trên không còn đúng nữa. Bài toán 1.4.4 (Việt Nam 1986). Cho f 2 R[x] có deg f = n và f (k) = 2k, với k = 0; 1; 2; : : : ; n. Tính f (n + 1). Lời giải. Xét đa thức g(x) = 1 + x + x(x 1) + : : : + x(x 1)(x 2): : : (x n + 1): 1! 2! n! Khi đó deg g = n và n g(k) = åCk i = 2 k = f (k) i=0 với n + 1 giá trị nên f g. Do đó n f (n + 1) = g(n + 1) = åCi + = 2n+1 1: n 1 i=0 Bài toán 1.4.5 (Singapore 1978). Cho đa thức P(x) bậc n và hai số a < b thỏa mãn P(a) < 0; P 0 (a) 0; P(a) 0; : : : ; ( 1) n P (n) (a) 0; P(b) > 0; P 0 (b) 0; P(b) 0; : : : ; P (n) (b) 0: Chứng minh các nghiệm thực của P(x) thuộc (a; b).
  • 40. Dịch vụ viết thuê đề tài – KB Zalo/Tele 0917.193.864 – luanvantrust.com Kham thảo miễn phí – Kết bạn Zalo/Tele mình 0917.193.864 35 Lời giải. Khai triển Taylor ta có P(x) = P(b) + P0(b) (x b) + P(b) (x b)2 + : : : + P(n)(b) (x b)n: 1! 2! n! Nếu x b thì P(x) > 0 suy ra P(x) không có nghiệm x b. Tương tự P(x) = P(a) +P 0 (a) (x a) + P(a)(x a)2 + : : : + P (n) (a)(x a)n 2! 1! n! = P(a) + P 0 (a) (a x) + P(a)(a x) 2 + : : : + ( 1) n (a) (a x) n 1! 2! n! Nếu x < a thì P(x) < 0 suy ra P(x) không có nghiệm x a Vậy các nghiệm phải thuộc (a; b). Bài toán được chứng minh xong. Ta gọi ước lượng về nghiệm ở trên là ước lượng Newton.
  • 41. Dịch vụ viết thuê đề tài – KB Zalo/Tele 0917.193.864 – luanvantrust.com Kham thảo miễn phí – Kết bạn Zalo/Tele mình 0917.193.864 36 Chương 2 Đa thức bất khả quy 2.1 Đa thức bất khả quy Định nghĩa 2.1.1. Cho đa thức f 2 Z[x]. Ta gọi đa thức f là bất khả quy trên Z[x] nếu f không phân tích được thành tích hai đa thức thuộc Z[x] với bậc lớn hơn hay bằng 1. Trường hợp f là một đa thức hữu tỷ, ta định nghĩa hoàn toàn tương tự. Mối quan hệ về tính bất khả quy trên Z[x] và Q[x] được phát biểu như sau: Định lí 2.1.2. Nếu đa thức f 2 Z[x] bất khả quy trên Z[x] thì f cũng bất khả quy trên Q[x]. Để chứng minh định lý này, ta cần bổ đề sau đây. Trước hết, ta gọi đa thức f 2 Z[x] là nguyên bản nếu các hệ số nguyên tố cùng nhau. Bổ đề 2.1.3 (Bổ đề Gauss). Tích của hai đa thức nguyên bản là một đa thức nguyên bản. Chứng minh. Cho hai đa thức nguyên bản f (x) = a0x n + a1x n 1 + : : : + an; g(x) = b0x m + b1x m 1 + : : : + bm:
  • 42. Dịch vụ viết thuê đề tài – KB Zalo/Tele 0917.193.864 – luanvantrust.com Kham thảo miễn phí – Kết bạn Zalo/Tele mình 0917.193.864 37 Khi đó f (x)g(x) = c0x n+m + c1x n+m 1 + : : : + cn+m: Giả sử đa thức tích f (x)g(x) không nguyên bản. Khi đó tồn tại một số nguyên tố p là ước chung của các hệ số c0; : : : ; cn+m Vì f nguyên bản nên gọi ai là số đầu tiên mà ai .. . p. Vì g nguyên bản nên gọi b j là số đầu tiên mà b j .. . p. Bằng cách xét hệ số theo lũy thừa x i+ j ta có hệ số tương ứng không chia hết cho p. Điều này vô lý. Vậy f (x)g(x) là đa thức nguyên bản. Chứng minh Định lý 2.1.2. Cho f 2 Z[x] là một đa thức bất khả quy. Giả sử f khả quy trên Q[x], tức là ta có biểu diễn f (x) = f1(x) f2(x) với f1; f2 2 Q[x], có bậc lớn hơn hoặc bằng 1. Đặt f1(x) = a1 g1(x); f2(x) = a2 g2(x) b1 b2 voi ai tối giản và g1(x), g2(x) nguyên bản. Khi đó b i a 1 a 2 p f (x) = f1 (x) f2 (x) = g1 (x)g2(x) = g1 (x)g2(x) với (p; q) = 1: b1b2 q Do đó f 2 Z[x] nên mọi hệ số của khai triển tích g1(x)g2(x) đều là bội số của q. Suy ra đa thức tích g1(x)g2(x) không nguyên bản. Điều này trái với kết quả của Bổ đề Gauss. Vậy f bất khả quy trên Q[x]. 2.1.1 Đa thức với hệ số thực và phức Cho một đa thức với hệ số thực thì chưa chắc đa thức đó có nghiệm trong trường số thực, cụ thể đa thức x2 + 1 không có nghiệm trong trường số thực. Dưới đây ta sẽ thấy mọi đa thức bậc n với hệ số phức có đúng n nghiệm phức. Để chứng minh ra hãy đưa vào các bổ đề sau đây: Bổ đề 2.1.4. Mọi đa thức với hệ số thực có bậc lẻ có ít nhất một nghiệm thực.
  • 43. Dịch vụ viết thuê đề tài – KB Zalo/Tele 0917.193.864 – luanvantrust.com Kham thảo miễn phí – Kết bạn Zalo/Tele mình 0917.193.864 38 Chứng minh. Giả sử f (x) = anxn + an 1xn 1 + :::: + a0; an 6= 0 và n lẻ. Qua giáo trình giải tích ta biết rằng với những giá trị dương và âm của x, khá lớn về giá trị tuyệt đối, hàm số f (x) có các dấu trái nhau. Vậy có những giá trị thực của x, a và b chẳng hạn, sao cho. f (a) < 0; f (b) > 0: Mặt khác hàm số f (x) là liên tục, vì vậy có một giá trị c của x, nằm giữa a và b, sao cho f (x) = 0. Bổ đề 2.1.5. Mọi đa thức bậc hai ax 2 + bx + c, với hệ số phức, bao giờ cũng có hai nghiệm phức. Bổ đề 2.1.6. Mọi đa thức bậc lớn hơn 0 với hệ số thực có ít nhất một nghiệm phức. Định lí 2.1.7. Mọi đa thức bậc lớn hơn 0 với hệ số phức có ít nhất một nghiệm phức. Chứng minh. Giả sử f (x) là một đa thức bậc n > 0 f (x) = a0 + a1x + :::: + anx n với hệ số phức. Đặt f(x) = a0 + a1x + :::: + anxn với các ai là các liên hợp của các ai với i = 0; : : : ; n. Xét đa thức g(x) = f (x) f (x): Ta có g(x) = b0 + b1x + :::: + b2nx 2n Với bk = å aia j; k = 0; 1; ::::; 2n: i+ j=k
  • 44. Dịch vụ viết thuê đề tài – KB Zalo/Tele 0917.193.864 – luanvantrust.com Kham thảo miễn phí – Kết bạn Zalo/Tele mình 0917.193.864 39 Vì b k = å a i a j = b k i+ j=k nên các hệ số bk là thực. Theo Bổ đề 2.1.6 g(x) có ít nhất một nghiệm phức z = s + it, g(z) = f (z) f (z) = 0: Do đó hoặc f (x) = 0 hoặc f (z) = 0. Nếu f (z) = 0, f (z) = a0 + a1z + :::: + anzn = 0: thì a0 + a1z + : : : anz n = a0 + a1z + : : : + anz n = 0; tức là f (z) = 0. Như vậy hoặc z hoặc z là nghiệm của f (x). Hệ quả 2.1.8. Các đa thức bất khả quy của vành C[x], C là trường số phức, là các đa thức bậc nhất. Chứng minh. Các đa thức bậc nhất là bất khả quy. Giả sử f (x) là một đa thức của C[x] có bậc lớn hơn 1. Theo Định lí 2.1.7, f (x) có một nghiệm phức c. Vậy f (x) có một ước thực sự x c, do đó f (x) không bất khả quy. Hệ quả 2.1.9. Mọi đa thức bậc n > 0 với hệ số phức có n nghiệm phức. Hệ quả 2.1.10. Các đa thức bất khả quy của R[x], R là trường số thực, là các đa thức bậc nhất và các đa thức bậc hai ax 2 + bx + c với biệt số b 2 4ac < 0. Chứng minh. Các đa thức bậc nhất và các đa thức bậc hai với biệt số âm rõ ràng là những đa thức bất khả quy của R[x]. Giả sử p(x) là một đa thức bất khả quy của R[x] với bậc lớn hơn một. Vậy p(x) không có nghiệm thực. Theo Định lí 2.1.7, p(x) có một nghiệm phức z và p(x) chia hết cho đa thức bậc hai với hệ số thực. g(x) = x 2 (z + z)x + zz:
  • 45. Dịch vụ viết thuê đề tài – KB Zalo/Tele 0917.193.864 – luanvantrust.com Kham thảo miễn phí – Kết bạn Zalo/Tele mình 0917.193.864 40 Đa thức g(x) không khả nghịch và là ước của phần tử bất khả quy p(x), vậy g(x) phải là liên kết của p(x), tức là p(x) = ug(x); 0 6= u 2 R: Ta có điều cần chứng minh. 2.1.2 Đa thức bất khả quy của vành Q[x] Đối với trường số thực R và trường số phức C, vấn đề xét xem một đa thức đã cho của vành R[x] hay C[x] có bất khả quy hay không rất đơn giản, nhưng trong vành Q[x] với Q là trường số hữu tỉ thì vấn đề phức tạp hơn nhiều. Đối với các đa thức bậc hai và ba của Q[x], việc xét xem có bất khả quy hay không được đưa về việc tìm nghiệm hữu tỉ của đa thức đó. Các đa thức bậc hai và bậc ba của Q[x] là bất khả quy khi và chỉ khi chúng không có nghiệm hữu tỉ. Đối với các đa thức bậc lớn hơn ba thì vấn đề phức tạp hơn nhiều. Chẳng hạn đa thức x 4 + 2x 2 + 1 = (x 2 + 1) 2 rõ ràng không có nghiệm hữu tỉ nào, nhưng nó có một ước thực sự x 2 + 1 , vậy không phải là bất khả quy. Ta đã biết, mọi đa thức f (x) với hệ số hữu tỉ đều có thể viết dưới dạng f (x) = b 1 g(x) trong đó b là một số nguyên khác 0, g(x) là một đa thức với hệ số nguyên. Trong vành Q[x], f(x) và g(x) là liên kết vậy f (x) là bất khả quy khi và chỉ khi g(x) là bất khả quy. Do đó tiêu chuẩn Eisenstein mà ta đưa ra dưới đây để xét một đa thức của Q[x] có bất khả quy hay không là tiêu chuẩn cho các đa thức với hệ số nguyên. Bổ đề 2.1.11. Nếu f (x) là một đa thức với hệ số nguyên có bậc lớn hơn 0 và f (x) không bất khả quy trong Q[x], thì f (x) phân tích được thành một tích những đa thức bậc khác 0 với hệ số nguyên. Trong nghiên cứu các đa thức bất khả quy, Tiêu chuẩn Eisenstein sau đây là đặc biệt quan trọng.
  • 46. Dịch vụ viết thuê đề tài – KB Zalo/Tele 0917.193.864 – luanvantrust.com Kham thảo miễn phí – Kết bạn Zalo/Tele mình 0917.193.864 41 Định lí 2.1.12 (Tiêu chuẩn Eisenstein). Cho f (x) 2 Z[x], deg f (x) = n, f (x) = a0x n + a1x n 1 + : : : + an: Nếu có số nguyên tố p thỏa mãn ba điều kiện (1) a0 không chia hết cho p; (2) a1; a2; : : : ; an chia hết cho p; (3) an không chia hết cho p 2 . thì đa thức f (x) bất khả quy trên Q[x]. Chứng minh. Giả sử f (x) có những ước thực sự trong Q[x] theo Bổ đề 2.1.11, f (x) có thể viết f(x) = g(x)h(x); trong đó g(x) = b0 + b1x + : : : + brxr; h(x) = c0 + c1x + : : : +s xs; bi 2 Z; ci 2 Z; 0 < r < n; 0 < s < n: Ta có a0 = b0c0; a1 = b1c0 + b0c1; : : : ak = bkc0 + bk 1c1 + : : : + b0ck; an = brcs: Theo giả thiết p chia hết a0 = b0c0; vậy vì p là nguyên tố nên hoặc p chia hết cho b0 hoặc p chia hết cho c0. Giả sử p chia hết b0 thế thì p không chia hết c0, vì nếu thế thì p 2 sẽ chia hết a0=b0c0, trái với giả thiết, p không thể chia hết mọi hệ số
  • 47. Dịch vụ viết thuê đề tài – KB Zalo/Tele 0917.193.864 – luanvantrust.com Kham thảo miễn phí – Kết bạn Zalo/Tele mình 0917.193.864 42 của g(x), vì nếu thế thì p sẽ chia hết an = brcs, trái với giả thiết. Vậy giả sử bk là hệ số đầu tiên của g(x) không chia hết cho p. Ta hãy xét ak = bkc0 + bk 1c1 + : : : + b0ck; trong đó ak; bk 1; : : : ; b0 đều chia hết cho nguyên tố, ta suy ra hoặc bk chia hết cho giả thiết về bk và c0. p. Vậy bkc0 phải chia hết cho p. Vì p là p, hoặc c0 chia hết cho p, mâu thuẫn với 2.2 Một số bài toán điển hình Bài toán 2.2.1 (IMO 1993). Cho n 2 N và n > 1. Chứng minh rằng đa thức f (x) = xn + 5xn 1 + 3 bất khả quy trên Z[x]: Lời giải. Với n = 2 có f (x) = x 2 + 5x + 3 thì bất khả quy trên Z[x]. Xét n 3. Giả sử f (x) = g(x)h(x) với g(x); h(x) 2 Z[x] và có bậc lơn hơn hoặc bằng 1. Do deg g(x) +deg h(x) = n 3 nên suy ra trong hai số deg g(x) và deg h(x) có một số lơn hơn 1. Vì f (0) = 3 là số nguyên tố nên hoặc jg(0)j = 1 hoặc jh(0)j = 1. Giả sử g(x) = x k + b1x k 1 + : : : + bk với k > 1 và jg(0)j = 1. Gọi a1; a2; : : : ; ak là các nghiệm (nói chung là nghiệm phức) của g(x). Khi đó ta có g(x) = (x a1)(x a2): : : (x ak): Vì jg(0)j = 1 nên ja1a2 : : : akj = 1: (2.1)
  • 48. Dịch vụ viết thuê đề tài – KB Zalo/Tele 0917.193.864 – luanvantrust.com Kham thảo miễn phí – Kết bạn Zalo/Tele mình 0917.193.864 43 Do g(ai) = 0 nên f (ai) = 0, với i 2 f1; : : : ; kg. Nhân các đẳng thức đó lại và sử dụng (2.1) ta được j(a1 + 5)(a2 + 5): : : (ak + 5)j = 3 k (2.2) Mặt khác ta có g( 5) = j(a1 + 5)(a2 + 5): : : (ak + 5)j và 3 = f ( 5) = g( 5)h( 5) nên j(a1 + 5)(a2 + 5): : : (ak + 5)j nhận giá trị 1 hoặc 3. Điều này trái với (2.2) vì k > 1. Từ đó ta có ta có điều phải chứng minh. Bài toán 2.2.2 (Dự tuyển IMO). Cho đa thức f (x) = Õ n = (x ai) 2 với n 3 và i 1 a1; a2; : : : an là các số nguyên đôi một khác nhau. Chứng minh rằng nếu f (x) khả quy trên Z[x] thì n = 3. Lời giải. Giả sử f (x) = g(x)h(x) với g(x); h(x) 2 Z[x] và deg g = p q = deg h 1. Ngoài ra, giả sử các hệ số bậc cao nhất của g và h đều bằng 1. Trong phép đặt cho x = ai ta sẽ nhận được f (ai) = g(ai)h(ai) = 2: Như vậy ta có (g(ai); h(ai)) 2 f( 2; 1); (2; 1); (1; 2); ( 1; 2)g Vậy với mọi i, ta có g(ai) + h(ai) = +1 với ít nhất n 2 giá trị của i, hoặc g(ai) + h(ai) = 1 với ít nhất n 2 giá trị của i. Giả sử g(ai) + h(ai) =1 với k giá trị của i 2 f1; : : : ; ng. Rõ ràng k < n (vì đa thức g(x) + h(x) 1 6= 0 và có bậc nhỏ hơn thực sự n. Giả sử k n 2 2, vì nếu k < n 2 thì xét đa thức g(x) + h(x) + 1. Giả sử g(ai) + h(ai) = 1 với i 2 f1; : : : ; kg, nếu không như vậy thì ta đặt lại thứ tự của chỉ số. Suy ra
  • 49. Dịch vụ viết thuê đề tài – KB Zalo/Tele 0917.193.864 – luanvantrust.com Kham thảo miễn phí – Kết bạn Zalo/Tele mình 0917.193.864 g(x) + h(x) 1 = (x a1)(x a2): : : (x ak)w(x) với w(x) 2 Z[x]:
  • 50. Dịch vụ viết thuê đề tài – KB Zalo/Tele 0917.193.864 – luanvantrust.com Kham thảo miễn phí – Kết bạn Zalo/Tele mình 0917.193.864 44 Cho x = ak+1 với chú ý g(ak+1) + h(ak+1) = 1 ta được 2 = (ak+1 a1)(ak+1 a2): : : (ak+1 ak)w(ak+1): Thế nhưng trị tuyệt đối của một tích gồm ít nhất là bốn thừa số nguyên tố khác nhau và khác 0 thì luôn luôn lớn hơn 2. Vậy nên k 3 và dẫn đến 2 k 3. Xét k = 3. Nếu n > 4 thì ta có (a4 a1)(a4 (a5 a1)(a5 a2)(a4 a2)(a5 a3)w(a4) = a3)w(a5) = 2; 2 Do đó fa1; a2; a3g fa4 1; a4 2; a4 2g. Nhưng a5 > a4 > a3 nên ja5 a3j 3 với mọi i 2 f1; 2; 3g. Điều này vô lý. Nếu n = 4 thì g(x) + h(x) 1 = (x a1)(x a2)w(x) do deg w 1 do deg g 3 và deg h 3. Suy ra (ai i = 3; 4. Nếu deg w = 1 thì deg g(x) = 3, h(x) = x a và a1)(ai a2)w(ai) = 2 với 0 = f (a) = (aa1)(a a2)(a a3)(a a4) = 2: Vậy w(x) là đa thức hằng. Do đó deg g = deg h = 2, w(x) 2 và w(a3) = w(a4) = 2. Từ đó 1 = (a3 a1)(a3 a2) và fa1; a2g fa3 1; a3 + 1g: Tương tự ta cũng có 1 = (a4 a1)(a4 a2) nên fa1; a2g fa4 1; a4 + 1g. Do đó a4 = a3 vô lý. Tóm lại, chỉ có n = 3 thỏa mãn yêu cầu của bài toán. Bài toán 2.2.3. Cho n số ai 2 Z. Chứng minh rằng đa thức P(x) = (x a1)(x a2): : : (x an) 1 bất khả quy trong Z[x].
  • 51. Dịch vụ viết thuê đề tài – KB Zalo/Tele 0917.193.864 – luanvantrust.com Kham thảo miễn phí – Kết bạn Zalo/Tele mình 0917.193.864 45 Lời giải. Giả sử ta có phan tích P(x) = G(x)H(x) trong Z[x] và deg G < n, deg H < n. Ta có P(ai) = G(ai)H(ai) = 1: Vậy G(ai) và H(ai) nhận hai giá trị 1 nên G(ai)+H(ai) = 0 Khi đó đa thức G(x)+H(x) có deg(G+H) < n mà có n nghiệm. Suy ra G(x) + H(x) 0, tức là G(x) = H(x). Do đó P(x) = [H(x)] 2 . So sánh hệ số của xn của hai vế thì vế trái lớn hơn không, vế phải nhỏ hơn không, điều này vô lý, nên ta có điều phải chứng minh. Bài toán 2.2.4. Cho n số ai 2 Z. Chứng minh rằng đa thức P(x) = (x a1) 2 (x a2) 2 : : : (xan) 2 + 1 bất khả quy trong Z[x]. Lời giải. Vì P(x) > 0 với mọi x nên đa thức P(x) vô nghiệm. Suy ra hai nhân tử G(x) và H(x) cũng vô nghiệm nên không đổi dấu. Giả sử G(x) và H(x) dương với mọi x. Ta có H(ai)G(ai) = 1 nên suy ra H(ai) = 1 và G(ai) = 1. Nếu deg H < n suy ra H(x) 1 có n nghiệm. Kéo theo rằng H(x) 1. Điều này vô lý, nên deg H = deg G = n, suy ra H(x) 1 = A(x a1): : : (x an); G(x) 1 = B(x a1): : : (x an): Thế vào ta có (x a1) 2 (x a2) 2 : : : (xan) 2 + 1 = [1 + A(x a1): : : (x an)][1 + B(x a1): : : (x an)]: So sánh hệ số của x 2n thì ta có 1 = AB. So sánh hệ số tự do a 2 1a 2 2 : : : a 2 n + 1 = 1 + (A + B)a1a2 : : : an + a 2 1a 2 2 : : : a 2 n: Suy ra A + B = 0. Do đó 1 = AB = A 2 . Điều này vô lý. Vậy bài toán được chứng minh.
  • 52. Dịch vụ viết thuê đề tài – KB Zalo/Tele 0917.193.864 – luanvantrust.com Kham thảo miễn phí – Kết bạn Zalo/Tele mình 0917.193.864 46 Chương 3 Một số chủ đề khác